Ευκλειδης Β 61

84
Ελληνική Μαθηματική Εταιρεία

description

Ευκλειδης Β 61

Transcript of Ευκλειδης Β 61

Page 1: Ευκλειδης Β 61

Ελληνική Μαθηματική Εταιρεία

Page 2: Ευκλειδης Β 61

MAPI.A fΥΣfλθΙΟΥ • Ει'ΙΕΥθΕΡ!ΟΣ ΠΡΟΤΟΠΜΑί

ΜΕθΟΔΟΛΟΓΙΑ

Μεθοδολογία Άλγεβρας Α' Ενιαίου Λυκείου

Μ. Ευσταθίου, Ε. Πρωτοπαπάς

ΜΕθΟΔΟΛΟΓΙΑ

Μεθοδολογία Άλγεβρας Β' Ενιαίου Λυκείου

Γ ενικής Παιδείας

Ε. Πρωτοπαπάς

ΕΚΔΟΕΕΙΕ ΠΑΤΑΚΗ

πρωτοπορ-ία tπην εmαίδεvση

I ' ' ! t _)

,. I / ι ... /

; i I �_) _/,·

i j 'ι

I I

/

ΕΚΠΑΙΔΕΠΙΚΑ ΒΙΒΛΙΑ yιa μαΒπτίς και εκπαιδευτικοιίς

Γεωμετρία Α' Ενιαίου Λυκείου Γ. Βιδάλης, Β. Γ κιμίσης

μαθηματικά Γ' Ενιαίου Λυκείου

� Uli.Ottlt φ ιuι:rΛΚΗ

Μαθηματικά Γ' Ενιαίου Λυκείου

Γ ενικής Παιδείας

Μ. Τ σιλπιρίδης

Γεωμετρία Β' Ενιαίου Λυκείου

Γ ενικής Παιδείας

Ε. Πρωτοπαπάς

Μαθηματικά Γ' Ενιαίου Λυκείου

Θετικής και Τεχνολογικής κατεύθυνσης

Α' τόμος: Ε. Πρωτοπαπάς Β' τόμος: Ε. Πρωτοπαπάς, Σ. Γκούμας

σ ε: ό ι4a. τ α. b ι b ι4 ι ο n w ι4 ε:ίa. ΒΙΒΛΙΟΠΩΛΕΙΟ ΠΑΤΑΚΗ: ΑΚΑΔΗΜΙΑΣ 65, I 06 78 ΑΘΗΝΑ. ΤΗΛ. 210.38.11.850

KENTPIKH ΔΙΑΘΕΣΗ: ΕΜΜ . ΜΠΕΝΑΚΗ 16, I 06 78 ΑΘΗΝΑ. ΤΗΛ. 210.38.31.078

ΥΠΟΚ/ΜΑ: Ν. ΜΟΝΑΠΗΡΙΟΥ 122. 563 34 ΘΕΣΣΑΛΟΝΙΚΗ, ΤΗΛ. 2310.70.63.54

Web site: http://www.patakis.gr - e-mail: [email protected]. [email protected]

Page 3: Ευκλειδης Β 61

ΕΛΛΗΝΙΚΗ ΜΑΘΗΜΑΤΙΚΗ ETAIPEIA Τεύχος 60 ο Ιούλιος ο Αύyουστος ο Σεπτέμβριος 2006ο Έτος Αθ' ο Ευρώ: 3,50

eomail: [email protected] www.hms.gr

ΜΑΘΗΜΑΤΙΚΟ ΠΕΡΙΟΔΙΚΟ ΓΙΑ ΤΟ ΛΥΚΕΙΟ

ΠΕΡΙΕΧΟΜΕΝΑ ./ Γράμμα από το Δ.Σ. της Ε.Μ.Ε • . . . . ο • • • • • • • • • • ο • • • • • • • • • • • • • • • • • • • • • • • • • 2 ./ Αθανάσιος Φωκάς . . . . . . . . . . . . . . . . . . . . . . . . . . . . . . . . . . . . . . . . . . . . . . . . . . 3 ./ Ιστορικές Μαθηματικές Αναφορές - γωνία της Ιστορίας των Μαθηματικών . . . . . . 5 ./ Το Βήμα του Ευκλείδη . . . . . . . . . . . . . ο • • • • • • • • • • • • • • • • • • ο ο • • • • • • • • • • • • 11 ./ Homo· Mathematicus . . . . . . . . . . . . . . . . . . . . . . . . . . . . . . . . . . . . . . . . . . . . . . . . . . . . . . . . . . 22

Μαθη,μ,οτ·ικά Α' Τάξης ./ Άλγεβρα

./ Γεωμετρία

• • • • • • • • • • • • • • • • • • • • • • • • • • ο • • • • • • • • • ο • • • • • • • • • • • • • • • • • • 26 ................................................................... 35

Μα:θιημ:ατικά Β.' Τάξη;ς ./ Άλγεβρα . . ο • • • • • • • • • • • • • • • • • • • • • • • • • • • • • • • • • • • • • • • • • • • • • • • • • • • • • 44 ./ Γεωμετρία . . . . . . . . . . . . . . . . . . . . . . . . . . . . . . . . . . . ο • • • • • • • • • • • • • • • • • • • • 49 ./ Κατεύθυνση 58

Μαθ,ηματι,κά Γ" Τά.ξη:ς ./ Μαθηματικά Κατεύθυνσης . . . . . . . . . . . . . . . . . . . . . . . . . . . . . . . . . . . . . . . . . . 62 ./ Μαθηματικά Γενικής Παιδεία . . . . . . . . . . . . . . . . . . . . . . . . . . . . . . . . . . . . . . . . . 68

./ Ο Ευκλείδης προτείνει ••• Ευκλείδη και ••• Διόφαντο . . . . . . . . . . . . . . . . . . . . . . . 7 4 ./ Τα Μαθηματικά μας Διασκεδάζουν . ο ο • • • • • • • • • • • • • • • • • • • • • • • • • • • • • • • • • 79

·······························································�··············

ΕΚΔΟΣΗ ΤΗΣ ΜΑΘΗΜΑτJΚΗΣ ΠΑΙΡΕΙΑΣ

ΠΑΝΕΠΙΠΗΜΙΟΥ 34 ο 106 79 ΑΘΗΝΑ Τηλ.: 210 3617784 ο 3616532 Fox:2103641025 Εκδότης: Εξαρχάκος Θcόδωρος Διcuθuντής: Τuρλής Ιω6ννης Κωδικός ΕΛ.ΤΑ.: 2055 ISSN: 1105-7998

Επιμέλεια 'Εκδοσης: Κυριακόπουλος Αντώνης Ευσταθίου Βαγγέλης

Εκτελεστική Γραμματεία Πρόεδρος: Κυριακόπουλος Αντώνης

Αντιπρόεδροι: Α·: Ευσταθίου Βαyyέλης

ι·: Ταααόnουλος Γιώργος

Γραμματέας: Χριστόπουλος Παναγιώτης

Μέλη: Αργυράκης Δ.

Δρούτσας Π.

Λουρίδας Σ. Ταπcινός Ν.

Σuvταvτική επιτροπή Αθανασόπουλος Γεώργιος Ανδρουλακάκης Νίκος Αντωνόπουλος Νίκος Αργυράκης Δημήτριος Βακαλόπουλος Κώστας Βλάχου Αγγελική Γράψας Κων/νος Δρούτσας Παναγιώτης Ευσταθίου Βαγγέλης Ζαχαρόπουλος Κων/νος Ζώτος Βαγγέλης Καλίκας Σταμάτης Καρακατσάνης Βασίλης Καρκάνης Βασίλης Κατσούλης Γιώργος Κερασαρίδης Γιάννης Κηπουρός Χρήστος Κόντζιας Νίκος Κοτσιφάκης Γιώργος Κυριακόπουλος Αντώνης Κυριακόπουλος Θανάσης Κυβερνήτου Χρυστ .

Λαζαρίδης Χρήστος Λουρίδας Σωτήρης Μαλαφέκας Θανάσης Μεταξάς Νικόλαος Μυλωνάς Δημήτρης Μώκος Χρήστος Ρέγκλης Δημήτρης Σσ:tτη Εύα Σταθόπουλος Γεώργιος Σταϊκος Κώστας Στάϊκος Παναγιώτης Στρατής Γιάννης Ταπεινός Νικόλαος Τασσόπουλος Γιώργος Τριάντος Γεώργιος Τσαγκάρης Ανδρέας Τσικαλουδάκης Γιώργος Τσιούμας Θανάσης Τυρλής Ιωάννης; Χαραλαμποπούλου Λίνα Χαραλάμπους Θάνος Χριστόπουλος Παναγιώτης

•••••••••••••••••••••••••••••••••••••••••••••••••••••••••••••••••••••••••••••• • Τα διαφημιζόμενα βιβλία δε σημαίνει ότι προτείνονται από την Ε.Μ.Ε.

• Οι συνεργάτες, τα άρθρα, οι προτεινόμενες ασκήσεις, οι λύσεις ασκήσεων κτλ. πρέπει να στέλνονται έγκαιρα, στα γραφεία της Ε.Μ.Ε. με την ένδειξη "Για τον Ευκλείδη β"'. Τα χειρόγραφα δεν επιστρέφονται. τιμή Τεύχους ευρώ 3,50 Ετήσια σuνδQομή (10,00 + 4,00 Ταχυδρομικά = εUQώ 14,00) Ετήσια συνδρομή για Σχολεία εUQώ 10,00 Το αντίτιμο για τα τεύχη που παραγγέλνονται στέλνεται με απλή επιταγή σε διαταγή Ε.Μ.Ε. Ταχ. Γραφείο Αθήνα 54 Τ. Θ. 30044 ή πληρώνεται στα γραφεία της Ε.Μ.Ε.

Εκτύπωαη: /ΝτΕΡΓΙΡΕΣ Α.Ε. τηλ.: 210 8160330 Υneιίluνος τunoypaφeίou: 8. Σωτηριάδης

Page 4: Ευκλειδης Β 61

Γράμμα από το Δ.Σ. της Ε.Μ.Ε .. Αγαπητοί συνάδελφοι, Μια νέα σχολική χρονιά αρχίζει και το Δ.Σ. της Ελληνικής Μαθηματικής Εταιρείας εύχεται σε όλους

σας να είναι επιτυχημένη και δημιουργική . Με την έναρξη της χρονιάς αυτής θα ήθελε επίσης να εγκαινιάσει έναν νέο τρόπο επικοινωνίας και

ενημέρωσης μέσα από τις περιοδικές εκδόσεις της Εταιρείας. Θα θέλαμε, έτσι, να υπενθυμίσουμε ότι πέρα από τα περιοδικά ΕΥΚΛΕΙΔΗΣ Α' και ΕΥΚΛΕΙΔΗΣ

Β·. όπου δημοσιεύονται άρθρα, μέσα στα πλαίσια σύγχρονων παιδαγωγικών επιστημονικών και διδακτικών μεθόδων κατάλληλα για όλες τις ηλικίες των μαθητών του Γυμνασίου και του Λυκείου αντίστοιχα απαραίτητο βοήθημα στο μαθηματικό ερευνητή και στο μαθηματικό εκπαιδευτικό είναι οι εκδόσεις Ευκλείδης Γ και Μαθηματική Επιθεώρηση .

Ο Ευκλείδης Γ δημοσιεύει ερευνητικά αποτελέσματα του πεδίου της διδακτικής των Μαθηματικών και της Μαθηματικής Εκπαίδευσης, καθώς και άρθρα που συμβάλλουν στην ευρύτερη και βαθύτερη κατάρτιση όσων διδάσκουν μαθηι.;.ατικά

Το περιοδικό Μαθηματική Επιθεώρηση (Μ.Ε.) έχει στόχο να ενημερώνει τους συναδέλφους μαθηματικούς της χώρας μας σε θέματα Μαθηματικών, Ιστορίας και Φιλοσοφίας των Μαθηματικών,

Εφαρμογών των Μαθηματικών, Εκπαιδευτικής Έρευνας, Ειδικών θεμάτων, Παιδαγωγικών

θεμάτων κ.λπ.

Ο Μικρός Ευκλείδης, πρωτοποριακή έκδοση στο χώρο της Α/θμιας Εκπαίδευσης, μπαίνει στην σκέψη των μαθητών του Δημοτικού Σχολείου. Είναι ένα εργαλείο για την προσέγγιση των Μαθηματικών με νέες ιδέες και ευρηματικότητα ώστε να αγαπήσουν οι μικροί μαθητές τα Μαθηματικά.

Πρέπει ακόμη να σας ενημερώσουμε για την επανέκδοση του Αστρολάβου με θέματα πάνω στις Νέες Τεχνολογίες της Πληροφορίας και Επικοινωνίας.

Η Ελληνική Μαθηματική Εταιρεία ζητά τη συμμετοχή σας στην έκδοση των περιοδικών αυτών αλλά και τη συνδρομή σας ώστε να γίνουν γνωστά και απαραίτητα σε μαθητές και εκπαιδευτικούς.

Ένα ακόμη σημαντικό θέμα είναι η προσπάθεια Ύια την τροποποίηση του Καταστατικού Όπως γνωρίζετε στη συνεδρίαση του Δ.Σ. της Ε.Μ.Ε. της 1 ης Φεβρουαρίου 2006 και σύμφωνα με την ημερησία διάταξη, συζητήθηκε η ομόφωνη πρόταση που κατέθεσε η 8μελής Επιτροπή που είχε οριστεί με ομόφωνη απόφαση της Γενικής Συνέλευσης της 27ης Φεβρουαρίου 2005 .

Το Δ.Σ. αποφάσισε ομόφωνα να αποδεχθεί την πρόταση, όπως αυτή έλαβε τελική μορφή μετά από νομοτεχνική επεξεργασία και την έθεσε υπόψη Καταστατικής Γενικής Συνέλευσης του 2006.

Την πρόταση σε πλήρη μορφή μπορείτε να τη βρείτε στην ιστοσελίδα της Ε.Μ.Ε. www.hms.gr, ή να την προμηθευτείτε σε έντυπη μορφή από τα γραφεία της εταιρείας.

Το πρωτοποριακό εκλογικό σύστημα που προτείνεται είναι μια σύζευξη του ενιαίου ψηφοδελτίου και της απλής αναλογικής όπως προκύπτει από την πρόταση της επιτροπής . Επειδή στην Καταστατική Συνέλευσης που πραγματοποιήθηκε το Φεβρουάριο του 2006 δεν υπήρξε απαρτία το Δ.Σ. της Ε.Μ.Ε. πιστεύει ότι πρέπει να γίνει μια νέα προσπάθεια για να πραγματοποιηθεί νέα Καταστατική Συνέλευση

στην αρχή του 2007, θεωρώντας δεδομένη την βοήθεια σας για την ενεργοποίηση όλων των μελών, προκειμένου να υπάρξει ένα θετικό αποτέλεσμα

Τέλος, θα θέλαμε να σας υπενθυμίσουμε την κορυφαία ετήσια εκδήλωση στο χώρο της Μαθηματικής Παιδειας, το 23° Πανελλήνιο Συνέδριο Μαθηματικής Παιδείας με θέμα

((Τα μαθηματικά ως πολιτισμός στο σύγχρονο κόσμο, προl:κτάσεις στην ιφιτιι.:ιί σιt·έψη, στην

επιχειρηματολογία και στην αισθητικιίη που θα γίνει στις 24, 25 και 26 Νοεμβρίου στην Πάτρα. Ο Πρόεδρος

Καθηγητής Θεόδωρος Εξαρχάκος

ΕΥΚΛΕΙΔΗΣ Β' τ. Ι /2

Ο Γ ενικός Γραμματέας Ιωάννης Τυρλής

Page 5: Ευκλειδης Β 61

Αθανάσιος Φωκάς Ένας σύγχρονος σκαπανέας της Επιστήμης

αντάξιος των μεγάλων διανοητών

του 19ου αιώνα.

Α. Φωκάς γεννήθηκε στην Κεφαλονιά το

1 952 και πήρε εκεί τη βασική εγκύκλια

μόρφωση μέχρι την προτελευταία τάξη του

Γυμνασίου, την εβδόμη (έτσι λέγανε τότε η

σημερινή Γ ' Λυκείου) την έβγαλε στην Αθήνα όπου

έκανε φροντιστήριο προκειμένου να δώσει

εισαγωγικές εξετάσεις για το Πολυτεχνείο. Δεν τα

κατάφερε !

.:::.επέρασε γρήγορα το σοκ της aπογοήτευσης

(ήταν άριστος μαθητής) κι έφυγε στο εξωτερικό, στην

Αγγλία, αν και αγνοούσε πλήρως την αγγλική

γλώσσα. Έπεσε με τα μούτρα στην εκμάθηση των

αγγλικών (έφτασε να μαθαίνει 300 λέξεις τη μέρα) και

χωρίς καμιά καθυστέρηση πήρε το πτυχίο

Αεροναυπηγικής απ ' το Ιμπιριαλ Κολλετζ του

Λονδίνου, πέρνωντας και βραβείο ως καλύτερος

φοιτητής.

Στη συνέχεια ξεκίνησε ένα διδακτορικό στην

του Γιάννη Δ. Στρατή

αεροναυπηγική, σύντομα ωστόσο διαπίστωσε ότι χρειαζόταν μαθηματικά υψηλού επιπέδου. Του

συνέστησαν το Ινστιτούτο Τεχνολογίας της Καλιφόρνια (ΚΑΛ ΤΕΚ) των Η.Π.Α. όπου είχε

ξεκινήσει ένα νέο τμήμα Εφαρμοσμένων Μαθηματικών με υψηλές προδιαγραφές. Εκεί

ερωτεύθηκε τα Μαθηματικά και αποφάσισε να τους αφοσιωθεί ολοκληρωτικά. Τελικά πήρε

διδακτορικό δίπλωμα στα Μαθηματικά με άριστες επιδόσεις.

Το 2000 η Μαθηματική Εταιρεία του Λονδίνου του απένειμε το βραβείο Νέιλορ για την

ανακάλυψη μιας τεχνικής επίλυσης διαφορικών εξισώσεων. Το βραβείο αυτό είναι απ ' τα πιο

σημαντικά για τη Μαθηματική Επιστήμη και την Μαθηματική Φυσική κι έχουν τιμηθεί με το

βραβείο αυτό εξέχουσες επιστημονικές προσωπικότητες, όπως ο Ρ. Πενρόουζ και ο Σ. Χώκινγκ.

Σήμερα είναι καθηγητής στην έδρα Μη Γραμμικών Επιστημών του Πανεπιστημίου

Κέιμπριτζ. Η έδρα αυτή ανήκει στο τμήμα Εφαρμοσμένων Μαθηματικών και Θεωρητικής

Φυσικής, όπου έχουν διδάξει μεγάλες φυσιογνωμίες της Επιστήμης όπως ο Ισαάκ Νιούτον και ο

Στίβεν Χώκινγκ.

Μία από τις ερευνητικές περιοχές του Α. Φωκά είναι η λύση μαθηματικών προβλημάτων

σχετικών με την μελέτη του ανθρώπινου εγκεφάλου καθώς βρίσκεται σε λειτουργία.

Παρατηρούμε ότι το 1986 πήρε δίπλωμα Ιατρικής απ ' το Πανεπιστήμιο του Μαϊάμι Η.Π.Α. . Σε

συνεργασία με τον Ίσραελ Γκέλφαντ (Ρώσος μαθηματικός και βιολόγος παγκόσμιας

ΕΥΚΛΕΙΔΗΣ Β' τ. Ι /3

Page 6: Ευκλειδης Β 61

Αθανάσιος Φωκάς

αναγνώρισης) με τη συνδρομή και άλλων επιστημόνων αναλόγου ειδικού βάρους, συνέβαλε στη

λύση του κύριου μαθηματικού προβλήματος στο οποίο στηρίζεται η λεγόμενη τεχνική SPECT η

οποία μας δίνει την δυνατότητα να παρατηρούμε, με όλο και μεγαλύτερη ακρίβεια τον

ανθρώπινο εγκέφαλο καθώς λειτουργεί. Η επίλυση του προβλήματος αυτού, που

πραγματοποιήθηκε πολύ πρόσφατα, ήταν κατά κοινή παραδοχή , εξαιρετικά δυσκολότερη από

την επίλυση του μαθηματικού προβλήματος, στο οποίο στηρίχθηκε ο Α. Κόρμακ ( φυσικός και

μαθηματικός, που τιμήθηκε με το βραβείο Νόμπελ της Σουηδικής Ακαδημίας Επιστημών το

1979) για την κατασκευή του αξονικού τομογράφου.

Ο αξονικός τομογράφος και ο μαγνητικός τομογράφος (για την ανακάλυψη του οποίου

τιμήθηκε με το βραβείο Νόμπελ το 2003 ο Πήτερ Μάνσφηλντ) έφεραν πραγματική επανάσταση

στη Νευρολογία. Και όχι μόνο, ολόκληρος ο κλάδος της Ιατρικής Επιστήμης επηρεάστηκε

θετικά. Παρ' όλα αυτά οι σημαντικότατες αυτές τεχνικές δίνουν στατικές εικόνες (στιγμιότυπα) .

Αν και είναι λεπτομερέστατες δεν μας επιτρέπουν να παρακολουθούμε , καθώς δουλεύει, ένα

όργανο ζώντος οργανισμού, πολύ περισσότερο τον ανθρώπινο εγκέφαλο.

Το Γενάρη του 2005 ο Πρόεδρος της Δημοκρατίας Κωνσταντίνος Στεφανόπουλος απένειμε

στον Α. Φωκά το παράσημο του Τάγματος του Φοίνικα. Το παράσημο αυτό απονέμεται σε

ανθρώπους που διακρίθηκαν στη Δημόσια Διοίκηση, τις Επιστήμες, τις Τέχνες και τα Γράμματα,

στο Εμπόριο, τη Βιομηχανία και τη Ναυτιλία.

Την Άνοιξη του 2004 τιμήθηκε με το Αριστείο της Ακαδημίας Αθηνών και το Δεκέμβρη

του ίδιου χρόνου εκλέχτηκε μέλος της. Είναι το νεότερο μέλος της Ακαδημίας Αθηνών ( από

αρκετούς επιστήμονες και διανοούμενους στο εξωτερικό η σύγχρονη Ακαδημία μας θεωρείται

συνέχεια της Ακαδημίας του Πλάτωνα). Επίσης έχει ανακηρυχθεί επίτιμος διδάκτορας του

Εθνικού Μετσοβείου Πολυτεχνείου και του Πολυτεχνείου Κρήτης, καθώς επίσης και των

Πανεπιστημίων Πάτρας και Αθήνας.

Το Μάρτη του 2005 , ανταποκρινόμενος στις παραινέσεις του δάσκαλου I. Γκέλφαντ,

ανέλαβε πρόεδρος του Εφορευτικού

Συμβουλίου της Εθνικής (Βαλλιάνειου)

Βιβλιοθήκης με μόνο κίνητρο όπως

δηλώνει «να συμβάλλει στο να κατακτήσει

η Βιβλιοθήκη τη θέση που της αρμόζει,

στον Ευρωπαϊκό χώρο».

Τώρα ζει στο Cambridge με τη σύζυγό

του Ρεγγίνα (γλωσσολόγος) και τα τρία

παιδιά τους Αλέξανδρο, Αναστασία,

Ιωάννα.

Όπως λέει ο Α. Φωκάς «όλες αυτές οι

εξαιρετικές τιμές με γεμίζουν βέβαια

συγκίνηση, αλλά κυρίως μεγαλώνουν

ακόμα περισσότερο το βαθύτατο αίσθημα

ευθύνης που νιώθω. Γιατί όλες αυτές τις

μεγάλες τιμές δεν τις είδα ποτέ σαν

αναγνώριση του έργου μου, αλλά σαν

κατάθεση προσδοκιών για συνέχιση της

πορείας μου».

ΕΥΚΛΕΙΔΗΣ Β ' τ.l/4

Page 7: Ευκλειδης Β 61

Επιμέλεια: Χρήστος Κηπουρός

Τα Προ- Ευκλείδεια Μαθηματικά

Γ' ια τα αρχαία ελληνικά μαθηματικά, ο Ευ­κλείδης κατέχει ένα ρόλο παρόμοιο με αυ­

. . . . .. τόν του Σωκράτη στην ελληνική φtλοσοφία. Είναι δηλαδή ο επιστήμονας που με το έρyο του χω­ρίζει την ιστορία των αρχαίων ελληνικών μαθηματι­κών σε δύο διακριτές ιστορικές περιόδους: την προ­Ευκλείδεια και την μετα-Ευκλείδεια. Η πρώτη από τις δύο αυτές περιόδους, η προ-Ευκλείδεια, παρου­σιάζει ιδιαίτερο ιστορικό ενδιαφέρον για δύο κυρίως λόγους: ο ένας είναι ότι ιστορικά έχει μελετηθεί πολύ λιγότερο από την μετα-Ευκλείδεια, και ο δεύτερος είναι ότι αρκετά από τα παγιωμένα συμπεράσματα της μαθηματικής ιστοριογραφίας ανατρέπονται από τα στοιχεία που φέρνει στο φως η σύγχρονη ιστορική έρευνα.

Για την ιστορική εξέταση της προ-Ευκλείδειας περιόδου υπάρχει μια πολύ μεγάλη αντικειμενική δυσκολία: η παντελής έλλειψη πρωτευουσών ιστορι­κών πηγών, δηλαδή μαρτυριών από πρώτο χέρι. Ως πρωτεύουσες ιστορικές πηγές θεωρούνται τα πραγ­ματικά αποσπάσματα του έρyου ενός συγγραφέα, ή έστω η κατά λέξη αξιόmστη μεταφορά τους από κά­ποιον άλλο.

Σπανίζουν όμως και οι δευτερεύουσες ιστορικές πηγές, δηλαδή οι μαρτυρίες πρώιμων αυθεντιών, ό­πως είναι ο Πλάτων, ο Αριστοτέλης και οι άμεσοι μαθητές τους.

Η παλαιότερη δευτερεύουσα ιστορική πηγή που έχουμε για την προ-Ευκλείδεια περίοδο είναι δύο αποσπάσματα από το μη διασωθέν έρyο "Γεωμετρι­

καί Ιστορία" του μαθητή του Αριστοτέλη Εύδημου

του Ρόδιου (β ' μισό 4ου αι. π.Χ.). Το πρώτο από αυ-

Στέλιος Λαμνής

τά αναφέρεται στον τετραγωνισμό των μηνίσκων από τον Ιπποκράτη τον Χίο και το δεύτερο στον τε­τραγωνισμό του κύβου από τον Αρχύτα τον Ταρα­ντίνο. Εννοείται πως οι δευτερεύουσες πηγές ενέχουν ήδη ένα στοιχείο αβεβαιότητας, εφόσον οι πρωτότυ­πες έννοιες μπορεί να έχουν διαστρεβλωθεί, παρου­σιαζόμενες έτσι ώστε να συμμορφώνονται προς έναν πλατωνικό ή aριστοτελικό τρόπο σκέψης.

Με την ευκαιρία να πούμε ότι ιστορίες των ελ­ληνικών μαθηματικών στην προ-Ευκλείδεια περίοδο, εκτός από τον Εύδημο, έγραψαν και οι Ξενοκράτης

ο Χαλκηδόνιος (4ος αι. π.Χ.) και Θεόφραστος ο Λέ­σβιος (372-287 π.Χ.).

Το δυστύχημα είναι ότι κανένα από τα δύο αυτά έρyα δεν διασώθηκε. Ιστορία των μαθηματικών στην μετα-Ευκλείδεια περίοδο έγραψε και ο Γεμίνος ο

Ρόδιος (περί το 70 π.Χ.), η οποία λέγεται ότι στηρι­ζόταν στην ιστορία του Εύδημου, αλλά δυστυχώς κι αυτή δεν διασώθηκε.

ΕΥΚΛΕΙΔΗΣ Β' τ. l/5

Page 8: Ευκλειδης Β 61

------------- Ιστορικές Μαθηματικές Αναφορέ

Ευκλείδης Ό,τι γνωρίζουμε για τα προ-Ευκλείδεια μαθημα­

τικά ουσιαστικά προέρχεται από μεταγενέστερους συγγραφείς, οι περισσότεροι από τους οποίους έζη­σαν μέχρι και Ι 000 χρόνια αργότερα από την περίο­δο που εξετάζουμε. Είναι άγνωστο αν αυτοί οι συγ­γραφείς (δοξογράφοι, υπομνηματιστές, σχολιογρά­φοι, χρονικογράφοι κ.λ.π.) είχαν πρόσβαση σε πηγές ή σε πρωτογενές υλικό. Το πιθανότερο είναι ότι βα­σίζονταν σε επιτομές έργων που έγιναν από την aρι­στοτελική σχολή (π.χ. η ιστορία του Εύδημου), οπότε είναι πιθανό οι μαρτυρίες τους να υπέστησαν δια­στρεβλώσεις κατά τη διαδικασία της συνεχούς μετα-βίβασης. Αυτές θα μπορούσαν να ονομαστούν τρι­τεύουσες πηγές, αν και το μεγαλύτερο μέρος τους δεν είναι καν από τρίτο χέρι.

Στις τριτεύουσες πηγές ανήκει ο Πρόκλος ο Λύ­

κειος ή Βυζάντιος ( 410-485 μ.Χ), που θεωρείται σήμερα η πιο αξιόλογη πηγή πληροφοριών για την ιστορία της προ-Ευκλείδειας Γεωμετρίας. Ο Πρό­κλος δεν ήταν ιστορικός ούτε έγραψε ποτέ ιστορία των μαθηματικών. Έγραψε σχόλια στο πρώτο Βιβλίο των "Στοιχείων" του Ευκλείδη, το "Υπόμνημα εις το

πρώτον των Ευκλείδου Στοιχείων", όπου περιέχονται βιογραφικά και βιβλιογραφικά στοιχεία για αρκετούς αρχαίους Έλληνες γεωμέτρες και τα έργα τους. Στο έργο του Πρόκλου υπάρχει μια ολιγοσέλιδη (σελίδες 65-70 του βιβλίου) παρουσίαση της εξέλιξης της ελ­ληνικής Γεωμετρίας από τους αρχαιοτάτους χρόνους, η οποία ονομάζεται σήμερα "Σύνοψις ".

Στις τριτεύουσες πηγές ανήκουν και οι σχολια­στές των έργων άλλων αρχαίων μαθηματικών. Τέ­τοιοι σχολιαστές ήταν ο Θέων ο Σμυρναίος (2°ς αι. μ.Χ.), ο Πάππος ο Αλεξανδρινός (3ος αι. μ.Χ.), ο Θέων ο Αλεξανδρινός (320-395 μ.Χ.) και η κόρη

του Υπατία (375-415 μ.Χ.), ο Ισίδωρος ο Μιλήσιος και ο μαθητής του Ευτόκιος (6ος αι. μ.Χ.), ο Σιμπλί­

κιος ( 6ος αι. μ. Χ.) και άλλοι. Με βάση τα παραπάνω στοιχεία είναι φανερό ότι

οι γνώσεις μας για την προ-Ευκλείδεια περίοδο αντι­κειμενικά είναι ελάχιστες και αποσπασματικές. Προ­κύπτουν έτσι μερικά εύλογα ερωτήματα σχετικά με την φερεγγυότητα των πληροφοριών που έχουμε σήμερα για την ιστορία των προ-Ευκλείδειων μαθη­ματικών.

Το πρώτο ερώτημα αφορά στον τρόπο με τον οποίο παρουσιάστηκαν οι μαθηματικοί της προ­Ευκλείδειας περιόδου από τους αρχαίους ιστορικούς. Θα διαmστώσουμε ότι υπάρχουν περιπτώσεις μαθη­ματικών της προ-Ευκλείδειας περιόδου που αντιμε­τωπίστηκαν με φανερή ιδεολογική προκατάληψη. Η μομφή αφορά κυρίως τον Πρόκλο, που όπως είπαμε θεωρείται η mo αξιόπιστη που διαθέτουμε για τα προ-Ευκλείδεια μαθηματικά. Ο Πρόκλος στη Σύνο­

ψή του (σελ. 65.21-66.18), παραθέτει ένα κατάλογο των Ελλήνων γεωμετρών που άκμασαν στην περίοδο μεταξύ Πυθαγόρα και Πλάτωνα. Σ' αυτόν τον κατά­λογο περιλαμβάνονται κατά χρονολογική σειρά εμ­φάνισης τα ακόλουθα ονόματα: Ι ) Αναξαγόρας ο Κλαζομένιος (500-428 π.Χ.), 2)

Οινοπίδης ο Χίος (περί το 440 π.Χ.), 3) Ιπποκράτης

ο Χίος (470-400 π.Χ.), 4) Θεόδωρος ο Κυρηναίος

(περί το 390 π.Χ.), 5) Λεωδάμας ο Θάσιος (περί το 400 π.Χ.), 6) Αρχύτας ο Ταραντίνος (428-365 π.Χ.) και 7) Θεαίτητος ο Αθηναίος (417-369 π.Χ.).

Η πρώτη παρατήρησή μας είναι ότι από τα επτά ονόματα που αναφέρονται, τα τέσσερα τελευταία σχετίζονται άμεσα με τον Πλάτωνα και τη σχολή του. Ο Θεόδωρος ο Κυρηναίος αναφέρεται από τον Ιάμβλιχο ως Πυθαγόρειος, ενώ σύμφωνα με μια άλ­λη πληροφορία υπήρξε δάσκαλος του Πλάτωνα στα μαθηματικά. Ο Λεωδάμας ο Θάσιος υπήρξε μαθητής του Πλάτωνα και σύμφωνα με τον Πρόκλο διδάχτη­κε από τον Πλάτωνα την αναλυτική μέθοδο, με την οποία ο ίδιος ανακάλυψε τις γεωμετρικές προτάσεις που του αποδίδονται. Ο Αρχύτας ο Ταραντίνος ήταν γνωστός Πυθαγόρειος και επιστήθιος φίλος του Πλά­τωνα, τον οποίο διέσωσε από βέβαιο θάνατο όταν ο

ΕΥΚΛΕΙΔΗΣ Β ' τ. t /6

Page 9: Ευκλειδης Β 61

Ιστορικές Μαθηματικές Αναφορές-------------

τελευταίος βρέθηκε στην Κάτω Ιταλία. Τέλος, ο Θεαίτητος ήταν μαθητής και φίλος του Πλάτωνα, αλλά αναφέρεται επίσης και ως μαθητής του Θεόδω­ρου του

Ιπποκράτης

Η δεύτερη παρατήρησή μας είναι ότι από τον κατάλογο του Πρόκλου λείπουν τα ονόματα ορισμέ­νων πραγματικά αξιόλογων μαθηματικών αυτής της περιόδου, όπως του Δημόκριτου του Αβδηρίτη (460-370 π.Χ.) και επίσης και των Σοφιστών Ιππία

του Ηλείου (β ' μισό του 5ου αι. π.Χ), Αντιφώντα

του Αθηναίου (περί 430 π.Χ.) και Βρύσωνα του

Ηρακλείωτη (περί 430 π.Χ.). Οι τέσσερις αυτοί δια­νοητές, παρά την εmκρατούσα αντίληψη, ασχολήθη­καν με τα μαθηματικά και μάλιστα είχαν διατυπώσει πρωτοποριακές μαθηματικές θεωρίες που προηγήθη­καν κατά πολύ της εποχής τους. Επειδή είναι αδύνα­το η μαθηματική συνεισφορά του Δημόκριτου και των Σοφιστών να μην ήταν γνωστή στην αρχαιότητα, το πιθανότερο είναι ότι δεν έχουμε να κάνουμε με μια αθώα ή τυχαία "αβλεψία" του Πρόκλου, αλλά με μια συνειδητή επιλογή του. Γιατί όμως ο Πρόκλος παρέλειψε αυτά τα ονόματα;

Μια πρώτη εξήγηση είναι ότι η "αβλεψία" του Πρόκλου οφείλεται στη γνωστή σε όλους ιδεολογική αντιπαλότητα του Πλάτωνα προς τον Δημόκριτο και τους Σοφιστές. Ο Πλάτων δεν είχε μαζί τους απλώς επιστημονικές ή φιλοσοφικές διαφορές, είχε πλήρη ιδεολογική και πολιτική διάσταση.

Ενδεικτική της αντιπάθειας· του Πλάτωνα προς τον Δημόκριτο είναι η πλήρης απάλειψη του ονόμα­τος του Δημόκριτου από το σύνολο των πλατωνικών διαλόγων. Σε όλους τους πλατωνικούς διαλόγους το

όνομα του Αβδηρίτη φιλοσόφου δεν αναφέρεται ού­τε μία φορά. Υπάρχει μάλιστα η μαρτυρία του Αρι­στόξενου, ενός μαθητή του Αριστοτέλη, ότι ο Πλά­των απεχθανόταν τόσο πολύ τον Δημόκριτο, ώστε σχεδίαζε να κάψει όσα συγγράμματά του είχε κατορ­θώσει να συγκεντρώσει. Τον συγκράτησαν όμως οι Πυθαγόρειοι Αμύκλας και Κλεινίας, οι οποίοι του εξήγησαν ότι κάτι τέτοιο ήταν άσκοπο γιατί τα βι­βλία του Δημόκριτου ήταν πολύ διαδεδομένα και τα είχαν πλέον στην κατοχή τους και πολλοί άλλοι (Δι­ογένης Λαέρτιος, ΙΧ. 40). Στο σημείο αυτό να λά­βουμε υπόψιν μας ότι από τα πολυάριθμα συγγράμ­ματα του Δημόκριτου δεν διασώθηκε ούτε ένα, σε αντίθεση με τα έργα του Πλάτωνα που σώθηκαν όλα.

Πασίγνωστη επίσης είναι η απέχθεια του Πλάτω­να και προς τους Σοφιστές. Στη διαμάχη του όμως προς τους Σοφιστές, ο Πλάτων επέλεξε την τακτική της ευθείας αντιπαράθεσης, γι' αυτό και οι απόψεις των Σοφιστών αποτέλεσαν το θέμα αρκετών πλατωνι­κών διαλόγων. Ποια ήταν όμως η βαθύτερη αιτία αυ­τής της διαμάχης; Την απάντηση μας δίνει ο καθηγη­τής Χαράλαμπος Θεοδωρίδης, ο οποίος στο βιβλίο του ''Επίκουρος. Η αληθινή όψη του αρχαίου κόσμου"

(σελ. 63), εmσημαίνει: «Δε χρειάζεται μεγάλος κόπος

γι(Χ να εννοήσει κανείς τη λύσσα της αριστοκρατίας για

τους "νεωτεροποιούς", που έβαζαν άλλη μια φορά σε

κίνδυνο τα ιερά και τα όσια. Πολέμησαν με πείσμα τη

σοφιστική. Αντίθετα στην καινούργια αγωγή σκόρπισαν

το σύνθημα να ξαναγυρίσουν στην "πάτριον αγωγήν",

όπως δημοκοπούσαν οι υπερπατριώτες με το πίσω στην

"πάτριον πολιτείαν ", εννοώντας μ' αυτό την πολιτεία

που οι ίδιοι ποθούσαν. Ο Πλάτων μια ζωή πολέμησε

τους σοφιστές μ' επιχειρήματα λίγο πειστικά, που άμα

τα εξετάσεις προσεχτικά περισσεύει η εκτίμησή σου στο

πρόσωπό τους. Στην προσπάθειά του είχε πολυάριθ­

μους βοηθούς, που δυνατότερός τους στάθηκε η νοητι­

κή και συναισθηματική κατάσταση των χρόνων ύστερα

από τον [ενν. Πελοποννησιακό] πόλεμο. Ώσπου κατά­

φεραν να παραμορφώσουν και να κάνουν αγνώριστες

τις μεγάλες εκείνες φυσιογνωμίες».

Ο Πρόκλος τώρα ήταν γνωστός νέο-πλατωνικός φιλόσοφος και ένας από τους τελευταίους διευθυντές της Ακαδημίας του Πλάτωνα. Ήταν ο άνθρωπος που

• ΕΥΚΛΕΙΔΗΣ Β' τ. Ι /7

Page 10: Ευκλειδης Β 61

------------- Ιστορικές Μαθηματικές Αναφορέ

συστηματοποίησε τη νεοπλατωνική φιλοσοφία και της έδωσε την τελειωτική της μορφή, αυτήν με την οποία έγινε γνωστή στους μεταγενέστερους χρόνους. Αξίζει να εmσημάνουμε ότι όλα τα σωζόμενα φιλο­σοφικά συγγράμματα του Πρόκλου σχετίζονται με τον πλατωνισμό (υπομνήματα στα πλατωνικά έργα τίμαιος, Πολιτεία, Παρμενίδης, Αλκιβιάδης και Κρα­

τύλος). Με βάση τα παραπάνω στοιχεία, η "αβλεψί­

α" του Πρόκλου φαίνεται ότι ήταν εσκεμμένη και οφείλετο στην φιλοσοφική τοποθέτησή του υπέρ του Πλατωνισμού.

Είναι ικανοποιητική όμως η εξήγηση ότι ο Πρό­κλος παρέλειψε τα ονόματα του Δημόκριτου και των Σοφιστών απλώς και μόνο επειδή θέλησε να συμ­μορφωθεί με τις επιταγές του Πλάτωνα; Νομίζω πως όχι. Κάτι βαθύτερο πρέπει να κρύβεται πίσω από αυτήν την παράλειψη.

Έχω την εντύπωση ότι ο Πρόκλος, όπως άλλω­στε όλοι οι Πλατωνικοί, διαφωνούσε πρωτίστως με το είδος των μαθηματικών ενασχολήσεων των ιδεο­λογικών αντιπάλων του. Ο Δημόκριτος και οι Σοφι­στές ήταν συνεχιστές της μαθηματικής παράδοσης που είχαν εγκαινιάσει ο Θαλής και οι άλλοι Ίωνες φιλόσοφοι, δηλαδή της παράδοσης που έδινε προτε­ραιότητα στα μαθηματικά των μετρήσεων και των υπολογισμών. Είναι όμως γνωστό πως αυτού του είδους τα μαθηματικά ήταν αδιανόητα για τους Πλα­τωνικούς. Γι' αυτούς, τα μαθηματικά και ειδικά η γεωμετρία έπρεπε να αναφέρεται αποκλειστικά και μόνο στον κόσμο των άϋλων όντων, δηλαδή στον ιδεατό πλατωνικό κόσμο των Ιδεών.

Η σύγκρουση Δημοκρίτου και Πλάτωνα λοιπόν δεν ήταν τίποτα άλλο παρά η διαμάχη υλισμού και ιδεαλισμού στον τομέα των μαθηματικών.

Με τι είδους μαθηματικά όμως ασχολήθηκε ο Δημόκριτος; Από τους τίτλους των μαθηματικών συγγραμμάτων του [(1) "Περί διαφορής γνώμης ή

περί ψαύσιος κύκλου και σφαίρας", (2) "Περί Γεωμε­

τρίης", (3) "Γεωμετρικών", (4) "Αριθμοί ", (5) "Πε­

ρί αλόγων γραμμών και ναστών", (6) 'Έκπετάσμα­

τα "] δεν αναμένουμε καμιά βοήθεια, απλούστατα γιατί δεν διασώθηκε καμιά ασφαλής πληροφορία σχετικά με το περιεχόμενό τους. Έτσι λοιπόν ό,τι

γνωρίζουμε για τα μαθηματικά του Δημόκριτου προέρχεται από δύο σύντομες αναφορές άλλων συγ­γραφέων, μια του Αρχιμήδη και μια του Πλούταρ­χου.

Θαλής ο Μιλήσιος

Ο Αρχιμήδης, στην εισαγωγή της περίφημης "Μεθόδου" του, που ως γνωστόν ανακαλύφθηκε τυχαία το 1906, αναφέρει ότι ο Δημόκριτος ήταν ο πρώτος που διατύπωσε προτάσεις σχετικά με τη σχέ­ση των όγκων του κώνου και του κυλίνδρου. Ο Δη­μόκριτος βρήκε ότι ο όγκος ενός κώνου ισούται με το ένα τρίτο του όγκου ενός κυλίνδρου, που έχει την ίδια βάση και ίσο ύψος. Επίσης βρήκε ότι ο όγκος μιας πυραμίδας ισούται με το ένα τρίτο του όγκου ενός πρίσματος, που έχει την ίδια βάση και ίσο ύψος (Heath, σ. 223). Είναι άγνωστο αν ο Δημόκριτος είχε καταφέρει να αποδείξει τις παραπάνω προτάσεις, τις οποίες πάντως απέδειξε περίπου πενήντα χρόνια αρ­γότερα ο Εύδοξος ο Κνίδιος.

Αρχιμήδης ο Συρακούσιος

Ο Πλούταρχος στο βιβλίο του "Περί των κοινών

εννοιών προς τους Στωικούς", περιλαμβάνει μια δή­λωση του Δημόκριτου σχετικά με τον όγκο του κώ­νου. Σ' αυτήν ο Δημόκριτος φέρεται να δηλώνει ότι

ΕΥΚΛΕΙΔΗΣ Β ' τ. Ι /8

Page 11: Ευκλειδης Β 61

------------- Ιστορικές Μαθηματικές Αναφορέr--------------

αν ένας κώνος τμηθεί από ένα επίπεδο παράλληλο προς τη βάση του, τότε υπάρχει ένα ερώτημα σχετικά με το είδος της τομής. Είναι αυτές ίσες ή άνισες; Αν οι τομές είναι άνισες, τότε θα έχουμε ως αποτέλεσμα έναν ακανόνιστο κώνο, ο οποίος θα εμφανίζει πολλές εγκοπές, θα έχει δηλαδή κλιμακωτή δομή και ανω­μαλίες. Αν οι τομές είναι ίσες, τότε ο κώνος θα απο­τελείται από ίσους κύκλους, δηλαδή ουσιαστικά θα είναι κύλινδρος.

Όπως παρατηρεί ο Heath, η φράση «ο κώνος θα

αποτελείται από ίσους κύκλους» μας επιτρέπει να υ­ποθέσουμε ότι ο Δημόκριτος είχε συλλάβει ήδη την ιδέα ότι κάθε στερεό σχήμα είναι το άθροισμα ενός απείρου πλήθους απείρως λεπτών παράλληλων εm­πέδων, τα οποία βρίσκονται μεταξύ τους σε απεφο­στή απόσταση(Ηeath, σ. 227). Είναι αδύνατο να μη συσχετίσει κανείς αυτή την ιδέα με την ατομική θε­ωρία του Δημόκριτου.

Συνεπώς, με τον Δημόκριτο έχουμε για πρώτη φορά στην ιστορία των μαθηματικών τη διατύπωση των βασικών αρχών της γεωμετρίας των απειροστών. Ο Δημόκριτος, δύο αιώνες δηλαδή πριν από τον Αρ­χιμήδη, διατύπωσε τον πυρήνα μιας ριζοσπαστικής μαθηματικής ιδέας, την οποία όπως γνωρίζουμε υιο­θέτησε και τελειοποίησε αργότερα ο μεγάλος Συρα­κούσιος σοφός.

Ας εξετάσουμε όμως τώρα και τα μαθηματικά των Σοφιστών. Από τους τρεις προαναφερθέντες Σοφιστές, πιο γνωστός για τη μαθηματική του συνει­σφορά είναι ο Ιππίας ο Ηλείος. Αυτός επινόησε μια ειδική καμπύλη, γνωστή ως τετραγωνίζουσα του Ιπ­

πία, για την επίλυση του προβλήματος της τριχοτό­μησης μιας γωνίας.

Πολύ αξιόλογη όμως ήταν η μαθηματική συνει­σφορά του Α ντιφώντα και του Βρύσωνα. Ο Α ντιφών ήταν ο πρώτος Έλληνας που είχε την ιδέα να υπολο­γίσει θεωρητικά το εμβαδόν του κύκλου. Η ιδέα του, όπως διασώθηκε από τους Θεμίστιο και Σιμπλίκιο (H.Diels-W.Κranz, "Dίe Fragmente der Vorsok­ratίker", σ. 340-34 1 ), ήταν να εγγράψει στον κύκλο ένα κανονικό πολύγωνο (ισόπλευρο τρίγωνο ή τε­τράγωνο) και μετά να διχοτομεί συνεχώς τα σχηματι­ζόμενα (ίσα) τόξα έτσι, ώστε να προκύπτουν συνε-

χώς κανονικά πολύγωνα που να καλύπτουν ολοένα και περισσότερο τον κυκλικό δίσκο. Ο Α ντιφών πί­στευε ότι έτσι θα κατέληγε σε ένα κανονικό πολύγω­νο, του οποίου οι πλευρές, λόγω του μικρού μεγέ­θους τους, θα συνέπιπταν με την περιφέρεια του κύ­κλου.

Περιττό βέβαια να πούμε ότι η ριζοσπαστική αυ­τή ιδέα του Α ντιφώντα προκάλεσε την οργή του πλα­τωνικού μαθηματικού κατεστημένου. Η ιδέα του Α­ντιφώντα θα παρέμενε για πάντα άγνωστη αν δεν είχε ερεθίσει τα συντηρητικά ανακλαστικά του Αριστοτέ­λη, ο οποίος άθελά του την διέσωσε όταν θέλησε να της ασκήσει κριτική ("Φυσικά", 1 85a). Η γνώμη του Αριστοτέλη ήταν ότι δεν υπήρχε ανάγκη να αντικρού­σει κανείς την ιδέα του Αντιφώντα, μιας και δεν βασί­ζεται σε αναγνωρισμένες αρχές της Γεωμετρίας.

Ο Βρύσων όχι μόνο αποδέχτηκε τη μέθοδο του Αντιφώντα αλλά την βελτίωσε ακόμη περισσότερο. Σύμφωνα με τον σχολιαστή του Αριστοτέλη Αλέ­ξανδρο τον Αφροδισιέα, ο Βρύσων θεώρησε αρχικά τρία τετράγωνα· το ένα εγγεγραμμένο στον κύκλο, το δεύτερο περιγεγραμμένο στον ίδιο κύκλο και το τρίτο ανάμεσά τους. Η ιδέα του ήταν να διπλασιάζει συνεχώς τις πλευρές τόσο των εγγεγραμμένων όσο και των περιγεγραμμένων τετραγώνων, όπως ακρι­βώς έκανε και ο Α ντιφών με τα εγγεγραμμένα κανο­νικά πολύγωνα.

Συνάδenqιοι

ΕΥΚΛΕΙΔΗΣ Β ' τ. Ι /9

Page 12: Ευκλειδης Β 61

------------- Ιστορικές Μαθηματικές Αναφορέ·ι---------------

Πίστευε ότι, αν αυτή η διαδικασία συνεχιστεί, τότε θα έχουμε ένα εγγεγραμμένο και ένα περιγε­γραμμένο πολύγωνο που θα διαφέρουν τόσο λίγο ως προς το εμβαδόν τους, ώστε αν μπορεί να οριστεί ένα πολύγωνο με εμβαδόν ίσο με το ημιάθροισμα των δύο ω.λων, τότε ο κύκλος, που έχει κι αυτός εμβαδόν ανάμεσα στο εγγεγραμμένο και στο περιγεγραμμένο πολύγωνο, θα είναι ισοδύναμος μ' αυτό το ενδιάμεσο πολύγωνο.

Η ιδέα του Βρύσωνα βεβαίως είναι λανθασμένη, ωστόσο, αποτελεί μια σαφή βελτίωση της μεθόδου του Αντιφώντα. Το σημαντικότερο όμως είναι ότι αυτή η βελτιωμένη μέθοδος των Α ντιφώντα - Βρύ­σωνα αποτελεί την πρώτη, την αρχική μορφή της "μεθόδου της εξάντλησης", η πατρότητα της οποίας ως γνωστόν αποδίδεται στον Εύδοξο τον Κνίδιο και στον Αρχιμήδη.

Η τρίτη παρατήρησή μας είναι ότι περιέργως ελά­χιστες είναι οι γνώσεις μας ακόμη και για τους γεωμέ­τρες που περιλαμβάνει ο Πρόκλος στη Σύνοψή του.

Για τον Αναξαγόρα τον Κλαζομένιο γνωρίζουμε απλώς ότι ασχολήθηκε με το πρόβλημα του τετρα­γωνισμού του κύκλου, όταν βρέθηκε κάποτε στη φυλακή στην Αθήνα γιατί δίδασκε ότι ο ήλιος και οι aστέρες ήταν νεκροί πύρινοι λίθοι.

Στον Οινοπίδη το Χίο, ο Πρόκλος αποδίδει δύο γεωμετρικές προτάσεις, οι οποίες περιέχονται στα "Στοιχεία" του Ευκλείδη. Πρόκειται για την πρόταση 1. 1 2, η οποία αναφέρεται στη χάραξη μιας ευθείας κάθετης προς δοσμένη ευθεία από σημείο εκτός αυ­τής, και στην πρόταση 1.23 , η οποία αναφέρεται στην κατασκευή γωνίας ίσης με δοσμένη γωνία.

Για τον Ιπποκράτη το Χίο, τον σημαντικότερο από τους επτά γεωμέτρες της Σύνοψης, ο Πρόκλος αναφέρει ότι ήταν ο πρώτος που συνέγραψε βιβλίο Γεωμετρίας, το οποίο όμως δεν σώθηκε. Ο Ιπποκρά­της όμως είναι περισσότερο γνωστός για δύο πάρα πολύ σημαντικά μαθηματικά επιτεύγματά του. Το πρώτο είναι ο τετραγωνισμός ορισμένων μηνίσκων που σήμερα φέρουν το όνομά του, και το άλλο είναι η αναγωγή του προβλήματος του διπλασιασμού του κύβου στην εύρεση δύο μέσων αναλόγων.

Οι λιγοστές πληροφορίες μας για τον Θεόδωρο τον Κυρηναίο προέρχονται κυρίως από τον πλατωνικό

διάλαyο Θεαίτητος. Εκεί αναφέρεται ότι ο Θεόδωρος απέδειξε πρώτος την ασυμμετρία των J3 , ..J5, . . . .Jl7 . Ακόμη πιο λιγοστές είναι οι πληροφορίες μας για τον Λεδάμαντα τον Θάσιο. Γνωρίζουμε μόνο ότι ήταν μαθητής του Πλάτωνα και ότι διδάχτηκε από τον δάσκαλό του την αναλυτική μέθοδο στη Γεωμετρία (Διογένη Λαέρτιος 111. 24). Ο Heath όμως πιστεύει ότι η πληροφορία του Διογένη είναι λάθος και οφείλεται σε παρανόηση. Κατά τη γνώμη του Πλάτων δεν επι­νόησε την αναλυτική μέθοδο.

Ο Αρχύτας ο Ταραντίνος υπήρξε μια πραγματι­κά πολυδιάστατη προσωπικότητα. Α ναφέρεται ως πολιτικός, φιλόσοφος, στρατιωτικός, αλλά επίσης και ως μηχανικός, εφευρέτης, aστρονόμος, μουσικός και γεωμέτρης. Το σίγουρο πάντως είναι ότι αν και προ­σωπικός φίλος του Πλάτωνα δεν συμμεριζόταν κα­θόλου τις ιδεολογικές προκαταλήψεις του φίλου του. Αυτό αποδεικνύεται από τις μαθηματικές και άλλες ενασχολήσεις του Αρχύτα.

Πρώτα απ' όλα ο Αρχύτας πέτυχε να λύσει το Δήλιο πρόβλημα με μια ευφυέστατη στερεομετρική κατασκευή. Επινόησε αρκετά θεωρήματα και προτά­σεις πάνω στην πυθαγόρεια θεωρία της μουσικής και στη θεωρία των λόγων. Το κυριότερο όμως είναι ότι ο Αρχύτας, σύμφωνα με τον Διογένη Λαέρτιο (VIII 79-83), ήταν ο πρώτος που εισήγαγε τη χρήση των μαθηματικών στη μελέτη της μηχανικής. Α ναφέρεται ότι σχεδίασε και ο ίδιος μερικές θαυμαστές μηχανές, όπως για παράδειγμα ένα ξύλινο περιστέρι που είχε τη δυνατότητα να πετά. τέλος, στον Αρχύτα αποδί­δεται και η ευφυής προσεγγιστική μέθοδος για τον υπολογισμό των τετραγωνικών ριζών.

Ο Θεαίτητος ο Αθηναίος γνωρίζουμε ότι ασχο­λήθηκε με δύο κυρίως θέματα: με τη θεωρία των αρρήτων και με τα πέντε κανονικά στερεά. Από τον πλατωνικό διάλογο που φέρει το όνομά του, μαθαί­νουμε ότι ο Θεαίτητος απέδειξε ότι κάθε αριθμός της μορφής � είναι aσύμμετρος, όταν ο κ δεν είναι τετράγωνο ακεραίου αριθμού. Το όνομα του Θεαίτη­του συνδέεται επίσης και με την γεωμετρική κατα­σκευή των κανονικών στερεών, ειδικά του οκταέ­δρου και του εικοσαέδρου. Ο Heath είναι βέβαιος ότι όλη η σχετική θεωρία των "Στοιχείων" οφείλεται στον Θεαίτητο.

ΕΥΚΛΕΙΔΗΣ Β' τ.Ι/10

Page 13: Ευκλειδης Β 61

π I

Η στήλη αυτή έχει ως στόχο την ανάπτυξη μαθηματικού διαλόγου. Φιλοδοξούμε να συμμετάσχουν όλοι

όσοι έχουν ένα γενικότερο ενδιαφέρον για τα Μαθηματικά.

Επιμέλεια: Γιάννης Στρατής - Βαγγέλης Ευσταθίου

Στο τεύχος 53 του περιοδικού μας (Ιούλι­ος - Αύγουστος - Σεπτέμβριος 2004) στο κομμάτι που αναφέρεται στη Α! τάξη του Λυκείου είχαμε δημοσιεύσει ένα άρθρο με τίτλο: «Για να κατα­νοήσουμε καλύτερα τους αριθμούς» (Γιάννης Στρατής). Ήταν μια σειρά θέματα πάνω στις ταυ­τότητες, τις εξισώσεις α! βαθμού, τις μόνιμες ανι­σότητες (και με περιορισμό), τις ανισώσεις α! βαθμού. Το άρθρο συμπληρωνόταν με μερικά προβλήματα (συνολικά 5 · δύο στις εξισώσεις, τρία στις ανισότητες) .

Στη σελίδα 1 9 είχαμε δημοσιεύσει το παρα­κάτω ζήτημα:

Αφού αποδειχθεί η ταυτότητα: ( α2 + β2 + γ2 )( χ2 + / + z2 )- ( αχ + βy + γz )2 =

= (βχ - αy )2 + ( γy - βz )2 + ( γχ - αz )2 , να βρεθεί στο εσωτερικό τριγώνου ΑΒΓ σημείο Ρ

έτσι ώστε: «αν χ, y, z είναι οι αποστάσεις του Ρ απ' τις πλευρές του τριγώνου και χ + y + z = c

όπου c κάποιος θετικός αριθμός σταθερός ( κατάλ­ληλος) το άθροισμα χ 2 + y2 + z2 να είναι το μι-κρότερο δυνατό» (το 'χε προτείνει ο συνάδελφος Θανάσης Κυριακόπουλος, μέλος της Συντακτικής Επιτροπής).

Πρέπει να παραδεχθούμε κατ' αρχάς ότι η λέξη «κατάλληλος» δεν υπήρχε στο κείμενο που είχαμε δημοσιεύσει. Απ' την όλη όμως διαπραγμάτευση του ζητήματος γινόταν φανερό ότι ο c είναι μεν σταθε­ρός, αλλά εννοείται ότι είναι προσδιοριστέος.

Ο συνάδελφος Παναγιώτης Οικονομάκος μας απέστειλε με γράμμα του (Γενάρης 2005) την πα­ρακάτω παρατήρηση :

«Το πρόβλημα 3 της σελίδας 1 9 (περιόδου 1-ουλ. - Αυγ. - Σεπτ. 2004) έχει λανθασμένη υπόθε­ση, διότι αν Ρ μεταβλητό σημείο στο εσωτερικό τυχαίου τριγώνου ΑΒΓ και χ, y, z είναι οι αποστά­σεις του από τις πλευρές του, τότε το άθροισμα χ + y + z δεν είναι σταθερό (Υπογράμμιση δική μας) .

Παρακάτω δείχνουμε με δύο τρόπους ότι η πα­ρατήρηση του συναδέλφου δεν ισχύει. Ακριβέστε­ρα υπάρχουν άπειρα σημεία Ρ στο εσωτερικό ενός σκαληνού τριγώνου, που το άθροισμα: χ + y + z των αποστάσεών του απ' τις αντίστοιχες πλευρές του τριγώνου είναι ένας σταθερός αριθμός c, ο οποίος διατρέχει ένα συγκεκριμένο διάστημα.

Ένας Δεύτερος Τρόπος 2"ς τρόπος (Γιάννης Δ. Στρατής)

Χωρίς βλάβη της γενικότητας δεχόμαστε ότι: α :::; β :::; γ . Α ν ρ η ακτίνα του εγγεγραμμένου κύ-κλου, τότε ισχύουν οι συνθήκες:

2Ε α + 2β 2α + β 2Ε (l) -- < ----ρ < ----ρ < --γ β α α Πραγματικά: 2Ε α + 2β α + β + γ α + 2β

• -- < ---- ρ � ρ < ---- ρ � γ β γ β � αβ + β2 + βγ < αγ + 2βγ � αβ + β2 < αγ + βγ � (α + β )β < (α + β )γ , που είναι αληθής.

ΕΥΚΛΕΙΔΗΣ Β ' τ. Ι/11

Page 14: Ευκλειδης Β 61

Το Βήμα του Ευκλείδη

α + 2β 2α + β 2 2 • --ρ <--ρ � α + 2αβ < 2αβ + β � β α

� α2 < β2 , που είναι αληθής. 2α + β 2Ε 2α + β α + β + γ

• --ρ <-�--ρ < ρ � α α α α � 2α + β < α + β + γ � α < γ , που είναι αλη-θής. Α ζ , , θ' 2Ε να ητουμε τωρα τη εση του β ως προς τις

ανισοτικές σχέσεις ( Ι ) α + 2β 2Ε Κατ' αρχάς έχουμε: --ρ < - . β β

Αφού : α + 2β α + β + γ --ρ < ρ � α + 2β < α + β + γ � β β � β < γ , που είναι αληθής.

• Έχουμε την ισοδυναμία: 2Ε 2α + β α + β + γ 2α + β -<--ρ � ρ <--ρ β α β α

β2 α2 + αβ � α2 + αβ + αγ < 2αβ + β2 � γ < - .

Συνεπώς διακρίνουμε τις περιπτώσεις: β2 - α2 + αβ (Ι): γ < .:...._ __ ..:.... α β2 - α2 + αβ (Π): γ 2 . α

Στην πρώτη περίπτωση ισχύει:

α

2Ε < α + 2β ρ < 2Ε < 2α + β ρ < 2Ε (3) γ β β α α Στη δεύτερη περίπτωση ισχύει:

2Ε < α + 2β ρ < 2α + β ρ::ς2Ε < 2Ε (4) γ β α β α

Εκλέγουμε τώρα ένα σταθερό c απ' το ανοικτό διάστημα: [ α + 2β 2α + β ) , -β

-ρ,-α-ρ κι ας ειναι

!ο, αν :

κ -2Ε - βc , αν : γ - β

2Ε - βc Οπότε: κ 2 Ο και κ 2 . γ - β

2Ε c 2 -β 2Ε c < -β

(5)

Αν ισχύουν οι συνθήκες (3) ο c δύο δυνατότη­τες έχει: . α + 2β 2Ε , 2Ε - βc (ι): --ρ < c <-. Οποτε: κ = . β β γ - β

, 2Ε - αc Κι επομενως: κ < (7). γ - α 2Ε - βc 2Ε - αc Αφού: < � γ - β γ - α

�2Ε( γ-α) -β( γ-α) c<2Ε{ γ-β) -α( γ-β) c � �2Ε[(γ-α) -(γ-β)] < c[ -{l( γ-β) +β(γ-α)J � � 2Ε < γc , που είναι αληθής.

( . . ) 2Ε 2α + β 0 , 0 Κ , ιι: -::ςc <--ρ . ποτε: κ = . ι επομε-β α νως: ισχύει η (7).

2Ε Αφού: 2Ε - αc > Ο � c > - , που είναι αλη­α θής. Αν ισχύουν οι συνθήκες (4) τότε: c < 2Ε . β ο , 2Ε - βc Κ , , (7) ποτε: κ = . ι επομενως: ισχυει η .

γ - β Δηλαδή σε κάθε περίπτωση ισχύει:

2Ε - αc , 'ζ , κ < και συνεπως ορι εται το ανοικτο γ - α

διάστημα: [κ, 2Ε - αc ) . γ - α Στη συνέχεια αποδεικνύουμε ότι: «για κάθε z ε [κ, 2Ε - αc ) το σύστημα: γ - α {x+y=c-z } έχει μοναδική θετική λύση» αχ +βy=2Ε-γz

Απόδειξη

Η ορίζουσα D του συστήματος είναι: D = β - α > Ο . Και συνεπώς το σύστημα είναι Cramer 2 χ 2 . Βρίσκουμε τώρα τις ορίζουσες Οχ και DY του συστήματος.

Έχουμε: Dx = βc - 2Ε + (γ - β ) z , DY = 2Ε - αc - (γ - α) z

D βc - 2Ε + (γ - β ) z και συνεπώς: χ =_χ = , D β - α

ΕΥΚΛΕΙΔΗΣ Β' τ. Ι /12

Page 15: Ευκλειδης Β 61

Το Βήμα του Ευκλείδη

y = Dy = 2Ε - αc - (γ - α)z D β - α

Έχουμε τώρα, λόγω της (6), 2Ε - βc 2Ε - αc

_ ____..:._ < z < . γ - β γ - α

(8)

Οπότε: Ο < βc - 2Ε + ( γ - β ) z , 2Ε - αc - (γ - α) z > Ο . Δηλαδή : χ > Ο , y > O .

Προφανώς ισχύει: 2Ε Ο < z < ­γ (9)

, 2Ε - αc 2Ε Αφου : z > κ � Ο και z < < - διότι: γ - α γ (2Ε - αc)γ < (γ - α) 2Ε � -αcγ < -α2Ε ή

2Ε . c > - , που ειναι αληθής. γ Εύκολα διαπιστώνεται ακόμη ότι: χ < hα,

y < hβ ( 1 0)

Ας είναι τώρα (χ , y) μια κάποια λύση του συ­στήματος.

Φέρουμε την ευθεία ε1 παράλληλη προς τη ΒΓ στο ημιεπίπεδο που βρίσκεται το Α και την ε2 παράλληλη προς την ΑΓ στο ημιεπίπεδο που βρί­σκεται το Β. Προφανώς τέμνονται σ' ένα σημείο Ρ στο εσωτερικό του τριγώνου.

Αποδεικνύουμε ότι: (ΡΜ ) = z . Πραγματικά: απ' τη δεύτερη εξίσωση του συ-. , Ι Ι Ι στηματος εχουμε: -αχ + -βy = Ε - -γz ή ισοδύ-2 2 2

. Ι ναμα. Ε( BPr) + E(ri>A) = Ε( ArB) -2γz ·

Και συνεπώς: Ε 1 ,Ι( )Ι ( " ) = -γz η -γ ΡΜ = -γz .

ΑΡΒ 2 2 2 Κι επομένως: z = ( ΡΜ ) .

Συνεπώς το σύνολο των σημείων Ρ, των οποίων οι αποστάσεις χ, y, z απ' τις πλευρές ΒΓ, Γ Α, ΑΒ είναι αντίστοιχα χ, y, z κι έχουν την ιδιότητα χ + y + z = c · όπου c μια σταθερή του ανοικτού διαστήματος: ( α + 2β 2α + β ) _β_'_α_ είναι ισοδύναμο με το ανοικτό

διάστημα: (κ, 2Ε - αc ) δηλαδή έχει τη δύναμη γ - α του συνεχούς.

Σ ' ένα δεύτερο επίπεδο αποδεικνύουμε ότι: «το Ρ γράφει το εσωτερικό r.νός συγκεκριμένου ευ­

θυγράμμου τμήματος» Πραγματικά: Αν απαλείψουμε το z απ' τις εξι­

σώσεις (8) φτάνουμε στην εξίσωση : (γ - α) χ + (γ - β ) y = cγ - 2Ε (Ι Ι)

γ

βημΓ = 2; ______ Α(βσυνΓ, βημΓ)

θ=90'-Γ Χ = γc-2Ε

1 (γ-α)εφΓ γ = γc-2Ε

I γ - α Σ( γc-2Ε Ο) (γ-α)ημΓ'

χ�· �����--���� Γ γ·

Ας πάρουμε την περίπτωση Γ < 90° [όμοια κα­τά βάση εργαζόμαστε στην περίπτωση : Γ > 90° . Η περίπτωση Γ = 90° είναι πιο απλή] .

Φέρνουμε την κάθετη της ΓΒ στο Γ και παίρ­νουμε τη διεύθυνση ΓΒ ως τη θετική διεύθυνση Χ 'Χ, ενώ η κάθετη της ΓΒ στο Γ προς το μέρος του Α λαμβάνεται ως θετική διεύθυνση του άξονα ΥΥ.

Αναζητούμε τις σχέσεις που συνδέουν τα χ, y με τις συντεταγμένες Χ = (ΓΚ ) , Υ = (ΓΗ ) του σημείου Ρ.

Επειδή τα μέτρα των γωνιών <rHP Λ και <rΗΓ Λ είναι ίσα [ αφού έχουν τις πλευρές τους κάθετες] συμπεραίνουμε ότι: χ = (ΓΗ ) = Υ και y = (ΘΡ)συνθ = (ΘΡ )ημΓ .

ΕΥΚΛΕΙΔΗΣ Β ' τ.l/13

Page 16: Ευκλειδης Β 61

Το Βήμα του Ευκλείδη

Αλλά: (ΘΡ) = (ΗΡ) - (ΗΘ) = Χ - (ΓΗ) εφθ = Χ - ΥσφΓ .

Άρα: χ = Υ και y = ΧημΓ - ΥσυνΓ ( 1 2) Έτσι η ( 1 1 ) γίνεται: [(γ - β)ημΓJ Χ + [(γ - α) - (γ - β)συνΓ] Υ =

= cγ - 2Ε , ( 1 3) , που είναι εξίσωση κάποιας ευθείας ε.

Αναζητούμε τώρα το σημείο Ν όπου τέμνονται οι ε και ΓΑ (εφ ' όσον φυσικά υπάρχει) . Κι επειδή η εξίσωση της ΓΑ είναι Υ = ΧεφΓ ( 1 4), η ( 1 3), λόγω της ( 14 ) , γίνεται: Χ =

( cγ -

)2Ε και συνε-γ - α εφΓ

πώς Υ = cγ - 2Ε . Δηλαδή η ε τέμνει τη Γ Α στο γ - α σημείο

Ν ( cγ - 2Ε cγ - 2E J (γ - α) εφΓ ' γ - α ( 1 5)

Για να δείξουμε ότι το Ν είναι εσωτερικό ση­μείο του ευθύγραμμου τμήματος Γ Α δείχνουμε τις ανισότητες:

θεία που διέρχεται απ' το Ν της Γ Α κι έχει συ­ντελεστή διεύθυνσης λε με

λ = - (γ - β)ημΓ ( 1 8) ε (γ - α) - (γ - β) συνΓ που είναι αρνητικός αριθμός [αφού : (γ - β)ημΓ > Ο και γ - α > γ -β 2:: (γ - β)συνΓ ] . Ο συντελεστής διεύθυνσης της ΑΒ είναι: λΑΒ

__ Ο - βημΓ ( 1 9), που είναι αρνητικός α­α - βσυνΓ ριθμός [αφού :

Ισχυριζόμαστε ότι: λε > λΑΒ (20)

Πραγματικά: Λόγω των ( 1 8) και ( 1 9) έχουμε:

λ - λΑΒ = βημΓ (γ - β)ημΓ = ε α - βσυνΓ (γ - α) - (γ - β)συνΓ

= γ (β - α)ημΓ > Ο (α - βσυνΓ) [( γ - α) - ( γ - β)συνΓ J

Συνεπώς οι ευθείες ε και ΑΒ τέμνονται· και

και

cγ - 2Ε 0 < ( ) < βσυνΓ γ - α εφΓ

ο cγ - 2Ε 2Ε < < -

( 1 6) μάλιστα το σημείο τομής τους - ας πούμε Τ - δε μπορεί να είναι στην προέκταση της ΑΒ προς την πλευρά του Α. Άρα το Τ ή θα ανήκει στο ευθύ­γραμμο τμήμα ΑΒ, συμπεριλαμβανομένου του Β ή

γ - α α ( 1 7)

• Για τη ( 1 6) : το πρώτο σκέλος είναι προφανές [ αφού έχουμε δεχθεί: Γ < 90° ] το δεύτερο σκέ­λος είναι ισοδύναμο με τη σχέση : cγ - 2Ε < β (γ - α)ημΓ . Η τελευταία είναι ισοδύναμη με τη συνθήκη : cγ -;Ε < βγημΓ -�

<=> cγ < βγημΓ <=> c < βημΓ = αβημΓ = 2Ε , που

είναι αληθής. α α

• Για τη ( 1 7) : το πρώτο σκέλος είναι προφανές. Το δεύτερο είναι ισοδύναμο με τη συνθήκη : αγc - 2αΕ < 2γΕ - 2αΕ <:::::> αγc < 2γΕ

2Ε , λ θ ' <:::::> c < - , που ειναι α η ης. α Η ε λοιπόν μπορεί να περιγραφεί ότι είναι η ευ-

θα ανήκει στο εσωτερικό του ΒΓ, οπότε το ονομά­ζουμε Σ. Τελικά ο γεωμετρικός τόπος του Ρ είναι το εσωτερικό ενός ευθύγραμμου τμήματος ΝΤ ή ενός ευθύγραμμου τμήματος ΝΣ.

Οπωσδήποτε υπάρχει μια ασάφεια στον προσ­διορισμό του γεωμετρικού τόπου. Πότε το Τ είναι στο ΑΒ και πότε στο ΒΓ. Η ασάφεια αυτή αίρεται με την παρακάτω διαδικασία.

Για Υ = Ο , η ( 1 3) γίνεται:

[(γ - β)ημΓ] Χ = cγ - 2Ε ή Χ = ( cγ -

)2Ε . γ - β ημΓ

Δηλαδή η ε τέμνει την ΑΒ στο σημείο

Q ( cγ - 2Ε oJ (γ - β)ημΓ ' .

ΕΥΚΛΕΙΔΗΣ Β' τ. Ι /14

Page 17: Ευκλειδης Β 61

Το Βήμα του Ευκλείδη

Έχουμε τώρα:

( cγ -

)2Ε � α <=> cγ - 2Ε � αγημΓ - αβημΓ γ - β ημΓ

<=> cγ 2: αγημΓ [ αφού 2Ε = αβημΓ ] 2Ε <=> c � αημΓ = - . β

Έτσι διαμορφώνουμε το παρακάτω κριτήριο. 2Ε ' Q ' Β ' δ ξ ' Β • Α ν c � - , τοτε το ειναι το η ε ια του . β

Οπότε η ευθεία ε συναντάει το ευθύγραμμο τμήμα ΑΒ σ' ένα σημείο τ.

Α 2Ε , Q , , , Σ • ν c < - , τοτε το ειναι καποιο σημειο β του ΓΒ. Αντίστροφα: Αν ένα σημείο Ρ ανήκει στο εσω­τερικό ενός ευθυγράμμου τμήματος ΝΤ ή ΝΣ και είναι χ, y, z οι αποστάσεις του απ' τις πλευ­ρές του τριγώνου, θα αποδείξουμε ότι x + y + z = c .

Π ραγματικά : Ας είναι Χ, Υ οι συντεταγμένες του Ρ. Τότε: [( γ - β)ημΓ] Χ + [(γ - α) - ( γ - β)συνΓ] Υ = = cγ - 2Ε <=>

(12) ( γ - α)Υ + ( γ - β) (ΧημΓ - ΥσυνΓ] = cγ - 2Ε <=> <=> (γ - α) χ + (γ - β) y = cγ - 2Ε <=> γ( χ + y) - (αχ + βy) = cγ - 2Ε (2 1 ) Κι επειδή : 2 Ε = αχ + βy + γz

[ Ε(

. ) = Ε(

. ) + Ε(

. ) + Ε( . ) ]

ΑΒΓ ΑΡΒ ΒΡΓ ΓΡΑ

συμπεραίνουμε απ' την (2 1 ) ότι: γ (χ + y) = cγ - γz ή x + y = c - z

δηλαδή : x + y + z = c .

Σχετικά με το ακρότατο της παράστασης : χ2 + y 2 + z 2

Εύκολα αποδεικνύονται ο ι συνθήκες:

α + 2β 2α + β (1) : --.-ρ < 3ρ < --ρ και β α 2Ε - αc (11) : κ < ρ < ---γ - α

• Η (I) είναι ισοδύναμη με τη συνθήκη : α < β [που ισχύει από υπόθεση]

• Για το πρώτο σκέλος της (11) διακρίνουμε τις περιπτώσεις

Α 2Ε , Ο ..,. ν c � - , τοτε κ = < ρ . β 2Ε , 2Ε - βc ..,. Α ν c < - , τοτε κ = < ρ <=> β γ - β

<=> (α + β + γ) ρ - βc < (γ - β) ρ

<=> c > α + 2β ρ [που είναι αληθής λόγω της (2)] β • Για το δεύτερο σκέλος της (11) διαπιστώνουμε

, θ , 2Ε - αc , δ , οτι η συν ηκη : ρ < ειναι ισο υναμη με γ - α θ , 2α + β [ , λ θ , τη συν ηκη : c < --. -ρ που ειναι α η ης α

λόγω της (2)] . Έτσι οι (8) για c = 3ρ γίνονται:

( 2β - α - γ) ρ + (γ - β) z χ = �--�-��-β - α

(β + γ - 2α) ρ - (γ - α) z y = �--�-��-β - α Θεωρούμε τώρα τη συνάρτηση

f ( z) = χ 2 + y2 + z2 Οπότε:

' ( ) dx dy f z = 2x ·- + 2y ·- + 2z = dz dz

= 2 { γ - β [ ( 2β - α - γ) ρ + (γ - β ) z] -β - α β - α β - α

_ γ - α [ (β + γ - 2α) ρ _

(γ - α) z] + z} = β - α β - α β - α

(22)

(23)

=2{�[(γ-β)(2β-α�γ) -(γ-α) (β+γ-2α)]+ (β-α)

ΕΥΚΛΕΙΔΗΣ Β' τ. Ι/15

Page 18: Ευκλειδης Β 61

Το Βήμα του Ευκλείδη

Κι επομένως: ( ( z) = Ο <=:> z = ρ και

r' (z) > O <=:> z > ρ . Άρα η f παρουσιάζει (ολικό) ελάχιστο για z = ρ . Αντικαθιστώντας την τιμή του z = ρ στις (22) παίρνουμε: χ = y = ρ . Ο συνάδελφος 11 . Οικονομάκος στο γράμμα του, που προαναφέραμε, γράφει: «Στο τυχαίο τρίγωνο ΑΒΓ ακυρώνουμε τη λανθασμένη και περιττή υπόθεση :χ + y + z = c, αφού μας αρκεί ότι ισχύει πάντα

αχ + βy + γz = 2Ε ( Ι ) όπου Ε το εμβαδό του τριγώνου ΑΒΓ. Καθώς γνωρίζουμε ισχύει η ταυτότητα Lagrange: (χ2 + y2 + z2 )(α2 + β2 + γ2 ) - (αχ + βy + γz)2 =

= (αy - βχ)2 + (αz - γχ)2 + (βz - γy)2 (2) Οπότε, λόγω της ( Ι ), έχουμε:

(χ2 + y2 + z2 )(α2 + β2 + γ2 ) - 4Ε2 =

= (αy - βχ)2 + (αz - γχ)2 + (βz - γy)2 ::::: 0 Και συνεπώς

Άρα ελαχίστη χ2 + y2 + z2 είναι:

τιμή για το άθροισμα 4Ε2

2 2 2 και επιτυγχά-α + β + γ

χ Υ z δ λ δ ' ' ' νεται όταν: - = - = - , η α η οταν το σημειο α β γ Ρ συμπίπτει με το σημείο από το οποίο περνούν οι τρεις συμμετροδιάμεσοι 1 του τριγώνου»

Σχόλιο τη ς Σ . Ε . Το ελάχιστο που βρίσκει ο συνάδελφος είναι

ολικό. Το πρόβλημα 3 που είχαμε θέσει ζητούσε το ελάχιστο της παράστασης χ 2 + / + z2 με τον περιορισμό χ + y + z = c όπου c μια κατάλληλη σταθερή .

Το ζήτημα μοιάζει με το παρακάτω παράδειγ­μα:

f(ξ1 ) = fmax - - - - - - - - - - Ρ ι

Α I I I I

ι Ρ0 : CΊ

- � - - I : Ρ2 _ J _ _ _ ι _ _ _ _ _ L _ _ _ - - - - - � I I I I

Β f(x.,)

f(ξ2) = fmin ---+-α��Χ�ο ---ξ�--ξ�---βL_ __ __ I 2

Η συνάρτηση του σχήματος έχει την ιδιότητα: f { ξ2 ) � f ( χ ) � f ( ξ1 ) , για κάθε χ Ε [α, β] . Δηλαδή : f {ξ2 ) = min { f { χ ) Ι χ Ε [α, β]} Ενώ f {x0 ) = min {f { χ ) ! χ Ε [α, ξ1 ]} Με άλλα λόγια η τιμή f(x0) είναι τοπικό ελάχι­στο δηλαδή είναι η ελάχιστη τιμή όταν η ανε­ξάρτητη μεταβλητή διατρέχει το διάστημα [α,ξ, ] .

1 Το σημείο αυτό αναφέρεται στη βιβλιογραφία ως σημείο

Lemoine. Για την έννοια της συμμετροδιαμέσου και τα

σχετικά θεωρήματα μπορεί ο αναγνώστης να ανατρέξει στα

παρακάτω συγγράμματα:

• Νικ. Α. Κισκίρα «Θεωρήματα και Προβλήματα Γεωμετρί­

ας» Επιπεδομετρία βιβλίον τέταρτον -βιβλίον πέμπτο. Α­

θήναι I 973 . Σελ. 93 και πέρα.

• Αριστείδου Φ. Πάλλα «Μεγάλη Γεωμετρία» Τόμος Α '

Επιπεδομετρία τεύχος Α ' πρώτον και δεύτερον βιβλίον.

Έκδοσις Τρίτη Αθήνα \ 962 σελ. 227 και πέρα.

• Γιάννη Ντάνη «Γεωμετρία», Αθήνα 1 97 1 σελ. 254

• Νείλου Σακελαρίου «Θεωρητική Γεωμετρία» Τόμος Β' σελ. \ 85

ΕΥΚΛΕΙΔΗΣ Β' τ.l/1 6

Page 19: Ευκλειδης Β 61

Το Βήμα του Ευκλείδη

& Το Σωστό και το Λάθος σε σχέση με τους Προτασιακούς τύπους

Τα τελευταία χρόνια στις εξετάσεις έχει καθιε­ρωθεί να δίνονται και θέματα στα οποία οι μαθη­τές καλούνται να απαντήσουν αν μια πρόταση εί­ναι Σωστή (Αληθής) ή Λάθος (Ψευδής).

Όμως τις περισσότερες φορές οι εξεταστές α­κόμη και στις Πανελλαδικές Εξετάσεις αντί να αναφερθούν σε προτάσεις, αναφέρονται σε προτα­σιακούς τύπους μιας ή περισσοτέρων μεταβλητών με αποτέλεσμα, όπως θα δούμε ,παρακάτω να μην έχει καν νόημα η ερώτηση .

Ως γνωστόν λέγοντας μεταβλητή εννοούμε ένα σύμβολο π.χ. ένα γράμμα χ που χρησιμοποιούμε για να συμβολίσουμε ένα οποιοδήποτε στοιχείο ενός συνόλου Ω το οποίο έχει τουλάχιστον δυο στοιχεία* . Λέμε τότε ότι το χ διατρέχει το σύνολο Ω. Σε αντιδιαστολή κάθε στοιχείο του Ω λέμε ότι είναι μια σταθερά.

Προτασιακός τύπος μιας μεταβλητής χ λέγε­ται κάθε έκφραση που περιέχει τη μεταβλητή αυτή και γίνεται πρόταση όταν η μεταβλητή αντικατα­σταθεί με ένα οποιοδήποτε στοιχείο του συνόλου Ω;t:0 το οποίο διατρέχει η μεταβλητή χ. Συμβολί­ζεται δε με ρ( χ). Το σύνολο των στοιχείων ξ του Ω για τα οποία η p(ξ) είναι αληθής πρόταση λέγεται σύνολο αλήθειας του προτασιακού τύπου p(x) και συμβολίζεται με Α= {χεΩ/p(χ) } , λέμε δε ότι για κάθε χεΑ, ο προτασιακός τύπος p(x) ισχύει, επα­ληθεύεται.

(Σχετικά με προτάσεις και Προτασιακούς τύ­πους βλέπε τα άρθρα περί Μαθηματικής Λογικής του συναδέλφου Αντώνη Κυριακόπουλου στα τεύ­χη 57 έως 60).

Για παράδειγμα αν η μεταβλητή χ διατρέχει το σύνολο Ω= { 1 ,2,3 } τότε οι παρακάτω εκφράσεις

Ρ 1 (χ) : Το χ είναι άρτιος αριθμός Ρ2(χ) : χ+ 1>2 Ρ3(χ) : χ+ 1 2::2 Ρ4(χ): χ+2<3 είναι προτασιακοί τύποι με σύνολα αλήθειας Αι= {2 } , Α2= {2,3 } , Α3= { 1 ,2 ,3 }=Ω, Α4=0 Κάθε προτασιακός τύπος λοιπόν μπορεί να γί-

νει αληθής πρόταση στις εξής περιπτώσεις : 1 . Για κάποιες τιμές της μεταβλητής χεΩ 2 . Για όλες τις τιμές της μεταβλητής χεΩ

Του Γιώργου Τασσόπουλου

3 . Για καμιά τιμή της μεταβλητής χε Ω Για τις τρεις αυτές περιπτώσεις χρησιμοποιού­

με τις ακόλουθες τρεις εκφράσεις αντιστοίχως, που χαρακτηρίζονται ως αληθείς προτάσεις. 1 . Υπάρχει χ ε Ω ώστε να ισχύει p(x), Συμβολικά

ΞJ χεΩ, p(x) 2 . Για κάθε χ εΩ ισχύει p(x), Συμβολικά

V χεΩ, p(x) 3 . Δεν υπάρχει χε Ω ώστε να ισχύει ρ( χ), Συμ-

βολικά - 3 χεΩ, p(x), ή 3χ ε Ω, p(x)

Δεν είναι λοιπόν ορθό να ζητήσουμε από τους μαθητές να μας χαρακτηρίσουν κάποια από τις εκφράσεις Ρ 1 (χ), Ρ2(χ), Ρ3(χ), Ρ4(χ) ως αληθή ή ψευδή, διότι αυτές δεν είναι προτάσεις αλλά προ­τασιακοί τύποι. Μπορούν να γίνουν προτάσεις μό­νον όταν το χ αντικατασταθεί με κάποιο στοιχείο του Ω= { 1 ,2,3 } , ή αν προτάξουμε μια από τις εκ­φράσεις υπάρχει, για κάθε, δεν υπάρχει, όπως φαίνεται παρακάτω.

Η πρόταση : Για κάθε χεΩ ισχύει χ+ Ι >2 είναι ψευδής, αφού για χ= 1 δεν ισχύει.

Η πρόταση : Για κάθε χ ε Ω, ισχύει χ+ 12::2 είναι αληθής

Η πρόταση : Υπάρχει χεΩ ώστε να ισχύει χ+ 2<3 είναι ψευδής

Η πρόταση : Δεν υπάρχει χ ε Ω, ώστε να ισχύει χ+ Ι> 3 είναι ψευδής

Π αραδείγματα • Α ν χ ε JR , τότε να χαρακτηρίσετε ως αληθή

ή ψευδή την ισότητα ημ2χ=l+συν2χ Η έκφραση αυτή δεν είναι πρόταση αλλά προ­

τασιακός τύπος αφού εξαρτάται από τη μεταβλητή χ, άρα δεν μπορεί να χαρακτηριστεί ως αληθής ή ψευδής.

Αν όμως γράφαμε: Για κάθε χ ε JR , ισχύει ημ2χ= Ι +συν2χ, τότε θα ήταν μια ψευδής πρόταση αφού για χ=Ο δεν ισχύει, διότι 0\t: Ι + Ι 2

Α ν γράφαμε: Υπάρχει χ ε JR , ώστε να ισχύει ημ2χ= Ι +συν2χ, τότε θα ήταν μια αληθής πρόταση αφού για χ = π ισχύει, διότι Ι 2= Ι +02. Μάλιστα

2

' ' θ π ' π ισχυει για κα ε χ = κπ + - ενω για χ :1:- κπ + -2 2

δεν ισχύει, όπου κ ε Ζ . • Α ν το Ω είναι μονοσύνολο τότε συμβατικά το χ

θεωρείται μεταβλητή εκφυλισμένη σε σταθερά ΕΥΚΛΕΙΔΗΣ Β ' τ.l/17

Page 20: Ευκλειδης Β 61

Το Βήμα του Ευκλείδη

Αν ΑΒΓ ορθογώνιο τρίγωνο με υποτείνουσα Τέλος είναι γνωστό ότι μια έκφραση είναι ΒΓ και ύψος ΑΔ, τότε να χαρακτηρίσετε ως πρόταση όταν έχει (πλήρες και αυτοτελές) νό-αληθή ή ψευδή την ισότητα ΑΒ2=ΒΓ.ΓΔ ημα και μπορεί κατά ένα και μόνο τρόπο να (θέμα Πανελλαδικών) χαρακτηρισθεί ως αληθής ή ψευδής.

Η έκφραση αυτή δεν είναι πρόταση, αφού Για παράδειγμα δεν είναι ορθό να ρωτή-Δ σουμε αν είναι αληθής ή ψευδής η πρόταση : εξαρτάται από τη μεταβλητή χ = Α Β Γ που δι- χ (χ + ι ) αγράφει το σύνολο των ορθογωνίων τριγώνων. Για κάθε χ Ε JR. , ισχύει = χ + ι , αφού

Αν γράφαμε: Για κάθε ορθογώνιο τρίγωνο χ ισχύει ΑΒ2=ΒΓΤ Δ, τότε θα ήταν ψευδής πρό- η έκφραση αυτή για χ=Ο δεν έχει νόημα, διότι ταση

Αν γράφαμε: Υπάρχει ορθογώνιο τρίγωνο ώστε να ισχύει ΑΒ2=ΒΓ·ΓΔ τότε θα ήταν αλη­θής πρόταση, διότι ισχύει για κάθε ισοσκελές ορθογώνιο τρίγωνο.

Ανάλογα οφείλουμε να εκφραζόμαστε και σε περίπτωση προτασιακών τύπων με περισ­σότερες από δυο μεταβλητές.

Για παράδειγμα:

τότε το πρώτο μέλος της δεν ορίζεται. Κανονι-κά πρέπει να γράψουμε: Για κάθε χ Ε JR.

ισχύ­χ (χ + ι ) ει = χ + I (αληθής) και ανάλογα: Υ-χ

• χ (χ + ι ) πάρχει χ Ε JR. ώστε να ισχύει = χ + ι χ (αληθής).

• Αν α,β e [O,+oo), τότε να χαρακτηρίσετε ως

Στη δεύτερη περίπτωση όμως συνηθίζεται να μην καθορίζουμε εμείς το σύνολο των τι­μών του χ για τα οποία έχει νόημα αυτή η έκ­φραση (πεδίο ορισμού εξίσωσης ή ανίσωσης), αλλά αφήνουμε να το βρει ο ερωτώμενος. Δη­λαδή ρωτάμε αν: Υπάρχει χ Ε JR. ώστε να ι-

αληθή ή ψευδή την ισότητα

.Jα + β = ..Γα + Jβ Η έκφραση αυτή δεν είναι πρόταση αλλά προ­

τασιακός τύπος με δύο μεταβλητές. Αν γράφαμε: Για κάθε α,β ε [Ο,+οο), ισχύει

.Jα + β = -Γα + Jβ θα ήταν ψευδής πρόταση, αφού για α= ι ' β= ι δεν ισχύει, διότι .J2 * 2 .

Αν γράφαμε: Υπάρχουν α,βε [Ο,+οο), ώστε να ισχύει .J α + β = -Γα + Jβ τότε θα ήταν α­ληθής πρόταση, αφού για α= ι , β=Ο ισχύει.

• Α ν α,β ε � τότε να χαρακτηρίσετε ως αλη­θή ή ψευδή την ισοδυναμία:

α2+β2=0<::>α=Ο ή β=Ο, Επίσης η έκφραση αυτή δεν είναι πρόταση . Αν όμως γράφαμε: Για κάθε α,β Ε JR. ισχύει

ισοδυναμία: α 2 + β 2 = Ο <=> α = Ο ή β = Ο , τότε θα ήταν

ψευδής πρόταση αφού για α= ι , β=Ο η πρόταση ι 2+02=0 είναι ψευδής ενώ η πρόταση ( Ι =Ο ή 0=0) είναι αληθής. Όπως ξέρουμε η ισοδυνα­μία p�q, δύο προτάσεων p, q είναι αληθής, μόνον όταν οι p, q είναι συγχρόνως αληθείς ή ψευδείς.

Αν γράφαμε: Υπάρχουν α,β Ε JR. ώστε να ισχύει η ισοδυναμία: α2+β2=0<=:>α=Ο ή β=Ο, τό­τε θα ήταν αληθής πρόταση, αφού για α=Ο, β=Ο είναι αληθείς και δύο προτάσεις: 02+02=0 και (0=0 ή 0=0). Αλλά και για α= ι , β=2 είναι και δύο προτάσεις ι 2+22=0 ( ι=Ο ή 2=0) ψευ­δείς. Άρα η ισοδυναμία ισχύει και στις δυο πε-ριπτώσεις.

. χ(χ + ι) ι . σχυει = χ + και εννοουμε: χ Α ν από τα χ Ε JR. για τα οποία αρχικά έχει

νόημα αυτή η έκφραση κάποια επαληθεύουν τον προτασιακό τύπο που προκύπτει.

Με αυτή τη λογική θεωρείται ορθό να ρω-τήσουμε άν: Υπάρχει χ ε:ΙR ώστε ι.(;1 = if;J (αντί του κανονικού : Υπάρχει χ Ε [Ο, +οο ) ώστε να ισχύει ι� = if;J ). Δεν είναι ορθό όμως να ρωτήσουμε αν: Για κάθε χ Ε JR. ισχύει ι� = if;J , αφού για χ<Ο δεν έχει νόημα αυ­τή η έκφραση, διότι δεν ορίζεται το δεύτερο μέλος της. Το ορθό είναι να ρωτήσουμε αν: Για κάθε χ ε [Ο,+οο) ισχύει: ι.(;1 = {;!

Κλείνουμε με ένα ακόμη μη ορθώς διατυ­πωμένο ερώτημα Πανελλαδικών εξετάσεων: Να χαρακτηρίσετε ως αληθή ή ψευδή την ισό-

2 2εφα Δ . . τητα εφ α = 2 • εν προκειται για προ-ι + εφ α ταση αλλά για προτασιακό τύπο ο οποίος για άπειρα α=κπ ισχύει και για άπειρα

π κπ π . α =Ι- κπ + - , -+-, κπ , οπου κ ε Ζ δεν ι-2 2 4 σχύει. Μερικά ακόμη ενδιαφέροντα παραδείγ­ματα υπάρχουν στο άρθρο Απόλυτες τιμές -Ρίζες πραγματικών αριθμών, για την Α· Λυκείου της συναδέλφου Κατερίνας Κλάδη .

ΕΥΚΛΕΙΔΗΣ Β ' τ.Ι/18

Page 21: Ευκλειδης Β 61

Το Βήμα του Ευκλείδη

r Γύρω από το Θεώρημα του Θαλή :

Δυο Αξιοσημείωτες Παρατηρήσεις

1 . Από το Πυθαγόρειο θεώρη μα στο Θεώ­ρη μα του Θαλή .

Είναι γνωστό ότι ένας τρόπος για να αποδει­χθεί το Πυθαγόρειο θεώρημα είναι να χρησιμοποι­ηθεί η ομοιότητα των τριγώνων, η οποία στηρίζε­ται στο θεώρημα του Θαλή . Θα δείξουμε και το αντίστροφο, δηλαδή ότι με τη βοήθεια του Πυθα­γορείου θεωρήματος μπορεί να αποδειχθεί το Θεώρημα του Θαλή . Επίσης θα δώσουμε μια από­δειξη του θεωρήματος του Θαλή με τη βοήθεια της έννοιας του εμβαδού ευθύγραμμου σχήματος. Εί­ναι φανερό ότι για να αποδειχθεί το γενικό θεώρη­μα του Θαλή, αρκεί να αποδειχθεί το θεώρημα του Θαλή σε τρίγωνο.

I. Ειδική περίπτωση. Έστω κατ' αρχήν ορθογώνιο στο Α τρίγωνο

ΒΔ ΒΕ ΑΒΓ και ΔΕ//ΒΓ. Τότε ισχύει - = - (ισο­

ΒΑ ΒΓ

ΒΔ ΒΕ δύναμα - = - )

ΔΑ ΕΓ

Απόδειξη Έστω κ=ΒΔ, λ=ΔΕ, μ=ΒΕ

Β

Α Ζ β

Έχουμε μ2=κ2+λ2, α2=β2+γ2 ( 1 ) Γ

Δημήτρης Ι. �πουνάκης

Φέρνουμε ΕΖ κάθετη στην ΑΓ. Από το ορθο­γώνιο τρίγωνο ΕΖΓ, λόγω και του ορθογωνίου ΑΔΕΖ, έχουμε

ΕΓ2=ΑΔ2+ΖΓ2, ή (α-μ)2=(γ-κ)2+(β-λ)2, ή λόγω των ( 1 ) αμ=γκ+βλ ή αμ-γκ=βλ, ή α2μ2+γ2r -2αμγκ=β2(μ2 -κ2), ή μ2(α2-β2) +κ2(γ2+β2)-2αμγκ=Ο, ή μ2γ2+κ2α2 -2αμγκ=Ο, ή

( )2 0 , , μ κ , ΒΕ ΒΔ μγ-κα = ' η μγ=κα η α = y ' η

ΒΓ =

ΒΑ

1 1 . Γενική Περίπτωση Έστω για τυχόν τρίγωνο ΑΒΓ και ΔΕ//ΒΓ.

Τότε ισχύει ΑΔ ΑΕ , ΑΔ ΑΕ

- = - (ισοδυναμα - = - ) ΑΒ ΑΓ ΔΒ ΕΓ

Α

Β Γ

Απόδειξη Αν η γωνία Β είναι οξεία (βλ. σχήμα) φέρνου­

με ΑΗ κάθετη στη ΒΓ, οπότε είναι κάθετη και στη ΔΕ. Από το ορθογώνιο τρίγωνο ΑΒΗ, όπου ΔΖ/ /ΒΗ, καθώς και από το ορθογώνιο τρίγωνο ΑΓΗ, όπου ΖΕ/ /ΗΓ λόγω της περίπτωσης (Ι)

ΑΔ ΑΖ ΑΖ ΑΕ , ΑΔ ΑΕ - = - - = - οποτε - = - . ΑΒ ΑΗ ' ΑΗ ΑΓ ' ΑΒ ΑΓ Όμοια εργαζόμαστε αν η γωνία Β είναι αμ­

βλεία, ενώ αν είναι ορθή έχουμε την περίπτωση (1). Επίσης όμοια είναι η απόδειξη αν η ΔΕ τέμνει τις προεκτάσεις των ΑΒ, ΑΓ.

ΕΥΚΛΕΙΔΗΣ Β' τ.l/19

Page 22: Ευκλειδης Β 61

Το Βήμα του Ευκλείδη

2 . Το Οr.ώ ρ η μ α του Θαλή με τη βοήθεια Ως γνωστό στα σχολικά βιβλία η απόδειξη του της i:ννο ιας του r.μβαδ οίJ .

Με τη βοήθεια του τύπου για την εύρεση του εμβαδού τριγώνου θα αποδείξουμε το θεώρημα του Θαλή .

Π ρ όταση

Έστω τρίγωνο ΑΒΓ και ΔΕ//ΒΓ. Τότε ισχύει ΑΔ ΑΕ , ΑΔ ΑΕ

- = - (ισοδυναμα - = - ) ΒΔ ΕΓ ΑΒ ΑΓ

Λπόδr. ιξη

Επειδή τα τρίγωνα ΑΔΕ, ΒΔΕ έχουν το ίδιο ύψος από το Ε, έχουμε

Β

(ΑΔΕ ) ΑΔ (ΔΕΒ ) ΒΔ Α

Γ

Γ

Όμοια από τα τρίγωνα ΑΔΕ, ΔΕΓ έχουμε (ΑΔΕ ) ΑΕ

= (ΔΕΓ ) ΕΓ Αλλά τα τρίγωνα ΔΕΒ, ΔΕΓ είναι ισοδύναμα,

αφού έχουν την ίδια βάση ΔΕ και ίσα ύψη, λόγω ΔΕ//ΒΓ.

Έτσι από τις παραπάνω αναλογίες προκύπτει

ΑΔ ΑΕ - = -

ΒΔ ΕΓ

Διδακτικό Σχόλιο

θεωρήματος του Θαλή συνήθως παραλείπεται, κυ­ρίως λόγω δυσκολίας που υπάρχει στην περίπτωση άρρητου λόγου. Το θεώρημα όμως αυτό μπορεί να αποδειχθεί με τη βοήθεια της έννοιας του εμβαδού όπως είδαμε, όπου βέβαια η δυσκολία μεταβιβάζε­ται στην απόδειξη του τύπου του εμβαδού τετρα­γώνου με πλευρά άρρητο. Με την βοήθεια του τύ­που αυτού αποδεικνύεται, ως γνωστόν, ο τύπος για το εμβαδόν τριγώνου που χρησιμοποιήσαμε παρα­πάνω στο (2). Όμως το εμβαδόν έχει μια άμεση εποπτεία και φυσικότητα που το καθιστά σχεδόν πρωταρχική έννοια για τα σχολικά μαθηματικά. Έτσι αν προηγηθεί στην ύλη της Γεωμετρίας της Β ' Λυκείου η έννοια του εμβαδού, θα δώσει τη δυνατότητα να αποδειχθεί, κατ' αρχήν το Πυθαγό­ρειο Θεώρημα (με τη θαυμάσια απόδειξη του Ευ­κλείδη) αλλά και το Θεώρημα του Θαλή, με μια από τις παραπάνω αποδείξεις. Υπόψη ότι με τη βοήθεια του εμβαδού μπορούν να αποδειχθούν ευκολότερα και κομψότερα και πολλές άλλες γεω­μετρικές προτάσεις και μπορεί να αποτελέσει μελ­λοντικά βάση για ένα διαφορετικό τρόπο διδασκα­λίας της Γεωμετρίας στη Β ' Λυκείου.

Σχόλιο του συναδέλφου κ. Γ ιώργου τ ασσόπουλου

Αγαπητέ συνάδελφε, η εργασία σας κρίθηκε αρκετά ενδιαφέρουσα και δημοσιεύθηκε με μεγά­λη μας ευχαρίστηση . Με ικανοποίησε δε ιδιαίτερα το γεγονός ότι έχετε πλήρη συναίσθηση του ότι δε βασίζεται μόνο στο Πυθαγόρειο Θεώρημα με τη μορφή εμβαδών (ΑΒΔΕ)+ (ΑΓΖΗ)=(ΒΓΚΛ), όπως υπάρχει στα στοιχεία του Ευκλείδη αλλά στην ι­σοδύναμη μορφή β2+γ2=α2 •

z

Λ κ

ΕΥΚΛΕΙΔΗΣ Β' τ. Ι /20

Page 23: Ευκλειδης Β 61

Το Βήμα του Ευκλείδη

γ

β

Δηλαδή όπως ορθώς επισημάνατε θα πρέπει να προηγηθεί, το ότι το εμβαδόν του τετραγώνου Τ με πλευρά χ, είναι (τ) = χ2, του οποίου άμεση εφαρ­μογή είναι ο τύπος του εμβαδού ορθογωνίου, και τριγώνου

Όμως και η απόδειξη αυτού του τύπου και η απόδειξη του θεωρήματος Θαλή γίνονται ουσια­στικά με βάση την πρόταση ότι: Αν δυο πραγματι­κοί αριθμοί είναι άνισοι τότε υπάρχει μεταξύ τους ρητός αριθμός ή με βάση τον ισοδύναμο αριθμό ισότητας λόγων κατά τον Εύδοξο. Η διαφορά είναι ότι αν προηγηθεί το Θ. Θαλή τότε η σχέση

β2+y2=α2 προκύπτει με δυο απλές ομοιότητες τρι­γώνων, ενώ αν προηγηθεί το Πυθαγόρειο με τη μορφή β2+γ2=α2 τότε όπως βλέπουμε η απόδειξη είναι αρκετά πιο δύσκολη .

Επ' ευκαιρία παραθέτω και μια πιο σύντομη απόδειξη του Θ. Θαλή με βάση το Πυθαγόρειο Θεώρημα (από τις παραδόσεις του καθηγητή κ. Στυλιανού Νεγρεπόντη στο Μεταπτυχιακό τμήμα Διδακτικής και Μεθοδολογίας των Μαθηματικών) .

Αν λοιπόν συμπληρωθεί το σχήμα σας ώστε να δημιουργηθεί το παραλληλόγραμμο ΒΑΓΡ τότε προφανώς (ΔΑΖΕ) = (ΕΗΡΘ) ( 1 )

Οπότε ( 1 ) κ λ ::::> λ · ( γ - κ ) = κ (β - λ) ::::> λγ = κβ ::::> - = ­γ β

κz λz κz + ').! μz κ λ μ ::::> - = - = = - => - = - = -γ2 βz γz + βz αz γ β α

Με φιλικούς χαιρετισμούς Γ, Τασσόπουλος

Μουσική κα ι Η.λupοφοpικιί

Υπεύθυνος: Νίκος Ανδρουλακάκης - Μουσικομαθηματικός.

Ο Χάρης Στυλιανάκης (Β ' Λυκείου Κορυδαλλού)

με επιστολή του μας ζητά να δημοσιεύσουμε «α­

ναλυτικό άρθρο σχετικά με Μουσική και Μαθημα­

τικά». Το θέμα είναι τεράστιο, οπότε θα το καλύ­ψουμε σε συνέχειες, και μάλιστα επεκτεινόμενοι στην «Μουσική και Πληροφορική».

Λεπτομερέστερα, στο μέλλον θα εξετάσουμε πώς ο Πυθαγόρας συνέδεσε την αρμονία της Μου­σικής με την αρμονία του σύμπαντος και των α­ριθμών. Πώς με απλούς αριθμούς, δημιούργησε τις νότες, τις κλίμακες, τα διαστήματα. Πώς «μετρά­

με» τους ήχους με τη συχνότητα και τα Ντεσι­μπέλ-Λογαρίθμους. Πώς, όσες/οι ενδιαφέρονται, θα παίξουν στα πλήκτρα, πρακτικά, αρχικά με το ένα δάκτυλο, αγαπημένες τους μελωδίες και μετά,

αν θέλουν, να τις συνοδεύσουν με το αριστερό χέ­ρι, με απλές συγχορδίες και ρυθμούς. Ακόμα, πώς οπτικοποιούμε τους ήχους με γραφικές παραστά­σεις. Επίσης, πώς θα χρησιμοποιήσουμε τον ηλ. υπολογιστή μας, ακόμα κι αν δεν είμαστε μουσι­κοί, ώστε, διαλέγοντας τραγούδια, ν' αναλύσουμε την ενορχήστρωσή τους και να τα διασκευάσουμε, αλλάζοντας ρυθμούς και όργανα, απ' όποιο είδος μουσικής προτιμάμε.

Τέλος, θα είναι χαρά μας, όποιες/οι αναγνώ­στριες/ες μας θέλουν να ενημερωθούν για κάτι σχετικό και ιδιαίτερο, να μας το ζητήσουν στην ηλεκτρονική διεύθυνση i n t'o(ιι\1n1 s . � Γ

ΕΥΚΛΕΙΔΗΣ Β ' τ. t /2 1

Page 24: Ευκλειδης Β 61

�� '1. " """ "r -n"

�'���� , ο ο .

. ::;;., � , Jι Η'Ν' NιiTHeNAT/�Vf

Η Homo Mathematicus είναι μια στήλη στο περιοδικό μας, με σκοπό την ανταλλαγή απόψεων και την ανάπτυξη προβληματι­σμού πάνω στα εξής θέματα: I ) Τι είναι τα Μαθηματικά, 2) Πρέπει ή όχι να διδάσκονται, 3) Ποιοι είναι οι κλάδοι των Μαθημα­τικών και ποιο το αντικείμενο του καθενός, 4) Ποιες είναι οι εφαρμογές τους, 5) Ποιες επιστήμες ή κλάδοι επιστημών απαιτούν καλή γνώση των Μαθηματικών για να μπορέσει κάποιος να τους σπουδάσει. Για τους συνεργάτες της στήλης: παράκληση ! τα κείμενα της στήλης αυτής, ως προς το περιεχόμενό τους και ως προς το επί­πεδό τους, θα πρέπει να είναι συμβιβαστά με τα ενδιαφέροντα και το επίπεδο κατανόησης από μέρους των παιδιών.

Επιμέλεια : Γ. Κερασαρίδης, Β . Ταπεινός, Β . Καρκάvης, Β . Ζώτος I . "οι συνεργάτες της στιί).ης γράφουν-ερωτούν "

Ι " θέμα: Οι μέλισσες σπουδάζουν Μαθη ματικά και Αρχιτεκτονική Προλεγόμενα . Συνεχίζουμε με το δεύτερο και τελευταίο μέρος της ενδιαφέρουσας αυτής εργασίας.

«Το κυνήγι της βέλτιστης λύσης», του Παναγιώτη Τριπικέλη (Κόρινθος)

Το πρόβλημα στις τρεις διαστάσεις : (2ο Μέρος) Ωστόσο είναι φανερό ότι οι μέλισσες αντιμε- Έχοντας στο μυαλό του το πρόβλημα χωρητι-

τωπίζουν το πρόβλημα στις τρεις διαστάσεις ,αφού κότητας των κελιών στις τρεις διαστάσεις ο Ούγ­τα κελιά τους εκτός από πόρτες έχουν και εσωτερι- γρος μαθηματικός Fejes Toth διατύπωσε το «ισο­κό. περιμετρικό πρόβλημα κυψέλης» όπου αναζητείται

Εδώ τα πράγματα δυσκολεύουν και η μελέτη στις τρεις διαστάσεις το σχήμα του κελιού με τη του αντιστοίχου προβλήματος ξεπερνά τις γνώσεις μεγαλύτερη χωρητικότητα. Αν και μέχρι το 1 998 το του αναγνώστη . πρόβλημα δεν είχε λυθεί έχει αποδειχθεί ότι η λύση

Αυτό που αξίζει να αναφερθεί είναι ότι αν και που έχουν δώσει οι μέλισσες πλησιάζει αλλά δεν θα περίμενε κανείς τα κελιά της κυψέλης να είναι είναι η ΒΕΛ τΙΣΤΗ! ! κανονικά εξαγωνικά πρίσματα, αυτό τελικά δεν ι- Η αίσθηση του συντάκτη του άρθρου είναι ότι σχύει. Η κάθετη τομή ενός κελιού είναι κανονικό οι μέλισσες έχουν το θεωρητικό υπόβαθρο για να εξάγωνο ,αλλά το πίσω τοίχωμα του αποτελείται δώσουν λύση ,αλλά δυσκολεύονται στη κατα­από τρεις ίσους ρόμβους. Η αντοχή τους ,δε, είναι σκευή ! ! εξωπραγματική . Αν και το τοίχωμα του κελιού εί-ναι μόνο 0,05 χιλιοστά παχύ , το κελί αντέχει κατα-πόνηση 25 φορές μεγαλύτερη από το βάρος του.

[ e-βιβλιογραφία:www .microscopy­uk.org.uk/artsep98/hexagon.html]

2° θέμα : Εφαρμογές από τη «θεωρία του μαθηματικού μπιλιάρδου» Προλεγόμενα . Συνεχίζουμε με το τρίτο και τελευταίο μέρος της ενδιαφέρουσας αυτής εργασίας.

«Τα Μ αθηματικά και το " αεικίνητο " » (3" μέρος), από το Θάνο Λύπα Ακολουθήσατε λαθεμένο δρόμο αν προσπαθήσα- προκαλεί καθόλου διαμαρτυρίες. Σε πολλές περι­

τε να βρείτε κάποιο σφάλμα στη Φυσική - στο γε- πτώσεις αυτή η αδιαφορία για τις διαστάσεις των γονός, για παράδειγμα, ότι αγνοήσαμε τη διάδοση σωμάτων είναι δικαιολογημένη . Για παράδειγμα, η της θερμότητας δια μεταφοράς. Γιατί; Επειδή μπο- κίνηση ενός σώματος υπό την επίδραση δεδομένων ρούμε να δημιουργήσουμε κενό μέσα στο δοχείο. δυνάμεων που ασκούνται στο κέντρο της μάζας του Όμως, τι άλλο υπάρχει μέσα στη μικρή μας «μηχα- δεν επηρεάζεται από το μέγεθός του και το σχήμα νή» εκτός από Μαθηματικά και Φυσική; Το σφάλ- του. Ή, όπως απέδειξε ο Νεύτων, ένα σώμα που μα κρύβεται στο σύνορο ανάμεσα σ' αυτές τις δύο αποτελείται από πλήθος ομόκεντρων ομογενών επιστήμες - στη μετάβαση από τη φυσική διαδικα- σφαιρικών στρωμάτων έλκει τα άλλα σώματα όπως σία στο μαθηματικό της μοντέλο. τα έλκει η ίδια μάζα συγκεντρωμένη στο κέντρο

Θυμηθείτε ότι ασχοληθήκαμε με δύο σώματα του. Βλέπουμε, λοιπόν, ότι η αντικατάσταση ενός τοποθετημένα στις εστίες. Επομένως αγνοήσαμε τις σώματος με ένα σημείο είναι μια συνηθισμένη διαστάσεις των σωμάτων. Αυτός είναι ένας συνη- πρακτική στη Φυσική . Στην περίπτωσή μας, όμως, θισμένος τρόπος σκέψης στη Φυσική, όπου η φρά- αυτό οδηγεί σε σφάλμα. ση «τοποθετούμε ένα σώμα στο σημείο Μ» δεν Ας δούμε κατά πόσο εξακολουθούν να ισχύουν οι συλλογισμοί μας όταν ως σώματα θεωρήσουμε σφαί­

ρες με μη μηδενικές ακτίνες και με κέντρο τους τις εστίες της έλλειψης.

ΕΥΚΛΕΙΔΗΣ Β ' τ.Ι/22

Page 25: Ευκλειδης Β 61

ιΥ,ιΙΙ, ιΙΙιiΤιΥ&ιΙΙΑ//CΙ/f --------------

σχήμα Ι σχήμα 2 σχήμα 3 Θεωρήστε τρεις ακτίνες που ανακλώνται στο ίδιο Εάν μια ακτίνα εκπέμπεται από την εστία μιας

σημείο Μ της έλλειψης, το οποίο βρίσκεται στο α- έλλειψης, μετά την πρώτη ανάκλασή της θα περά­ριστερό της τμήμα (σχήμα Ι ) : μια ακτίνα που σει από την άλλη εστία. Α ν δεν υπάρχει εκεί κά­«προέρχεται» από την εστία F2 , και δύο ακτίνες ποιο σώμα να την εκτρέψει, ανακλάται και πάλι και ΑΜ και ΒΜ που προέρχονται από τα σημεία Α και επιστρέφει στην πρώτη εστία, έπειτα επιστρέφει Β, τα οποία απέχουν απόσταση r από την εστία F2• ξανά στη δεύτερη , κοκ. Αυτό είναι αρκετά προφα­Μετά την ανάκλαση , η πρώτη ακτίνα θα περάσει νές. Είναι λιγότερο προφανές, όμως, και αρκετά από την εστία F ι και οι άλλες δύο θα περάσουν από παράξενο, το ότι έπειτα από κάθε ανάκλαση η τ ρο­σημεία που απέχουν ίση απόσταση R από την F ι χιά προσεγγίζει την ευθεία που διέρχεται από τις [διότι γων. AMF2 =γων. F2ΜΒ=γων.Β Ι Μ δύο εστίες, και ότι οριακά ταυτίζεται με το τμήμα F Ι =γων. F I MA I ] . Από την ομοιότητα των ορ- αυτής της ευθείας το οποίο εμπεριέχεται στην έλ­θογωνίων τριγώνων AMF2 και F I MA I , έχου- λειψη . Όταν το πρώτο τμήμα της τροχιάς τέμνει με r/R= AF2/A I F I = F2M/ F I M. Το Μ βρίσκε- αυτή την ευθεία σ' ένα σημείο διαφορετικό από τις ται στο αριστερό μέρος της έλλειψης, και επο- εστίες, η τροχιά δεν περνά ποτέ από κάποια εστία.

Επιπλέον αν το σημείο τομής βρίσκεται έξω από το μένως F2M<F I M, και άρα R>r. Αυτό σημαίνει ευθύγραμμο τμήμα που συνδέει τις δύο εστίες, το ότι κάποιες από τις ακτίνες που θα ξεκινούν ίδιο θα ισχύει για όλα τα σημεία τομής (σχήμα 2). από μια σφαίρα ακτίνας r και με κέντρο στο F2 Το πιο εντυπωσιακό είναι ότι υπάρχει μια μικρότε­δεν θα επιστρέψουν ποτέ σ ' αυτήν έπειτα από ρη έλλειψη με τις ίδιες εστίες με την πρώτη, στην κάποιες aντανακλάσεις και ούτε θα χτυπήσουν οποία η τροχιά εφάπτεται έπειτα από κάθε ανάκλα­στην ιδίων διαστάσεων σφαίρα στο F l (αυτές ση . οι ακτίνες διασπείρονται) . Και το γεγονός αυτό Ομοίως, μια τροχιά που τέμνει το ευθύγραμμο καταστρέφει όλους τους συλλογισμούς και τις τμήμα μεταξύ των δύο εστιών (σχήμα 3 ) θα κάνει κατασκευές μας. .

το ίδιο έπειτά από κάθε ανάκλαση, και θα εφάπτε-Όσοι αγαπούν τη Φυσική μπορούν τώρα να ανα- τα ι μιας υπερβολής με τις ίδιες εστίες. (Υπερβολή

πνεύσουν ανακουφισμένοι: 0 Δεύτερος Νόμος της είναι η καμπύλη η οποία σχηματίζεται από όλα ε­Θερμοδυναμικής παραμένει απαραβίαστος. Και για κείνα τα σημεία που η διαφορά των αποστάσεών όσους αγαπούν τα Μαθηματικά έχουμε μερικά επι- τους από τις εστίες είναι σταθερή) πλέον περίεργα φαινόμενα που σχετίζονται με τη συ-μπεριφορά των ακτίνων που ανακλώνται σε ένα ελ-λειπτικό κάτοπτρο.

3° Θέμα: Για την τριχοτόμη ση της γωνίας Προλεγόμενα: Από το φίλο της στήλης μας , Νίκο Βαδιβούλη, λάβαμε μια εργασία σχετικά με το θέμα που ανέπτυξε ο Ιπποκράτης Δάκογλου ("Homo mathematicus", ΕΥΚΛΕΙΔΗΣ Β ' , τεύχος 58 ,). Εμείς το θεωρούμε σαν συμβολή στην ενημέρωση πάνω σ' αυτό το θέμα, γι' αυτό και το δημοσιεύουμε.

«Μια τριχοτόμος καμπύλη», του Νίκου Βαδιβούλη (Άρτα) Το πρόβλημα της τριχοτόμησης μιας τυχαίας πρόβλημα της τριχοτόμησης στηριζόμενοι στη θεω­

γωνίας είναι αλγεβρικό τριτοβάθμιο και αποδεί- ρία των κωνικών καμπύλων. χθηκε άλυτο με τον ευκλείδειο τρόπο μαθηματικής Σημειώνω ότι οι κωνικές δεν είναι οι μοναδικές συμπεριφοράς. αλγεβρικές καμπύλες που επιλύουν το πρόβλημα.

Ο ευκλείδειος τρόπος μαθηματικής συμπεριφο- Μια τέτοια, προσιτή στους μαθητές, θα πα-ράς δέχεται την ύπαρξη ως κατασκευαστική δυνατό- ρουσιάσω εκκινώντας από τις μετρικές σχέσεις της τητα με τη βοήθεια του κανόνα και του διαβήτη. ευκλείδειας γεωμετρίας.

Οι αρχαίοι Έλληνες με υπέρβαση του ευκλείδει­ου τρόπου κατασκευής, πέτυχαν να επιλύσουν το

Ας θεωρήσουμε, λοιπόν, την όχι ευθεία και μη μηδενική κυρτή γωνία ΧΑ Ψ και τα σημεία Β,Γ στις πλευρές της ΑΧ, Α Ψ αντιστοίχως, ώστε ( ΑΒ )=(Α Γ)= Ι .

ΕΥΚΛΕΙΔΗΣ Β ' τ.l/23

Page 26: Ευκλειδης Β 61

χ

Α ψ I

I (ε)

I I ψ

(- Ι ,α' ) f(2,α') Ι ,... e� ,

I ι' \ 1 (ξ, 2δυνΑ) I I

I/ ' , , (0, α2-2) I I \ ι , A( J ,O ιΔ(ξ,Ο) \ A( l ,O) �

χ ' I ο ' / χ χ ,' ο \ ,' Δ(ξ,Ο) I (��;_4) Ι \ I i (-2, α'-4) Ι

I ' Ι

', /

ψ ψ ' ψ ' ( ;, -;) σχ1Ί μα 4 σχ1Ί μα 5 σχ1Ί μα 6

στο τρίγωνο ΑΒΓ, του σχήματος που προηγήθηκε, τομείται με τριχοτόμο μήκος w=J3 - 1>0. λαμβάνουμε: α2= 1 2+ 1 2-2 · 1 · 1 · συνΑ� Αν θέσουμε (γων.ΧΑΨ)=Α, τότε με την εφαρ­-2συνΑ= α2-2, οπότε η εξίσωση : χ3-3χ+α2-2=0 κα- μογή του θεωρήματος του συνημιτόνου γωνίας για θίσταται ισοδύναμη της εξίσωσης: χ3-3χ-2συνΑ=Ο. κάθε ένα των διαστημάτων [-2,0] , [0, 1 ] , [ 1 ,2] ισχύ-

Αν τώρα θεωρήσουμε τη συνάρτηση f με ουν οι προϋποθέσεις του θεωρήματος Bo1zano. f(x)=x3 -3χ+α2 -2, διαπιστώνουμε ότι γι' αυτή και Άρα η εξίσωση f(x)=O, έχει ανά μία ρίζα σε καθένα Ακολούθως θεωρούμε την ύπαρξη των τριχοτόμων των διαστημάτων (-2,0), (0, 1 ), ( 1 ,2) με δεκτή τη ρί­ΑΤ, ΑΣ που τέμνουν την ευθεία ΒΓ στα σημεία ζα ξε ( 1 ,2), αφού πρέπει w=ξ- 1 >0. Με την προϋπό­Δ,Ε αντιστοίχως, ώστε το σημείο Δ να ευρίσκεται θεση : α2>2<=:>(γων.ΧΑ Ψ)>90° και ύστερα από τη μεταξύ των Β,Ε ( σχ11 μα 4 ) . μελέτη της συνάρτησης f, παρουσιάζουμε το στικτό Εύκολα διαπιστώνεται ότι τα τρίγωνα ΑΒΔ, της διάγραμμα στο 9ix 9i επίπεδο ( σχ11μα 5 ) ΑΓΕ είναι tσα, οπότε μπορούμε να θέσουμε: Η δεκτή ρίζα ξ παρουσιάζεται στο σχ11 μα 5 ίση (ΒΔ)=(ΓΕ)=y, (ΑΔ)=(ΑΕ)=w και (ΒΓ)=α με το μήκος (ΟΔ), οπότε για το τριχοτόμο μήκος w Επειδή για το διχοτόμο μήκος δα στο τρίγωνο ισχύει: w=(ΑΔ)=(ΟΔ)-(ΟΑ)=ξ- 1 >0. ΑΒΓ ισχύει: δ � =βγ[ 1 -(-α- )2] , κατ' αντιστοιχία Επειδή οι εξισώσεις:

β + γ χ3-3χ+α2-2=0 και χ3-3χ-2συνΑ=Ο

και στο τρίγωνο ΑΒΕ θα ισχύει: w= 1 -( α - Υ )2, w + 1 αφού w>O.

Από την εφαρμογή του θεωρήματος της εσωτε­ρικής διχοτόμου στο τρίγωνο ΑΔΓ προκύπτει: _Υ_=_!_ � . . . . �w3+3w2+α2-4=0. α - 2y w

Η τελευταία εξίσωση, ύστερα από την αντικα­τάσταση w=x - 1 γίνεται

χ3 -3χ+α2 -2=0. Είναι προφανές ότι: α2=2<=:>(γων.ΧΑ Ψ)=90° α2>2<=>(γων.ΧΑ Ψ)>90° και

α2<2<=:>(γων.ΧΑ Ψ)<90° Έτσι στην περίπτωση κατά την οποία

(γων.ΧΑ Ψ)=90°, η εξίσωσή μας λαμβάνει τη μορ-φή χ3 -3χ=Ο<=:> χ( χ+ J3 )(χ- J3 )=0, από την οποία προκύπτει η δεκτή λύση χ= J3 και εκείθεν η λύση w= J3 - 1 , που αποδεικνύει ότι η ορθή γωνία τριχο-

είναι ισοδύναμες, από τη δεύτερη προκύπτει η ισο­δύναμη προς τις δύο προηγούμενες εξίσωση : χ3-3χ=2συνΑ, η οποία έχει τρεις ρίζες πραγματικές ή το ίδιο ακριβώς η ευθεία (ε) : y=2συνΑ τέμνει την καμπύλη (C) : y= χ3-3χ σε τρία σημεία ( συ1μα 6 ) και το ζητούμενο είναι εκείνο με συντεταγμένες (ξ,2συνΑ) με ξ> 1 . Επιπροσθέτως από τη μελέτη της συνάρτησης h με h(x)= χ3 -3χ, προκύπτει ότι για τα τοπικά της ακρότατα ισχύουν hmax=h( - 1 )=2, hmin=h( l )=-2 .

Επίσης ότι η τεταγμένη τυχαίου σημείου της ευθείας (ε) ανήκει στο διάστημα (-2,2), αφού -2<2συνΑ<2, επειδή η γωνία ΧΑΨ, μέτρου Α, είναι κυρτή μη μηδενική και όχι ευθεία.

Με την προϋπόθεση ότι Α <90° , η απεικόνιση της καμπύλης ( c) και της ευθείας (ε), ύστερα από τη μελέτη τους, παρουσιάζεται στο 9ix 9i επίπεδο με το σχ1Ίμα 6 .

Από την προηγούμενη απεικόνιση συμπεραίνουμε ότι η ζητούμενη ρίζα είναι: w=(ΑΔ)=(ΟΔ)­(ΟΑ)=ξ- 1 , που εκφράζει το τριχοτόμο μήκος. Κατανοώντας τα προηγούμενα ο μαθητής, παραλαμβάνει από τον υπολογιστή του το τριχοτόμο μήκος και με τη βοήθεια του διαβήτη το μεταφέρει για χρήση στο χαρτί σχεδίασης.

Θέλω να σημειώσω ότι η τριχοτόμηση μιας μη κυρτής γωνίας ανάγεται στην τριχοτόμηση της αντι­στοίχου κυρτής και ότι η τριχοτόμηση της ευθείας γωνίας είναι σχεδόν απλή .

Επίσης θέλω να σημειώσω ότι ο P.L. Wantzel ανακοίνωσε το έτος 1 837 το άλυτο του προβλήματος με τον ευκλείδειο τρόπο μαθηματικής συμπεριφοράς, αφού απέδειξε ότι υπάρχουν άπειρες γωνίες που

ΕΥΚΛΕΙΔΗΣ Β' τ.l/24

Page 27: Ευκλειδης Β 61

τριχοτομούνται στοιχειωδώς και επισημαίνω ότι η διαπίστωση αυτή δεν αναιρεί το άλυτο του προβλήμα­τος στη γενική του μορφή.

Από τα προηγούμενα μπορεί ο μαθητής να δια- τρεις ρίζες πραγματικές με προσδιορισμό του προ­πιστώσει ότι η ευκλείδεια γεωμετρία τροφοδοτεί την ανάλυση με σπουδαίες ασκήσεις. Από αυτές που εξάγονται από το προηγούμενο πόνημα θα α­ναφέρω μόνο δύο και κατά την άποψή μου τις α­ντιπροσωπευτικότερες.

σήμου αυτών. ί\σκη ση 2 '1 : Αν Ο<α<2 και αε91, να μελετηθεί

η συνάρτηση h με h(x)= χ3 -3χ+α2 -2, να γίνει η γραφική της παράσταση στο 91χ 91 επίπεδο και να αποδειχθεί ότι η εξίσωση h(x)=O έχει τρεις ρίζες πραγματικές με προσδιορισμό του προσήμου αυ-Λσκηση l η : Να μελετηθεί η συνάρτηση f με

f(x)= χ3-3χ-2συνΑ,όπου Α το μέτρο γωνίας τριγώ­νου, να γίνει η γραφική της παράσταση στο 91χ 91 επίπεδο και να αποδειχθεί ότι η εξίσωση f(x)=O έχει

τών.

4" Θi;μα: Μ ισ: νi:α αντίλη ψη γω το μi:τρο γωνίας Π ρ ολr.γόμε,· α : Από το φίλο της στήλης Χρήστο Δ. Αγγελόπουλο, λάβαμε μια εργασία, με θέμα «Ένας νέος ορισμός της έννοιας ''μέτρο γωνίας"». Εμείς σας παραθέτουμε τις σκέψεις του και περιμένουμε τις παρατηρήσεις σας.

Υ Β '

Δ '

σχήμα 7

ο ρ ισμός ηnJ μf.τρου γωνiης " , του Χρι1στου Αγγελόπουλου (AOl'i \'ff.) k

Ορ ισμός του μi:ηHnJ γωνίας : Θεωρούμε (σχ. 7) σύστημα αναφοράς Oxy. Προσαρτούμε σ' αυτό

ένα τετράγωνο ΑΒΓ Δ, τυχαίας πλευράς, με τις διαγωνίους του πάνω στους άξονες. Θεωρούμε μια γωνία kox, μέτρου θ, με την αρχική πλευρά οχ πάνω στον ομώνυμο άξονα και την τελική πλευρά της ok να τέμνει την πλευρά (π.χ. ΑΒ) του τετραγώνου, στο Ζ. Συμβολίζουμε με s την αλγεβρική τιμή του ΑΖ.

Ορίζουμε σαν μέτρο της γωνίας κοχ και το συμβολίζουμε με θ, το μέ­γεθος

Λ Λ

θ = !__ R

Το μέτρο μ( θ ) της γωνίας θ (όπως φαίνεται από το σχήμα 7), δεν επηρεάζεται από την εκλογή του τετραγώνου, αφού για ένα άλλο τε­τράγωνο Α 'Β 'Γ Δ ' , τα τρίγωνα ΑΟΖ, Α Ό Ζ' είναι όμοια, άρα

θ=� =� R R Λ Λ

l:υμβι'lσεις : α) Στο εξής θα συμβολίζουμε με θ την ίδια τη γωνία και με μ( θ ) ή θ το μέτρο της γωνίας kox.

β) Η κίνηση, πάνω στην περίμετρο του τετραγώνου ΑΒΓ Δ, που έχει φορά αντίθετη προς την κίνηση των δεικτών του ρολογιού, θα θεωρείται θετική .

γ) Στην περίπτωση αυτή οι μονάδες μέτρησης των γωνιών δεν ονομάζονται πλέον μοίρες ή ακτίνια, ο­νομάστε τες όπως θέλετε (π. χ. "αγγελίνια") . Το μέτρο της πλήρους γωνίας θα το συμβολίζουμε με 2π* . Π ρ<φλημα : Με βάση τα παραπάνω, μπορείτε να αποδείξετε τους παρακάτω τύπους;

12 - R r: Λ π* Λ Υ θ Λ π* 2π*=4 -- => π*=2 ν2 , Os θ s- , εφ θ = - = ----τ:--::- με Os θ <-R 2 χ ν2 - θ 2

J2 - θ θ συνθ=---------.---ι , ημθ= ι

J2 { ι + θ2 - J2θ )2 J2 { ι + θ2 - J2θ )2 * Λ Π , με Οs θ s- , 2

i\il ια παρατi1J>ηση : Από τη μορφή του παραπάνω τυπολογίου προκύπτει ότι, σε αντίθεση με την κλασική Τριγωνομετρία, εδώ μπορούμε να υπολογίσουμε τους τριγωνομετρικούς αριθμούς οποιασδήποτε γωνίας, χωρίς να καταφύγουμε στους τριγωνομετρικούς πίνακες.

ΕΥΚΛΕΙΔΗΣ Β ' τ.Ι/25

Page 28: Ευκλειδης Β 61

h'ΩaΙιιιι•••• [}JO(]J fJ[j]iJ/ liJ1 fJ(/j{J[j] fJ(f)fJ) c6Jr!JωC3fJ(f)r!J

Άλγ.ε.βρα

Απόλυτες Τιμές - Ρίζες Πραγμα.τικών Αριθμών

Η απόλυτη τιμή ενός πραγματικού αριθμού α, συμβολίζεται με I α I και ορίζεται ως εξής:

ιαι= (Ι) ( ορισμος σχο ικου ι ιου {α, αν α� Ο , λ , β βλ' ) -α, αν α< Ο π.χ 1 3 1 = 3 , 1 0 1 = ο, I -5 1 = --(-5)=5 Ισοδύναμα ισχύει: {α, αν α > Ο ΙαΙ= (11), -α, αν α ::; Ο {α, αν α � Ο Ια Ι= (IV) -α, αν α ::; Ο

{α, ανα> Ο ΙαΙ= Ο, αν α= Ο (ΙΙΙ) , -α, ανα<Ο

Οι ορισμοί (I) και (11) φαίνεται να μεροληπτούν υπέρ των θετικών ή των αρνητικών αριθμών αντι­στοίχως, εVώ οι (ΠΙ) και (IV) είναι πράγματι αντι­κειμενικοί. Θα δούμε παρακάτω ότι για την απο­φυγή παρερμηνειών, συμφέρει να χρησιμοποιή­σουμε τον τελευταίο ορισμό

Εφαρμογή l η

Να λυθούν οι εξισώσεις: i) I χ I =χ, ii) I χ I = -χ

Λύση i) I χ I =χ � χ=::Ο, αφού για χ<Ο είναι αδύνατη . ii) Για την εξίσωση I χ I = -χ, πιθανόν να παρερ­

μηνευθεί ο ορισμός του σχολικού βιβλίου και οι μαθητές να απαντήσουν I χ I = -χ � χ<Ο, ενώ ο ορισμός λέει ότι χ<Ο => I χ I = -χ και όχι αντιστρόφως. Εξετά­ζουμε αν ισχύει και για χ=::Ο. Τότε Ι χ Ι = - χ <:::::> χ = -χ � 2χ = Ο � χ = Ο.

της Κατερίνας Κλάδη

Τελικά I χ I = -χ� χ::Ξ:Ο

Ε φαρμογή 2η

Να λυθούν οι ανισώσεις : i ) Ι χ Ι :::: χ, ii) Ι χ Ι :::: -χ, iii) Ιχ Ι>χ, iv) Ι χ Ι> -χ

Λύση Αφού I χ I =::χ και I χ Ι =::-χ, για κάθε χ ε IR , θα

έχουμε: i) I χ I :::: χ <=> I χ I =χ <=> χ:::ο ii) I χ I ::::-χ � I χ I = -χ� χ::::ο iii) I χ I > χ � χ <Ο ίν) l x l > -χ� χ>Ο

Εφαρμογή 3η Να λυθούν οι παρακάτω βασικές εξισώσεις -

ανισώσεις:

i) l -2x-l l =3 iii) l -3x+4 1 =-6 ν) l -2x+l i ::Ξ:3 vii) l 3x+ l l 2:-2

Λύση:

ii) I χ-1 1 = l 2x-5 l iv) Ι χ-4 1 =3χ+2 vi) Ι χ-2 1 >4 viii) l 4x-l l ::;ο

Για τη λύση των βασικών εξισώσεων, χρησι­μοποιούμε τις ισοδυναμίες: 1 . Ι χ Ι =α � χ=α ή χ= -α με α=::Ο 2 . l x l = l α l � χ=α ή χ= -α Για τη λύση βασικών ανισώσεων, χρησιμοποι­ούμε τις ισοδυναμίες: 3 . I χ I ::Ξ:θ � -θ::Ξ: χ::Ξ:θ, για θ>Ο. 4. I χ I :::θ � χ ::Ξ: -θ ή χ�θ για θ>Ο.

ΕΥΚΛΕΙΔΗΣ Β ' τ.Ι/26

Page 29: Ευκλειδης Β 61

Μαθηματικά για την Α' Λυκείου

i) l -2x-1 1 =3 <::> -2χ- 1=3 ή -2χ-1=-3 <=:>-2χ=4 ή -2χ= -2<::>χ=-2 ή χ= 1

ii) Ι χ- 1 1 = 1 2χ-5 Ι <=>χ-1=2χ-5 ή χ-1= =-2χ+5<=:>-χ= --4 ή 3χ=6 <::>χ=4 ή χ=2

ίίί) l -3x+4 1 =-6 αδύνατη, διόtι l-3x+4 1 :::ο για κάθε χ Ε IR

iν) Αφού δεν γνωρίζουμε το πρόσημο του 3χ+2. Διακρίνουμε τις περιπτώσεις:

α) Αν 3χ+2<0 δηλαδή χ< -2/3 τότε είναι αδύνατη . β) Αν 3χ+2:Ξ:Ο δηλαδή χ::: -2/3 τότε 1 χ--4 1 =3χ+2<=> χ--4=3χ+2 ή χ--4= -3χ-2 <=:>-2χ=6 ή 4χ=2<::> χ= -3 ή χ= 1 /2 -3 � [-% , + οο) άρα η τιμή χ= -3 απορρίπτεται,

1 [ 2 ) . 1 , δ , ενώ - Ε -- + οο άρα η τιμη χ = - ειναι εκτη . 2 3 , 2 Μια άλλη λύση δίνεται με απαλοιφή του απολύτου διακρίνοντας τις περιπτωσεις χ - 4 � Ο και χ - 4 � 0 .

Ένα λάθος που μπορεί να γίνει είναι να γράψου­με Ι χ--4 1 =3χ+2<::> χ--4=3χ+2 ή χ--4=

= -3χ-2 <::>χ= -3 ή x= l /2 ν) l -2x+ 1 1 ::; 3 <::> -3 :=:: -2χ + 1::;3 <=>

<=> - 4 :::: -2χ :::: 2 <=> 2 ::: χ :Ξ:-1<=:>---1::; χ:Ξ:2 νί) Ι χ-2 1 >4<::>χ-2<--4 ή χ-2>4<::>χ<-2 ή

χ>6 νiί) l 3x+ 1 1 :Ξ:-2 <=> χ Ε IR νiίί) l 4x-1 I ::;Ο <=> l 4x-I I =0<::>

<::>4χ-1 =Ο<::> χ= Ι /4

Εφαρμογή 4'1 Να λύσετε τις ανισώσεις:

i) d(x,2) :S 5 και ii) d(2x,l) :S Ο

iii) d(α5-7α,-6) :S -d(α2,α)

Λύση i) d(x,2) ::; 5<::> I χ-2 1 :Ξ: 5<=:>-5:Ξ:χ-2:Ξ:5

<=:>-3:Ξ:χ:Ξ:7 ii) d(2x, Ι ) ::; 0<::> l 2x-1 1 :::: 0<::>2χ-Ι = Ο

<::>χ= Ι /2 iii) d(α5-7α,-6):Ξ:-d(α2,α)<=>jιi -Ία+� �-1ιi -� <=>

<::> Ια5 - 7α + 6 1 + Ια2 - α ! � Ο

Ια5 - 7α + 6 1 = 0} α5 - 7α + 6 = 0} <=> <=> 2 lαz - α ! = Ο α - α = Ο

α5 - 7α + 6 = 0} α5 - 7α + 6 = 0} " <=> <=> ' <=> α(α - 1 ) = 0 α = Ο η α = l

<=> (α5 - 7α+ 6 = 0) ή (15� 7 · 1 + 6 = 0)<=>α= l α = Ο α - Ι

Άσκηση 1 '1 Να χαρακτηρίσετε με (Σ), Σωστό ή (Λ), Λάθος,

κάθε μια από τις παρακάτω προτάσεις και να

αιτιολογήσετε την απάντησή σας.

i) Για κάθε χ, ψ Ε IR , ισχύει: I χ-ψ I + χ-ψ:Ξ:Ο

ii) Για κάθε α, β Ε IR , ισχύει: I α-β I < I α I + I β I iii) Για κάθε χ, ψ Ε IR , ισχύει, I χ I + I Ψ I =Ο <=>

χ = Ο ή ψ = Ο

iv) Για κάθε α,β Ε IR , ισχύει, I α I =β<:::>α=β ή

α= -β

ν) Για κάθε χ Ε IR , αν ισχύει

Ι-3χ+12 1=3χ-12 τότε θα ισχύει χ:Ξ:4.

Λύση i) Σωστό, διότι Ι α I :::-α για κάθε α Ε IR . Άρα για

α= χ-ψ έχουμε: I χ-ψ I :Ξ:-( χ-ψ)::::> 1 χ-ψ 1 + χ-ψ:Ξ:Ο για κάθε χ, ψ Ε IR

ii) Λάθος, διότι π. χ. αν α = 5 και β = -3 τότε 1 5-(-3) 1 = 1 5 1 + 1 -3 1 Γενικά για κάθε α,β Ε ΙR ισχύει: I α - β I � I α I + I β I

ίίί) Λάθος, διότι αν χ=Ο, ψ=5 τότε η I ο I + I 5 I =Ο είναι ψευδής και η (0=0 ή 5=0) είναι αληθής

iv) Λάθος, διότι για α = 5 και β = -5 η ( 1 5 1 = -5) είναι ψευδής) ενώ η (5 = -5 ή 5 = -(-5)) είναι αληθής.

ν) Σωστό διότι για χ=α, συγκεκριμένο, με ι-3α + Ι 21 = 3α - Ι 2 , οι συνεπαγωγές

Ι -3α + Ι 2 1 =3α - 1 2::::::> -3α - Ι 2:Ξ:Ο ::::::> α:Ξ:4 είναι σωστές και η αρχική υπόθεση σωστή . Άρα και το συμπέρασμα σωστό.

Σημείωση : Κατά την απόδειξη την ανισότητας Ι α + β ι � ΙαΙ + ιβ ι για κάθε α, β Ε JR γίνεται φανερό ότι το ίσον ισχύει μόνο όταν αβ � Ο καθώς και

ΕΥΚΛΕΙΔΗΣ Β' τ.Ι/27

Page 30: Ευκλειδης Β 61

Μαθηματικά για την Α' Λυκείου

στην !α - β ! � !α ! + lβ l το ίσον ισχύει μόνο ό­ταν α ( -β ) � Ο δηλαδή, αβ � Ο .

Άσκηση 2 η Για κάθε α,β,γ,δ Ε IR . Να δείξετε ότι i) Αν α<β και l γ+δ l < l γ l +lδ l τότε αγδ>βγδ. ii) Ια+βi+Ια-βΙ = 2 ΙαΙ+2 Ιβ l � α·β = ο

Λύση i) Σύμφωνα με την προηγούμενη σημείωση :

I γ+δ I < I γ I +lδ l=>γδ<Ο. Άρα α<β=>αγδ>βγδ ii) Γνωρίζουμε ότι για κάθε α, β Ε IR ισχύει:

iα + β l � iα i + lβ l} Ια - β i � Ια i + Ιβ l

Άρα iα + β i + Ια - β i � 2 iα i + 2 lβ l Η ισότητα Ι α+β I + I α - β I = 2 1 α I + 2 1 β I ισχύει μόνον όταν: iα+� = l� + l�} (Σ) Αλλά (Σ)� αβ � Ο}�αβ=Ο !α-� = 1� + 1� αβ � Ο

)\.σκηση 3 '� Για κάθε α, β ε IR , να αποδείξετε ότι:

l α+β l + l α-β l = l α l + l β l � l α l = l β l ι\(Jση Ένας τρόπος για την απόδειξη σχέσεων με απόλυ­τες τιμές, είναι να υψώνουμε και τα δύο μέλη στο τετράγωνο, εφόσον βέβαια είναι θετικά ή μηδέν

' δ ' I 1 2 2 Έ και να χρησιμοποιουμε την ι ιοτητα χ = χ . -

τσι έχουμε: l α+β l + l α-β l = l α l + l β l � < Ι α+ β Ι + Ι α-β Ι )2=( I α Ι + I β I )2 � (α+β)2+2 1 α2-β2 1 +(α-β)2 =α2+2 1 αβ I +β2�2+β2-2 1 αβ I +2 1 α2-β2 1 =Ο�

lαl-l β i=O } < Ι α i - I β Ι )2+2 1 α2-β2 1 =ο� lα2 - β2 1 = 0 ( l α l-l β l )2 � Ο}

� Ι α l = l β l , αφού Ια2 - β2 ι � ο

Λ σ κη ση 4'� Για κάθε α, β Ε IR να δείξετε ότι: Α ν I α I <2 και

1 3α + 2β l Ι β Ι < 3 τότε: < 1 αβ + 6

Λύση

iα i < 2} => Ι α 1 · 1 β Ι <6=> I αβ I <6 ,β , < 3 =>-6<αβ<6=>αβ+6>0 άρα αβ+6*0

, 1 3α + 2β I 1 , δ 'ξ Για να δείξουμε οτι < αρκει να ει ουμε αβ + 6 ότι: l 3α+2β I < I αβ+6 1 , ή l 3α+2β 1 2< I αβ+6 1 2, ή (3α+2β)2<(αβ+6)2, ή 9α2+ 1 2αβ+4β2< α2β2+ 1 2αβ+36, ή α2(9-β2)-4(9-β2)<0, ή (9-β2)( α2-4)<0, η οποία ισχύει διότι: I α I <2=> I α 1 2<4=>α2 -4< Ο και Ι β Ι <3=> I β l 2<9=>9-β2>0.

Λ σ κη ση 5η (Απλοπο ί η ση παρ αστιΊσr.ων μr. από­λυτα) Να γράψετε τις παρακάτω παραστάσεις με μορφή χωρίς το σύμβολο της απόλυτης τιμής i) Α=2χ-Ι-3χ+1 Ι

ii) Β = ι ι + χ ι - ιι - χ Ι (άσκηση 2, σελίδα 43 της χ

Άλγεβρας Α' Λυκείου) και να βρείτε το ευρύτερο υποσύνολο του IR * στο οποίο η Β είναι σταθερή.

Λύση :

Π α ρ ατή ρη ση : Για να aπλοποιήσουμε μια παρά­σταση με απόλυτες τιμές, θα πρέπει να γνωρίζουμε το πρόσημο της παράστασης που βρίσκεται μέσα στο απόλυτο και να εφαρμόσουμε τον ορισμό της απόλυτης τιμής. π. χ. Ι χ2+2ψ2 Ι= χ2+2ψ2 διότι χ2+2ψ2Ξ':Ο για κάθε χ,ψ Ε IR ενώ 1 -α2+2αβ-β2 1 = 1 -(α-β)2 1 = (α-β)2, διότι -( α-β)2 s Ο για κάθε α, β Ε IR . Στην περίπτωση που η παράσταση η οποία βρίσκε­ται μέσα στο απόλυτο, έχει μεταβλητό πρόσημο, εξετάζουμε το πρόσημό της για τις διάφορες τιμές της μεταβλητής και μετά εφαρμόζουμε τον ορισμό της απόλυτης τιμής, κατά προτίμηση τον (IV).

ΕΥΚΛΕΙΔΗΣ Β' τ.Ι /28

Page 31: Ευκλειδης Β 61

Μαθηματικά για την Α ' Λυκείου

Ι -3χ + Ι 2': 0 � -3χ 2': -Ι � χ � - οπότε: 3 {2χ - (-3χ + Ι ) = 5χ - Ι , αν χ � .!_

Α = 3 Ι 2χ - (3χ - Ι ) = -χ + Ι, αν χ 2': -3

Ι Παρατηρείστε ότι για χ = - οι παραστάσεις 3 5χ - Ι και - χ + Ι παίρνουν και οι δύο την τι-

, 2 μη -3 ii) Στην περίπτωση αυτή, έχουμε δύο διαφορετι­

κές παραστάσεις μέσα στα απόλυτα, γι' αυτό θα εξετάσουμε το πρόσημό τους και θα χρησι­μοποιήσουμε συγκεντρωτικό πίνακα. Στο IR. * λοιπόν έχουμε:

Ι + χ ;::: Ο � χ Ε [- Ι, Ο) υ (Ο, + οο) και Ι - χ 2': Ο � χ Ε (-οο, Ο) υ (Ο, Ι]

χ - οο - Ι ο 1 Ι +χ - + ' + Ι -χ + + +

α) Αν χ Ε (-οο,- Ι ] τότε: Β = - Ι - χ - (1 - χ) = -2 χ χ

β) Αν χ Ε [- Ι , Ο) υ (Ο, Ι ] τότε:

+οο +

-

Β = Ι + χ - ( 1 - χ ) = 2χ = 2 χ χ γ) Αν χ Ε [Ι , +οο) τότε:

Β = Ι + χ - (- Ι + χ) = �

Άρα Β = 2, 2 χ

χ 2 '

χ χ αν χ Ε ( -οο, - Ι ] αν χ Ε [-Ι, Ο) υ (Ο, Ι ] αν χ Ε [Ι , +οο)

Η παράσταση λοιπόν Β είναι σταθερή στο [- Ι, Ο) υ ( Ο, Ι] και όχι μόνο στο (- Ι , Ο) υ (Ο, Ι ) . (Λάθος στο οποίο θα οδηγούνταν οι μαθητές από παρανόηση του ορισμού (II)). Ομοίως η παράστα-ση Β ισούται με 3_ στο [ l, +οο) και όχι μόνο στο χ ( Ι , +οο) . (Λάθος στο οποίο οδηγεί η παρανόηση του ορισμού(!)). Σύγχυση μεταξύ των δύο αυτών ορισμών συμβαίνει όχι μόνο στους μαθητές αλλά

και σε πολλά βιβλία. Το ίδιο το Σχολικό βιβλίο εδώ χρησιμοποιεί τον ορισμό (II) αντί του ορισμού (I) που έχει δώσει. Αποφεύγουμε λοιπόν σχετικά λάθη εφαρμόζοντας τον ορισμό (IV).

Ση μείωση Οφείλουμε να παρατηρήσουμε ότι ανάλογο πρό­βλημα εμφανίζεται και στις γραφικές παραστάσεις συναρτήσεων, χωρίς την έννοια της συνέχειας συ­ναρτήσεων. π. χ. για τη γραφική παράσταση της f(x) = I χ-2 1 +2χ . με τον ορισμό (I) του σχολικού βιβλίου έχουμε: f(x) = ' οπότε {3χ - 2 αν χ 2': 2

χ + 2, αν χ < 2 δεν είναι φανερό ότι οι δυο κλάδοι της γραφικής παράστασης της f διέρχονται από το σημείο Α(2,4). Με τον ορισμό όμως (IV) έχουμε: f(x) = ' και αυτό είναι προφανές. {3χ - 2 αν χ 2': 2

χ + 2, αν χ � 2 Τέλος εκεί που δεν μπορούμε να αποφύγουμε τον ορισμό (IV) είναι στην Γ Λυκείου για την εύρεση π.χ. του ολοκληρώματος ι ιχ�χ .

ψ

χ ' χ

t ψ' I

Άσκηση 6" Να βρεθούν τα χ Ε JR για τα οποία ισχύει:

2� l -2x+6 1 �.

Λύση Ουσιαστικά έχουμε να λύσουμε το σύστημα των δυο ανισώσεων: 2� l -2x+6 1 (I) και l -2x+6 I �4 (II) (I) � -2χ + 6 � -2 ή -2χ + 6 ?: 2 �

ΕΥΚΛΕΙΔΗΣ Β' τ.Ι /29

Page 32: Ευκλειδης Β 61

Μαθηματικά για την Α' Λυκείου

-2χ :5 - 8 ή -2χ 2: -4 <=> χ2:4 ή χ :5 2 1 7 Τελικά λοιπόν: Α = 4 <:::> χ = - ή χ = 1 3 . 5 (Π)<=> -4 :5 -2χ+6:54 <:::> -4 -6 :::; -2χ+6 -6:5 4- 6<:::> Εντελώς ανάλογα: -1 0:5 - 2x:S: - 2<:::> 5 2: χ 2:1 <::> I :::; χ :::; 5 . α) Στο (-οο,3] έχουμε : Πρέπει να συναληθεύσουμε τις δυο ανισώσεις: Α < 4 <=> -χ + 9 < 4 <=> -χ < -5 <=> χ > 5

οι τιμές αυτές απορρίπτονται διότι δεν ανήκουν -(1) (Ι)-

(Π) χ ' ο ι 2 4 5 χ

Άρα 2 s l-2x + 61 s 4 <:::> χ Ε [Ι , 2] υ [4, 5]

Άσκηση 7ΙJ Να απλοποιηθεί η παράσταση :

Α= 3 1 χ - 5 1 -2 1 3 - χ I και να λυθεί η εξίσωση : Α=4 και η aνίσωση

Α < 4

Λύση Για τη λύση μιας εξίσωσης ή μιας ανίσωσης, στην οποία οι παραστάσεις που βρίσκονται μέσα στα απόλυτα είναι διαφορετικές, πρέπει να κάνουμε πίνακα προσήμων αυτών των παραστάσεων να λύσουμε την εξίσωση ή την ανίσωση στα επιμέ­ρους διαστήματα του πίνακα και να εξετάσουμε αν οι τιμές που βρήκαμε ανήκουν στα αντίστοιχα δια­στήματα. Όπως και στην άσκηση 5 βρίσκουμε:

Οπότε:

{-χ + 9 αν x s 3 Α = -5x + 2 l αν 3 s x s 5

χ - 9 αν χ � 5

α) Στο ( -οο,3] έχουμε: Α = 4 <=> -χ + 9 = 4 <:::> χ = 5 η τιμή αυτή απορρί­πτεται διότι δεν ανήκει στο (-οο,3 ] . β) Στο [3 , 5] έχουμε:

Α = 4 <=> -5χ + 2 1 = 4 <=> χ = .!2 5 Δεκτή τιμή αφού ανήκει στο [3 , 5] . γ) Στο [5 , +οο) έχουμε:

Α = 4 <:::> χ - 9 = 4 <:::> χ = 1 3 Δεκτή τιμή αφού ανήκει στο [5 , +οο).

στο (-οο,3 ] . β) Στο [3 , 5] έχουμε:

1 7 Α < 4 <:::> -5χ + 2 1 < 4 <:::> -5χ < -1 7 <:::> χ > -

<:::> Χ Ε ( ·; , 5] γ) Στο [5 , +οο) έχουμε:

Α < 4 <=> χ - 9 < 4 <=> χ < 1 3 <=> χ Ε [ 5 , 1 3 )

- οο 3 1 7/5 5 1 3 Τελικά Α < 4 <=> χ Ε ( ι; , 5 J υ [ 5, 1 3 ) <=> χ Ε ( ι; , 1 3) ΡΙ ΖΕΣ Π ΡΑΓΜΑτ t ΚΩΝ ΑΡ Ι ΘΜΩΝ

5

Για κάθε α 2 Ο και ν Ε Ν * , αποδεικνύεται ότι υπάρχει ένα μόνο χ 2 Ο τέτοιο ώστε χν=α και συμβολίζεται με χ = � , δηλαδή :

Χ = �� . (Χ ν = α] ορσ χ 2 0

Άμεση συνέπεια αυτού είναι η ισότητα:

Άσκηση 8'1 Να χαρακτηρίσετε με (Σ), Σωστό ή (Λ) Λάθος

τις παρακάτω προτάσεις και να αιτιολογήσετε

την απάντησή σας.

ί) .J25 = -5

ίί) �( -2)2 = -2

ίίί) Για κάθε α ε JR ισχύει Ν = α ,

iv) Για κάθε α � Ο ισχύει ( .rαγ = α

ν) Υπάρχει α Ε ffi. με if;;! = � vi) Για κάθε α � Ο ισχύει � = α ,

vii) Για κάθε α, β Ε ffi. ισχύει:

ΕΥΚΛΕΙΔΗΣ Β' τ. t /30

Page 33: Ευκλειδης Β 61

Μαθηματικά για την Α · Λυκείου

J;! = Jii <::> α = β , viii) Για κάθε α, β Ε IR , με α β � Ο ισχύει:

Jεψ = vfai .Jιii ix) Για κάθε α � Ο ισχύει .J4α * -2-.Γα χ) Για κάθε α < Ο, β > Ο ισχύει

α1β = -�-α3β xi) Υπάρχουν τουλάχιστον δύο τιμές του

α Ε IR ώστε να ισχύει αifi = -� .

Λύση i) Λάθος, διότι η τετραγωνική ρίζα ενός θετι­

κού αριθμού α, είναι η μη αρνητική λύση της εξίσωσης χ2=α. Άρα J25 = +5

ii) Λάθος, διότι για το λόγο που προαναφέραμε ισχύει: �( -2 )2 = 1-21 = +2

iii) Λάθος, π.χ. αν α = -3 τότε �( -3 )2 = .J9 = 3 * -3 . Γενικά για κάθε α Ε JR ισχύει: Ν = Μ = lαl

ίν) Σωστό, διότι α � Ο ν) Σωστό, π. χ. για α = 5, αλλά και για κάθε

α � Ο. νί) Σωστό, διότι α5�0<=> α�Ο. νίί) Λάθος, διότι αν π.χ. α=3 και β= -3 τότε η

J3i = �( -3 )2 είναι αληθής ενώ η 3 = -3 εί-ναι ψευδής. Γενικά Ν = # <=> Ια l = Ιβ l <=> α = ±β

νίίί) Σωστό, διότι � = νfαβί = νraϊίί3ί = νfαί . ν1βί

ix) Λάθος, αφού για α=Ο είναι ίσα.

ισχύει:

χ) Σωστό, αφού o@=-{-α)<ffi=�=�, καθόσον -α > Ο .

xi) Λάθος διότι μόνο για α=Ο ισχύει.

Άσκηση 9η Ποιες από τις παρακάτω προτάσεις είναι σω­στές και γιατί; i) Για κάθε α Ε [ 2, + οο) και β Ε [ -5, + οο) αν

ισχύει Vα - 2 + �β + 5 = 0 (Ι) τότε θα ι­

σχύει: 1) α = 2 ή β = -5 2) α = 2 και β = -5

3) α= -2 και β = 5 ii) Η ισότητα �α2 + β2 = α + β , με α,β � Ο ισχύ­

ει: 1) πάντοτε 2) μόνο όταν α = Ο και β = Ο 3) μόνο όταν α = Ο ή β = Ο

Λύση i) Ι και 2, διότι (1) => α = 2 και β = -5 =>

=:> α = 2 ή β = -5 ii) 3 , διότι �α2 + β2 = α + β <=> (�α2 + β2 )2 = (α + β)2 <=> α2 + β2 = α2 + 2αβ + β2

<=> 2αβ = Ο <=> α = Ο ή β=Ο.

Άσκηση 1 0'1 Να λύσετε τις εξισώσεις:

i) � = .J3 - 4x ii) .,/9 χ '

- 6 χ + 1 + 7 χ - 2 = ο

Λύση i) Για να ορίζεται η εξίσωση πρέπει και αρκεί:

3χ - 2 � ο} 2 3 , δηλαδή - � χ � - . 3 - 4χ � Ο 3 4 Τότε έχουμε:

.J3x - 2 =.J3 - 4x <=>{.J3x - 2 )2 ={.J3 -4x )2 <=> 5 <=> 3χ - 2 = 3 - 4χ <=> 7χ = 5 <=> χ = - . 7

Η ' 5 ' δ ' δ ' 2 5 3 τιμ η χ = - ειναι εκτη ιοτι - � - � - . 7 3 7 4 , 5 2 Ι 5 3 Ι (αφου - - - = - > 0 και - -- = -- < 0 ) 7 3 2 Ι 7 4 28

ii) Για να ορίζεται η εξίσωση πρέπει και αρκεί 9χ2 - 6χ + Ι � Ο πράγμα που ισχύει για κάθε χ Ε JR , αφού 9χ2 - 6χ + Ι = (3χ - Ι )2 � Ο. •

Άρα .J9x2 - 6χ + Ι + 7χ - 2 = 0 <=> �(3χ - Ι )2 + 7χ - 2 = 0 <=> l3x - I I + 7x - 2 = 0

α) Αν χ Ε ( -οο, j] τότε (I) <=> ( -3χ + Ι ) + 7χ - 2 = Ο <=> χ = .!. δεκτή 4

' δ ' Ι ( 1 ] τιμη ιοτι '4 Ε -οο, '3

ΕΥΚΛΕΙΔΗΣ Β' τ.1/31

Page 34: Ευκλειδης Β 61

Μαθηματικά για την Α· Λυκείου

β) Α ν χ ε [ � , + οο) τότε (I) <::::> ( 3 χ - Ι ) + 7 χ - 2 = Ο <::::> χ = 2_ Η τιμή 1 0

. . δ . 3 [ Ι ) αυτη απορριπτεται ιοτι

-� - , + οο 1 0 3

Τελικά .J9x2 - 6χ + Ι + 7χ - 2 = Ο <::::> χ = ..!_ 4

λ(FΚψ:Jη 1 I ' 1

Αν Ι χ Ι <3 τότε να απλοποιηθεί η παράσταση :

.J4x2 - 24χ + 36 .Jx2 + 6χ + 9 Α = + ----

/ �·. ��; ιυ Ί�-�

χ - 3 χ + 3

Α �..--4(-χ --3-)2 + �( χ + 3)2 = 2 lx - 3l )χ + 3ι χ -3 χ + 3 χ - 3 χ + 3

= -2 (χ - 3) + χ + 3 = -2 + Ι = - Ι , επειδή χ - 3 χ + 3 lxl < 3�-3< x < 3}� χ -3 < 0}

χ+ 3 > 0

Άσκη(ιη 1 2 '� Ν' απλοποιηθούν οι παραστάσεις:

Α = ��α3�α 2Ν , Β = �, Γ = {Γσ!θ Δ = �13 - 4J3

ΛiΗ'Η! Εφαρμόζοντας τις ιδιότητες α4β = � , ffα = μ� και μ� = � με α, β :::: Ο έχουμε: Για να ορίζονται οι Α, Β πρέπει και αρκεί α ::=:: Ο . Τότε

....------:--=

Α = �α3 �.Jα4α2 = �α3� = Wa = � = -Γα Β = �α5·4+3 = � · W = α4W . Η παράσταση Γ όμως ορίζεται για κάθε α ε R οπότε Γ = νfar = �ιαι 7 ·S+) = Ια15 • fαr ' αφού πι­θανόν να είναι α<Ο. Χρειάζεται λοιπόν προσοχή όταν ο εκθέτης του υπόριζου είναι άρτιος. Δ = �Ι2 + ( 2J3)2 - 2 · Ι · 2J3 = �( Ι - 2J3)2 =

= Ι Ι - 2J3 1 = 2J3 - Ι , αφού 2J3 > Ι .

Λσκηση 1 3 '� Να μετατρέψετε τα παρακάτω κλάσματα σε ι­

σοδύναμα κλάσματα με ρητό παρονομαστή. 2 αβ

α) -

, β) , co ' αβ > Ο � ναβ ι α - 3 γ) , r---:; , α :;e Ο δ) ι-;; , α > 3

�α2 να - 3 ι 2J3 + ιο

ε) � ' α > Ο ζ) 2J3 - ιο

Λίιση α) Αν ο παρονομαστής είναι της μορφής W με α>Ο, ν>κ, τότε πολ/με τους όρους του κλάσματος με την παράσταση v α ν-κ έτσι ώστε ο παρονομα-στής να γίνει W = α , δηλαδή

2 2ifiί 2 if24 2 if24 7 - = = - = - = J.fϊ6 ifi3 ifi3 if24 if2ί 2

αβ αβ�α2β2 αβ�α2β2 αβ�α2β2 β) - = = = � ��α2β2 ψαβγ αβ

= �α2β2

Ι I{[af {[af {[af γ) w = Μ \fσf = {[af = �α�

α - 3 ( α - 3)νΓα"=3 _ ( α - 3)νΓα"=3 _ δ) .Jα- 3 = .Jα- 3-Jα- 3

-�( α- 3)2

-

= (α - 3)-Γα="3 = �α - 3 α - 3 1 1 � -

�-�

ε) � = α2� = α2��

-α2if;!

-αJ

ή

. Ι {Γα! {Γα! συντομοτερα 1 Γ7 = 1 ΓQ = -3 να7 να9 α ζ) Όταν ο παρονομαστής είναι της μορφής � ± .Jβ , πολ!ζουμε και τους δυο όρους του κλάσματος με τη συζυγή παράσταση δηλαδή -Γα + Jβ , έτσι ώστε να δημιουργηθεί η ταυτότητα: (α + β)( α - β) = α2- β2 Άρα:

ΕΥΚΛΕΙΔΗΣ Β' τ.Ι /32

Page 35: Ευκλειδης Β 61

Μαθηματικά για την Α ' Λυκείου

2ν'3 + 10 ( 2ν'3 + 10) ( 2ν'3 + 10) (2ν'3 + 10)2 =

2ν'3 - 10 ( 2ν'3 - 10) ( 2ν'3 + 10) ( 2ν'3)2 - 102

= (2../3 )2 + 2 · 2../3 · 10 + 102 12 + 40../3 + 100 = 12 - 100 88 1 1 2 + 40.J3 1 4 + s.J3

=

88 1 1

Άσκηση 1 4 '1 (Λύση εξισώσεων της μορφής χν=α)

Να επιλύσετε τις εξισώσεις:

i) 2χ6=32χ2 ii) χ4=-27χ iii) χ4+256=0

ίν) (x+l)3= -64 ν) (2χ+4)4=81 vi) χ4=α6

vii) χ3=λ-2

ΛίJση Έχουμε μάθει στα προηγούμενα μαθήματα ότι για να λύσουμε μια εξίσωση πρέπει να δούμε τι βαθ­μού είναι. Εάν είναι πρωτοβάθμια χωρίζουμε γνω­στούς και αγνώστους, Εάν όμως είναι από 2ου βαθ­μού και πάνω εξετάζουμε αν είναι στη μορφή χν=α ( 1 ) . Εδώ υπάρχουν οι εξής περιπτώσεις: i) Αν α=Ο τότε ( 1 ) <=:>χ=Ο ii) Α ν α>Ο και ν άρτιος τότε ( 1 ) <=:> χ = ±ifO.

(2 λύσεις αντίθετες) iii) Α ν α>Ο και ν περιττός τότε ( 1 ) <=> χ = ifO.

( 1 λύση θετική) ίν) Αν α<Ο και ν άρτιος τότε είναι αδύνατη ν) Α ν α<Ο και ν περιττός τότε

( 1 ) <=> χ = -� ( 1 λύση αρνητική) Στην περίπτωση όμως που δεν είναι έτοιμη η εξί­σωση στη μορφή χ ν = α, τότε φέρνουμε όλους τους όρους στο α ' μέλος έτσι ώστε το β ' μέλος να είναι μηδέν, παραγοντοποιούμε και εφαρμόζουμε την αβ = Ο <=> α = Ο ή β = Ο. Άρα: i) 2χ6 = 32χ2 <=> 2χ6 - 32χ2 = ο <=> <=:> 2χ2 (χ4 - 1 6) = 0 <=:> χ2 = 0 ή χ4 - 1 6 = 0 <=:> χ = Ο ή χ4 = 1 6 <=:> χ = 0 ή χ = � ή χ = -� <=> χ Ε {Ο, - 2, 2} ίί) Χ4 = -27χ <=> χ4 + 27χ = ο <=> χ (χ3 + 27 ) = 0 <=:> χ = 0 ή Χ3 = -27 <=:> χ = ο ή χ = -ifii <=> χ = ο ή χ = -3 .

iii) χ4 + 256 = Ο <=> χ4 = -256 αδύνατη . ίν) {χ + 1 )3 = -64 <=:> χ + 1 = --V'64

<=> χ + 1 = -4 <=> χ = -5 ν) {2χ + 4)4 = 8 1 <=:> 2χ + 4 = 3 ή 2χ + 4 = -3

<=> 2χ = -1 ή 2χ = -7 <=> χ = _.!. ή χ = _7._ 2 2 νί) χ4 = α6 <=> χ = ±W <=:> χ = ±Μ <=:>

x = lα iM ή χ = - Ια ΙΜ · νίί) αν λ - 2 � Ο , δηλαδή, λ � 2 , τότε

χ 3 = λ - 2 <=> χ = �λ - 2 αν λ - 2 < Ο , δηλαδή, λ < 2 , τότε χ 3 = λ - 2 <=> χ = -� 2 - λ

Άσκηση ! 5'1 Για κάθε ν ε Ν με ν � 2 να δείξετε ότι: ifi + ifi > ifS

Λ\Jση ο < χ < 1 � χ κ < 1 � χ κ+μ < χμ ' για κάθε κ ' μ Ε Ν *

Άρα ω � ω οπότε 'm � 'm δηλα­δή Η � Jf . Ομοίως JI � Ji Άρα: ν {Ι + vfi � {Ι + fi Αλλά � + {i > 1 \15 \15 vs vs vs vs , αφού (.J2+.J3)2 =5+2J6 >5�

� fi +ν'3 > J5 � Η +)Ι > ι

Οπότε: Jf + Ji > Ι, δηλαδή ifi + if3 > if5

Π α ρατή ρη ση : Αν θεωρηθεί γνωστό το ανάπτυγμα του διωνύμου του Νεύτωνα, τότε η παραπάνω aνί­σωση αποδεικνύεται ευκολότερα ως εξής: Γνωρίζουμε ότι για κάθε α,β>Ο έχουμε:

Αν α = ifi και β = if3 τότε: ( 42 +43)" >42-ν +43ν = 2 + 3 = 5�42 +43 > :15

ΕΥΚΛΕΙΔΗΣ Β ' τ.l /33

Page 36: Ευκλειδης Β 61

Μαθηματικά για την Α ' Λυκείου

Άσκηση 1 6'1 Α ν x,y θετικοί αριθμοί να αποδείξετε ότι:

χ + y � �χ2 + y 2 + 2FY - JhY (I) και να βρεί­

τε πότε ισχύει το ίσον

Λύση Υψώνουμε και τα δυο μέλη στο τετράγωνο, εφό­σον είναι μη αρνητικά και προσπαθούμε να την αναγάγουμε σε μια σχέση που ισχύει: (I) � χ + Υ + JbY � �χ 2 + y2 + 2.,ΓχΥ � [ (χ + Υ) + .fbYT � [�χ 2 + y2 + 2.,ΓχΥ τ �

( χ + Υ )2 + 2 ( χ + y )J2xy + 2xy � χ 2 + y2 + +4�χ2 + y2 FY + 4xy � J2 (χ + Υ) � 2J,...-x 2_+_y_2 � 2χ2 + 2y2 + 4xy � 4χ2 + 4y2 � �2xy � x2 + y2 � χ2 +I -2xy2:: 0�(x-y)2 2:: 0 που ισχύει για κάθε x,y>O. Η ισότητα ισχύει μόνο όταν (χ - ψ)2 = Ο, δηλαδή χ = ψ

Άσκηση 1 7'1 Θεωρούμε το κλάσμα Α = 2χ2 - 3χ + l

.J2x - χ2 - .Jι - χ2

α) Για ποιες τιμές του χ ορίζεται;

β) Να λυθεί η εξίσωση Α= 1 - χ

Λύση α) Για να ορίζεται η Α πρέπει και αρκεί: 2χ - χ2 2:: 0 ( 1 ) , l - x2 2:: 0 (2) , .J2x - x2 - � :;t O (3) ( Ι ) � χ2 - 2χ � ο � χ ( χ - 2) � ο � (: � � � ο) ή (: � � 2:: ο)� χ Ε [0, 2] .

Ομοίως ( 2) � χ Ε [ - 1 , Ι ] . Οι ( 1 ), (2) συναληθεύ­ουν στο [0, 1 ] , οπότε: (3)�2χ-χ2 :;t } - x2 �x:;t��xE [ Ο, �)υΘ, ι]

(2χ-Ι) (χ-ι)(� +Μ) β) A=l-x� -{ ) =1-χ� 2x-i 1-i

( x - l ) (.J2x - x 2 + �) + (χ - 1 ) = 0 �

(x - I ) (.J2x - x2 + � + l ) = o �

χ = Ι , αφού .J2x-x2 + � + Ι > Ο

Πρόβλημα

Δίνεται ορθογώνιο τρίγωνο ΑΒΓ (Α = 90° ) Αν

τα μήκη των πλευρών του ΑΒ και ΒΓ είναι ίσα

με κάποιους απ' τους αριθμούς - I α2 -2 1 , α2+2,

I α+2 1 - l -2-α I , α2-2, να βρείτε

α) τα μήκη των πλευρών του

β) το εμβαδόν του τριγώνου

Λύση Α) Τα μήκη των πλευρών του είναι θετικοί αριθ­μοί. Αφού - l α2-2 I �O και l α+2 1 - l -2-α l = = Ι α + 2 1 - 1 α+ 2 1 =Ο για κάθε α Ε 1R , αποκλείεται αυτοί να είναι μήκη πλευρών

Γ

Β

Παρατηρούμε εξάλλου ότι: α2 - 2 � Ο � α2 � 2 � Ια l2 � 2 � Ια Ι � J2 � -J2 � α � J2 Αν λοιπόν α ε [ -.J2 , .J2] τότε και ο αριθμός α2-2 αποκλείεται να είναι μήκος πλευράς Α ν όμως α2 - 2 > Ο δηλαδή α > J2 ή α < -J2 . Τότε (ΑΒ ) = α2 - 2, (ΒΓ ) = α2 + 2 αφού ΒΓ> ΑΒ. Εφαρμόζουμε το Πυθαγόρειο θεώρημα και έχου­με:

(ΑΓ)2 = (ΒΓ)2 - (ΑΒ)2 = ( α2 + 2/ - ( α2 - 2/ = = ( α2 + 2 + α2 - 2 ) ( α2 + 2 - α2 + 2 ) = 2α2 · 4 = 8α2

Άρα ( ΑΓ) = J&1 = J8.N = 2J2 Ια l > Ο αφού Ια Ι > J2 .

β · υ (ΑΒ)(ΑΓ) ( α2 - 2) · 2J2 iα l β) Ε = - = = ....:....________:_ __ 2 2 2 = J2Ια l (α2 - 2) .

ΕΥΚΛΕΙΔΗΣ Β ' τ.t/34

Page 37: Ευκλειδης Β 61

Μαθηματικά για την Α' Λυκείου

Άσκηση 1 8'1 Να λυθεί η εξίσωση : 2( χ2 + 2χ)2 + 2 1χ2 + χ- 2 1 + 5Jx3 - 6χ-4 = 0 (1)

Λύση Για να ορίζεται η ( Ι ) πρέπει και αρκεί χ3 - 6χ - 4 :=:: Ο (2). Τότε έχουμε: 3 ( χ2 + 2χ )2 + 2 Ιχ2 + χ - 2 1 + 5.J'"x.,--3 --6χ---4 = ο <=:>

χ2 + 2χ = Ο (3) } {χ2 + 2χ )2 � 0 χ: + χ - 2 =

_ο (4) , αφού Ιχ2 + χ - 2 1 � ο

χ - 6χ - 4 - Ο (5) .Jx3 _ 6χ _ 4 � ο (3 ) <=> χ (χ + 2 ) = 0 <=:> χ = 0 ή χ = -2 Παρατηρούμε ότι οι (4) και (5) επαληθεύονται μό­νο για χ= -2, η οποία είναι δεκτή, διότι για χ= -2 έχουμε χ3 -6χ--4 = 02:0, δηλαδή ισχύει η (2). Άρα ( Ι ) <=:> χ=-2

Άσκηση 1 9'1 Ν α λυθεί η εξίσωση :

.J2x - x2 - .Jx2 - 4 = 3x - λ (1),

όπου λ παράμετρος

Λύση Για να ορίζεται η ( Ι ) πρέπει και αρκεί 2χ - χ2 � ο (2) χ2 - 4 � 0 (3) Έχουμε: ( 2 ) <=:> χ (χ - 2 ) � Ο <=:> χ Ε [Ο, 2] . (3 ) <:::> xz � 4 <=> lx lz � 4 <:::>

<=:> lx l � 2 <=:> χ Ε ( -οο,-2] υ [2, +οο ) -2 ο 2 - οο

1 1 ! χ ' (3) (2)

+οο

(3) χ:

Επομένως η εξίσωση ( Ι ) ορίζεται μόνο για χ=2. Για να επαληθεύεται για χ=2 πρέπει και αρκεί JO - JO = 6 - λ, δηλαδή λ = 6 Άρα όταν λ :j:. 6 είναι αδύνατη, ενώ όταν λ = 6 έχει μοναδική ρίζα την χ = 2.

Γεωμετρία Ευθύγραμμα Τμήματα

Άσκηση 1 '1 Σε μια ευθεία (ε) παίρνουμε διαδοχικά σημεία Α, Β,Γ, Δ ώστε: ΑΒ=α, ΒΓ=β, ΓΔ=γ. i) Να βρείτε σημείο Κ εσωτερικό του ΑΒ ώ­

στε: ΔΚ=δ. Ποια συνθήκη πρέπει να ικανο­ποιείται ώστε να έχει λύση το πρόβλημα;

ii) Όμοια να βρείτε σημεί Λ εσωτερικό του Γ Δ ώστε: ΑΛ=δ. Ποια συνθήκη πρέπει να ικα­νοποιείται ώστε να έχει λύση το πρόβλημα;

iii) Βρείτε τη συνθήκη πρέπει να ικανοποιείται ώστε να ισχύουν οι ερωτήσεις Ι & 11. Στη συνέχεια υπολογίστε το μήκος του τμήματος ΚΛ. (α,β,γ,δ γνωστά μήκη)

Λύση Ε Α κ Β Γ Λ Δ

Σαμπάς Θεόδωρος

ί) ΑΚ=ΑΔ-ΔΚ=α+β+γ-δ ( I ) Για να έχει λύση, πρέπει να ισχύει:

ΒΔ<ΔΚ <ΑΔ <=:> β+γ<δ<α+β+γ (2) ίί) ΔΛ=ΑΔ-ΑΛ=α+β+γ-δ (3) Για να έχει λύση,

πρέπει να ισχύει: ΑΓ <ΑΛ <ΑΔ <=:> β+γ<δ<α+β+γ ( 4)

iii) Θα πρέπει να ισχύουν ταυτόχρονα οι συνθήκες (3) & (4) <=:> maχ {β+γ,α+β}<δ<α+β+γ ΚΛ=ΑΔ - ΑΚ - ΛΔ = = α+β+γ - (α+β+γ - δ) - (α + β + γ - δ) ΚΛ = α+β+γ-α-β-γ+δ-α-β-γ+δ= 2δ-(α+β+γ) (5)

Η (5) έχει νόημα γιατί: δ > β + γ}

=> 2δ > α + β + γ + β > α + β + γ δ > α + β

ΕΥΚΛΕΙΔΗΣ Β' τ. Ι/35

Page 38: Ευκλειδης Β 61

Μαθηματικά για την Α· Λυκείου

ί\σκηση 2 1 1 Επί ευθείας χ 'χ, δίνονται τέσσερα διαφορετικά

σημεία Α,Β,Γ,Δ με οποιαδήποτε σειρά.

Θεωρούμε τις προτάσεις:

p: «τα ευθύγραμμα τμήματα ΑΒ, Γ Δ έχουν κοι­

νό μέσο>>.

q: «ΑΓ=ΒΔ>>

i) Δείξτε τη συνεπαγωγή p=> q ii) Ισχύει η συνεπαγωγή q=> p; iii) Πώς πρέπει να διαμορφωθεί η υπόθεση,

ώστε να ισχύει η ισοδυναμία: p<::>q

/\1:Jση Γ - Α Ο Β Δ

,__�- --Ι---------Ι---------.----1---τ--χ· χ

ί) Έστω τα ευθύγραμμα τμήματα ΑΒ, Γ Δ & Ο το κοινό μέσον τους αν ΓεΑχ ' τότε: ΟΓ>ΟΒ άρα ΔεΒχ Για το σχήμα (Σ) : Γ Α=ΟΓ -ΟΑ=ΟΔ-ΟΒ=ΒΔ όμοια, όταν ΔεΒχ Αν Γ είναι εσωτερικό σημείο του ΑΒ, τότε: ΟΓ<ΟΑ &ΟΔ<ΟΒ. Έχουμε το σχήμα (Σ ')

Α Γ Ο Δ Β -4--l------t------------1--1-x ' χ

Έτσι: ΑΓ=ΟΑ - ΟΓ=ΟΒ - ΟΔ=ΒΔ Σε κάθε περίπτωση ισχύει η συνεπαγωγή p=>q ίί) Στο σχήμα Σ", έχουμε ΑΓ=ΒΔ, όμως τα ευθύ­

γραμμα τμήμτα ΑΒ & Γ Δ δεν έχουν κοινό μέ-σο.

Ο Δ Β Γ Α ----ι-----1 ----e ---1-----1---χ' χ

!Ί ρ ιiγματ ι : αν Ο είναι κοινό μέσο των ΑΒ & ΓΔ τότε: ΟΓ>ΟΑ=> ΟΔ>ΟΒ (άτοπο) Άρα δεν ισχύει η συνεπαγωγή q=> p

ίίί) Η υπόθεση πρέπει να διαμορφωθεί ως εξής: «Δίνονται τα σημεία Α,Β,Γ,Δ της ευθείας χ 'χ ώστε Γ, ΔεΑΒ ή Γ,Δίι!ΑΒ, τότε ισχύει η ισο­δυναμία p<::>φ>. Π ρfηματ ι : Το πρόβλημα υπάρχει με τη συνε­παγωγή q=>p. Έχουμε τις περιπτώσεις, των σχημάτων: ΑΓ=ΔΒ & ΟΓ=ΟΔ (Ο μέσο του ΓΔ)

Α Γ ο Δ Β χ ' -�·�----�---+�

χ Άρα ΑΓ+ΟΓ=ΔΒ+ΟΔ=>ΟΑ=ΟΒ, δηλαδή Ο μέσο του ΑΒ. Όμοια και για το σχήμα (Σ2)

Α ο Β

χ' Γ

ΑΓ=ΒΔ & ΟΑ=ΟΒ (Ο μέσο του ΑΒ)

Δ χ

Άρα ΑΓ+ΟΑ+ΟΒ+ΒΔ => ΟΓ=ΟΔ, δηλαδή Ο μέσο του ΓΔ Έτσι, ισχύει η συνεπαγωγή q=> p

Λσκηση 3 '1 Δίνεται ευθεία χ 'χ, στην οποία παίρνουμε τα

σημεία Α, Β 'ώστε ΑΒ=α (α γνωστό μήκος)

Να βρείτε τα σημεία Μ, Ν της ευθείας χ 'χ, ώ­

στε:

ΜΑ-ΜΒ= ΝΑ-ΝΒ

ΛίJση Επειδή ΜΑ-ΜΒ & ΝΑ-ΝΒ?::Ο , δηλαδή ΜΑ?::ΜΒ & ΝΑ?::ΝΒ ( 1 ) Έχουμε:

χ ' χ Α ο Β

Τα Μ, Ν θα ευρίσκονται στην ημιευθεία Οχ ' (Ο το μέσο του ΑΒ), γιατί μόνο τότε ισχύουν οι συνθή­κες ( 1 ) . Αν το Μ συμπίπτει με το Ο, τότε: ΜΑ-ΜΒ & ΝΑ-ΝΒ=Ο. Επομένως και το Ν συμπίπτει με το Ο, συνεπώς τα Μ, Ν συμπίπτουν με το Ο. Αν το Μ ανήκει στην ημιευθεία Βχ ' , τότε: ΜΑ-ΜΒ =ΑΒ <=> ΝΑ-ΝΒ=ΑΒ, άρα και το Ν α­νήκει στην ημιευθεία Βχ ' . Είναι προφανές ότι τα Μ, Ν δεν συμπίπτουν υποχρεωτικά, έχουμε απει­ρία ζευγών σημείων Μ, Ν (κάθε ζεύγος σημείων της ημιευθείας Β χ ') . Απομένει, τα Μ, Ν να είναι εσωτερικά σημεία της ΟΒ. Τότε έχουμε:

ΕΥΚΛΕΙΔΗΣ Β' τ.l/36

Page 39: Ευκλειδης Β 61

Μαθηματικά για την Α ' Λυκείου

ΜΑ-ΜΒ = ΝΑ-ΝΒ <::::> (ΟΑ+ΟΜ) -(ΟΒ-ΟΜ)=(ΟΑ+ΟΝ) <::::> ΟΜ=ΟΝ Άρα τα σημεία Μ, Ν συμπίπτουν.

Άσκη ση 4 '� Δίνεται ευθεία χ 'χ, στην οποία παίρνουμε τα σημεία Α, Β 'ώστε ΑΒ=α (α γνωστό μήκος) Να βρείτε τα σημεία Μ, Ν της ευθείας χ 'χ, ώ­στε:

χ ' χ <� --+----.----+---Α 0 Β

Γl ρ{:πε ι : ΜΑ2:ΜΒ & ΝΑ2:ΝΒ ( 1 ), για να έχει νόη­μα η σχέση που δίνεται προς απόδειξη. Τα σημεία Μ, Ν ανήκουν στην ημιευθεία Οχ '(Ο το μέσο του ΑΒ), τότε μόνο ισχύουν οι σχέσεις ( 1 ) . i) Αν το Μ συμπίπτει με το Ο, τότε και το Ν συ­

μπίπτει με το Ο. Γιατί: ΜΑ2-ΜΒ2=0<::::> ΝΑ2-ΝΒ2=0

ii) Λν το \I είνω �;σωτr.ρ ι κό του Ο Β , τότε: ΜΑ -ΜΒ < ΑΒ} � ΜΑ2 - ΜΒ2 < ΑΒ2 ΜΑ + ΜΒ = ΑΒ (2)

και το Ν είναι εσωτερικό του ΟΒ, σε άλλη πε­ρίπτωση (δηλαδή Ν Ε Β χ '), έχουμε: ΝΑ - ΝΒ = ΑΒ} � ΝΑ2 - ΝΒ2 � ΑΒ2 ΝΑ + ΝΒ � ΑΒ (3)

από (2) & (3) καταλήγω σε άτοπο (ΑΒ2<ΑΒ2) Έτσι: (ΜΑ-ΜΒ)(ΜΑ+ΜΒ)= (ΝΑ - ΝΒ)(ΝΑ + ΝΒ) � (ΜΑ - MB).}df = (ΝΑ - ΝΒ) ΑΒ� ΜΑ - ΜΒ=ΝΑ - ΝΒ(ΟΑ+ΟΜ)--(ΟΒ - ΟΜ)= (ΟΑ+ΟΝ)-(ΟΒ-ΟΝ) � ΟΜ=ΟΝ άρα τα Μ, Ν συμπίπτουν.

ii i) Λ ν το Μ Ε Β χ · , τ6τ:; κ α ι το :\! Ε Β χ ' ( σύμφω­να και με την (11)) Έτσι: (ΜΑ-ΜΒ)(ΜΑ+ΜΒ)= (ΝΑ-ΝΒ)(ΝΑ+ΝΒ) � }df (ΜΑ+ΜΒ) = }df (ΝΑ+ΝΒ) �

ΜΑ+ΜΒ=ΝΑ+ΝΒ� fJ1(. +ΟΜ+ΟΜ-)dB = ΟΑ+ΟΝ+ ΟΝ -)dB �ΟΜ=ΟΝ� Μ, Ν συ­μπίπτουν

Ι η Λυκείου (τΟ Ξ Α )

Άσκηση 5 η Δίνεται κύκλος (O,R) και τα διαδοχικά σημεία Α, Β, Γ, Δ ώστε: -- -- --

ΑΒ + ΒΓ + Γ Δ < 360° Αν είναι α · ΑΒ = β · ΒΓ (α, β γνωστοί θετικοί), τότε: ---ί) Ν α εκφράσετε το ΒΔ σε σχέση με τα ΑΔ

και ΓΔ ίί) Αν είναι ΑΟΓ = 90° , τότε να βρείτε τα τόξα --- --

ΑΒ και ΒΓ ίίί) Α ν τα σημεία Α, Γ είναι aντιδιαμετρικά να --- --

βρείτε τα τόξα ΑΒ και ΒΓ

Λύση

i) ΒΔ = ΑΔ - ΑΒ ΒΔ = ΒΓ + Γ Δ στην συνέχεια: ---- -- ---

α · ΒΔ = α · ΑΔ - α · ΑΒ --

+ β · ΒΔ = β · ΒΓ + β · Γ Δ ( α + β)Μ = α · Μ + β · fΔ

---Γιατί: β · ΒΓ = α · ΑΒ

--τ λ , Β..---Δ α · ΑΔ + β · ΓΔ ε ικα: = -----'---

α + β i i) ΑΟΓ = 90° � ΑΓ = 90° . Έχουμε:

( 1 )

AB + Br = 90° } ..--- __ επιλύω το σύστημα και έχω:

α · ΑΒ = β · ΒΓ

ΕΥΚΛΕΙΔΗΣ Β ' τ.Ι/37

Page 40: Ευκλειδης Β 61

Μαθηματικά για την Α ' Λυκείου

� α -- β ΒΓ = --90° και ΑΒ = --90° α + β α + β

� ίίί) Α, Γ aντιδιαμετρικά � ΑΓ = 1 80° . Έχουμε: ΑΒ + ΒΓ = 1 80° -- � }

__ � επιλύω το σύστημα και έχω: α · ΑΒ = β · ΒΓ

Βr = -α-1 80° και ΑΒ = -β-1 80° α + β α + β

Ά σκη ση 6'1 Μια ευθεία γωνία διαιρείται σε διαφορετικές

γωνίες με ημιευθείες από την κορυφή της.

i) Α ν οι γωνίες είναι τρεις, τότε: «μια τουλάχι­

στον είναι μικρότερη των 60° και μια του­

λάχιστον είναι μεγαλύτερη των 60°»

ii) Α ν οι γωνίες είναι τέσσερις, τότε: «μια του­

λάχιστον είναι μικρότερη των 45° και μια

τουλάχιστον είναι μεγαλύτερη των 45°»

iii) Αν οι γωνίες είναι ν (ν�5) τότε: «μια τουλά­

χιστον είναι μικρότερη των 180°/ν και μια

τουλάχιστον είναι μεγαλύτερη των 180° /ν»

Λ1J ση

ί) Θα εργαστώ με απαγωγή σε άτοπο, έστω: α1 � 60° } α2 � 60° � α1 + α2 + α3 > 1 80° (άτοπο, η ισότητα α3 � 60° δεν ισχύει από την υπόθεση)

α1 � 60° } α2 � 60° � α1 + α2 + α3 < 1 80° (άτοπο, α3 � 60° όπως προηγούμενα) άρα ισχύει η πρόταση (ί)

Az

Αι

όμοια

ίί) Όπως και στην απάντηση (ί) έχουμε: αν ισχύει:

As

α1 � 45° α1 :s; 45o α2 � 45° ή α2 ::; 45° α3 � 45° α3 ::; 45° α4 � 45° α4 ::; 45°

Az

Αι

Σε κάθε περίπτωση άτοπο, η ισότητα ισχύει όταν α ι = αz = α3 = «4 = 45° (απορρίπτεται από την υπόθεση), άρα απομένει η πρόταση (ίί)

ίίί) (Η Γενίκευση), ν= ακέραιος θετικός �5 απορρίπτονται τα δυο ενδεχόμενα:

1 80° αι �-­ν 1 80° αz �-­ν

1 80° α >--v - ν

ή

1 80° αι :s;-­ν 1 80° αz :s;-­ν

1 80° α <--v - ν

Αι

Σε κάθε περίπτωση άτοπο, η ισότητα: 1 80° ( ' ' αι = αz = . . . = α ν=-- απορριπτεται απο ν

την υπόθεση), άρα απομένει η πρόταση (ίίί)

ΕΥΚΛΕΙΔΗΣ Β ' τ.l/38

Page 41: Ευκλειδης Β 61

Μαθηματικά για την Α· Λυκείου

-Ασκήσεις στην Ισότητα

και τις Ανισοτικές Σχέσεις Τριγώνων

Γιώργος Τριάντος

Φίλοι μαθητές Τα θέματα που αναπτύσσουμε στη συνέχεια, καλύπτουν το περιεχόμενο κυρίως του τρίτου κεφαλαίου της Γεωμετρίας. Κρίνεται απαραίτητη η γνώση :

• Των κριτηρίων ισότητας τριγώνων ( ορθογωνίων και μη ) • Των ιδιοτήτων της διχοτόμου γωνίας και της μεσοκαθέτου ευθυγράμμου τμήματος . • Των βασικών ανισοτικών σχέσεων μεταξύ των γωνιών και των πλευρών τριγώνου .

Για περισσότερα θέματα - στη συγκεκριμένη ύλη - μπορείτε να ανατρέξετε σε τεύχη παλαιοτέρων ετών του Ευκλείδη Β · , όπου υπάρχουν πολύ καλές επιμελημένες εργασίες άλλων συναδέλφων .

Άσκηση 1 . Σε κυρτό τετράπλευρο ΑΒΓ Δ είναι:

ΑΒ = ΒΓ και A = r . Να αποδείξετε ότι: ι. ΔΑ = ΔΓ 2. Η διαγώνιος ΒΔ είναι η μεσοκάθετη της δι­

αγωνίου ΑΓ.

Απόδειξη

Δ Ι . Επειδή το τρίγωνο ΑΒΓ είναι ισοσκελές με

κορυφή Β, οι γωνίες που πρόσκεινται στη βά-ση ΑΓ είναι ίσες, δηλ. ΒΑΓ = Br Α ( Ι )

Από την υπόθεση είναι: ΒΑΔ = Br Δ (2).

Απο τις σχέσεις ( Ι ),(2) προκύπτει ότι: ΒΑΔ - ΒΑΓ = Βf'Δ - ΒΓΑ δηλ. Γ ΑΔ = Ar Δ (3) . Από τη σχέση (3) συνάγε­ται ότι το τρίγωνο ΔΑΓ είναι ισοσκελές με κο­ρυφή το Δ και άρα ΔΑ=ΔΓ.

2 . Επειδή είναι ΒΑ=ΒΓ και ΔΑ=ΔΓ τα σημεία Β, Δ ανήκουν στη μεσοκάθετη της ΑΓ, οπότε

η ΒΔ ταυτίζεται με τη μεσοκάθετη της ΑΓ.

Άσκηση 2 Σε τρίγωνο ΑΒΓ είναι ΒΓ = 2 · ΑΒ , Β = 2 · Γ ,

Α

ΒΔ η διχοτόμος της γωνίας Β και Ε το μέσον της ΒΓ. Να αποδείξετε ότι: ι . Η ΒΔ είναι μεσοκάθετη της ΑΕ. 2. Το τρίγωνο ΑΒΓ είναι ορθογώνιο στο Α.

Απόδειξη Γ

Β

Ι . Επειδή το Ε είναι το μέσον της ΒΓ, έχουμε ΒΕ = ..!_ · ΒΓ = ..!._ · 2 · ΑΒ = ΑΒ οπότε το τρίγωνο 2 2 ΑΒΕ είναι ισοσκελές με κορυφή το Β με αποτέλε-σμα η διχοτόμος της γωνίας Β να είναι διάμεσος και ύψος του, δηλ. μεσοκάθετος της ΑΕ.

� Ι � � 2 . Επειδή Γ = -Β = ΔΒΓ , το τρίγωνο ΔΒΓ είναι 2 ισοσκελές με κορυφή το σημείο Δ και διάμεσο ΔΕ ,που είναι και ύψος του. Δηλ. ΔΕ j_ ΒΓ . Τα τρίγω­να ΑΒΔ, ΒΕΔ (από το κριτήριο Π-Γ -Π) είναι ίσα , αφού έχουν κοινή την πλευρά ΒΔ, ΒΑ=ΒΕ και

ΕΥΚΛΕΙΔΗΣ Β' τ. Ι/39

Page 42: Ευκλειδης Β 61

Μαθηματικά για την Α ' Λυκείου

ΑΒΔ = ΕΒΔ . Απο την ισότητα των τριγώνων αυ­τών προκύπτει η ισότητα: ΔΑΒ = ΔΕΒ = I ορθή, δηλ. το ζητούμενο.

Ά σκηση 3 " Δύο μη ορθογώνια τρίγωνα ΑΒΓ, Α'Β 'Γ έχουν τις γωνίες Α,Α' παραπληρωματικές και ΑΒ = Α 'Β ', ΑΓ = Α'Γ' .

Να αποδείξετε ότι τα ύψη τους, που άγονται από τις κορυφές Γ, Γ είναι ίσα. Το αυτό για τα ύψη που άγονται από τις κορυφές Β,Β '.

Γ '�,- - - - - - υ ·\χ ι \� :

Δ'

\ '"'-- ι \ '·" ι \\ >�Α' \ · · · \ ' I \ I \ '

\j Β '

Α

Είναι: Α + Α: = 2 ορθές .Υποθέτουμε ότι Α < I ορθή, τότε Α' > I ορθή , δηλ. το τρίγωνο Α 'Β Τ' είναι αμβλυγώνιο στο Α' , με αποτέλεσμα το ίχνος Δ ' του ύψους Γ ' Δ ' να βρίσκεται στην προέκταση της ΒΆΌ

Τα 1ψΟ()JγΙ.iJνΗκ τρ ίγΗη'σ ΑΓΔ, ΑΤ' Δ ' είναι iσΗ. αφού έχουν τις υποτείνουσες ΑΓ, Α 'Γ ' ίσες και τις οξείες γωνίες: Γ ΑΔ = 2ορθ - Β 'Α 'Γ ' = Γ ' Α' Δ ' (δηλ. ίσες)

Από την ισότητα των τριγώνων αυτών προκύπτει ότι: ΓΔ=Γ 'ΔΌ

Όμοια, αποδεικνύεται και το δεύτερο σκέλος (Ά­σκηση )

λσκηση .:ι Αν για δύο τρίγωνα ΑΒΓ, Α'Β 'Γ είναι

ΑΒ=Α Ή ', Β = Β ' και ισχύει ΒΔ=Β 'Δ ', όπου ΒΔ,

Β ' Δ ' είναι οι διχοτόμοι των γωνιών Β, Β ' αντί­

στοιχα, τότε τα τρίγωνα αυτά είναι ίσα.

Απ6δ:.:ιξη Ι) Συγκρίνουμε τα τρίγωνα ΑΒΔ, Α 'Β ' Δ ' . Έχουμε:

ΑΒΔ = �Β =�Β ' = Α'Β 'Δ ' ! ΑΒ = Α'Β '

ΒΔ = Β 'Δ ' Σύμφωνα με το κριτήριο Π-Γ-Π , τα τρίγωνα ΑΒΔ, Α 'Β ' Δ ' είναι ίσα οπότε απέναντι των ίσων πλευρών του βρίσκονται ίσες γωνίες. Άρα, θα είναι Α = Α· .

11) Συγκρίνουμε τα τρίγωνα ΑΒΓ, Α 'ΒΤΌ Έχουν {� = � 'Β ' Α = Α' Β = Β '

οπότε σύμφωνα με το κριτήριο Γ -Π-Γ τα τριγω­να ΑΒΓ, Α 'Β 'Γ ' είναι ίσα.

Άσκψηι 5. Δίνεται ισοσκελές τρίγωνο ΑΒΓ με βάση ΒΓ και τυχαίο σημείο Ρ της διχοτόμου Α Τ της γωνίας

Α του τριγώνου. Α ν η ΒΡ τέμνει την πλευρά ΑΓ στο Δ και η ΓΡ τέμνει την πλευρά ΑΒ στο Ε, τό­τε να αποδείξετε ότι: 1 . ΒΔ=ΓΕ. 2. Η Α Τ είναι μεσοκάθετος της ΔΕ.

Απόδειξη Επειδή ΑΒ=ΑΓ η διχοτόμος Α Τ είναι διάμεσος και ύψος δηλ. η Α Τ είναι μεσοκάθετος της ΒΓ.

Β Γ

Το σημείο Ρ ανήκει στη μεσοκάθετο Α Τ της ΒΓ και συνεπώς είναι ΡΒ=ΡΓ, οπότε από το ισοσκελές τρίγωνο ΡΒΓ προκύπτει η ισότητα ΡΒΓ = Pf'B .

ΕΥΚΛΕΙΔΗΣ Β' τ. Ι/40

Page 43: Ευκλειδης Β 61

Μαθηματικά για την Α ' Λυκείου

Στη συνέχεια, συγκρίνουμε τα τρίγωνα ΑΒΔ, ΑΓΕ. Τα τρίγωνα αυτά έχουν: Α κοινή . ΑΒ=ΑΓ και ΑΒΔ = ΑfΈ , σαν διαφορά ίσων γωνιών.

Από το κριτήριο Γ -Π-Γ προκύπτει η ισότητα των τριγώνων αυτών με αποτέλεσμα την ισότητα των πλευρών ΒΔ=ΓΕ, που βρίσκονται απέναντι στη κοινή γωνία Α , καθώς και την ισότητα ΑΔ=ΑΕ.

2 . Από την ισότητα ΑΔ=ΑΕ, στο ισοσκελές τρίγω­νο ΑΕΔ η διχοτόμος Α Τ είναι διάμεσος και ύψος του, δηλ. μεσοκάθετος της ΔΕ.

Άσκηση 6. Αν για δύο τρίγωνα ΑΒΓ, Α'Β'Γ ισχύουν

Β = Β ', Γ = Γ' και υα = υα. , τότε να αποδείξετε

ότι τα τρίγωνα αυτά είναι ίσα.

Απόδειξη Ι) γ ποθέτουμε ότι ίσες γωνίες των τριγώνων είναι οξείες. Φέρουμε τα ύψη ΑΔ = υα , Α'Δ ' = υa· · Τα ίχνη Δ, Δ ' των υψών τους είναι σημεία εσωτερικά των πλευρών ΒΓ,Β 'Γ αντίστοιχα.

Α. Τα ορΟογι:ί)νια τρ ίγωνα ΑΒΔ, Α'Β 'Δ ' είναι ίσα, αφού έχουν ΑΔ=Α 'Δ' και Β = Β ' . Άρα είναι και ΒΔ = Β 'Δ ' .

Β. Τα ορθογώνια τρίγωνα ΑΓ Δ, Α'Γ Δ ' είναι ί­σα, αφού έχουν ΑΔ = Α' Δ ' και Γ = Γ· . Άρα εί­ναι και ΓΔ=Γ Δ ' .

Α

Β Γ Α '

Β ' Γ Παρατηρούμε ότι: Β = Β ' , Γ = Γ · και ΒΓ = ΒΔ + ΔΓ = Β 'Δ ' + Δ 'Γ ' = ΒΤ . Από το κριτή-ριο ισότητας Γ -Π-Γ προκύπτει ότι τριγ ΑΒΓ = τριγ Α'Β 'Γ . 11) γ ποθέτουμε ότι τα τρίγωνα είναι αμβλυγώνια

με Β = Β' > Ι ορθή . Α

Α ' Γ

Β ' Γ Τότε, Τα ίχνη Δ, Δ ' των υψών τους είναι σημεία που βρίσκονται στις προεκτάσεις των πλευρών ΓΒ, ΓΒ ' αντίστοιχα.

Α. Τα ορθογώνια τρίγωνα ΑΓΔ, Α'ΓΔ' είναι ίσα, αφού έχουν ΑΔ=Α' Δ ' και Γ = Γ · . Άρα είναι και ΓΔ=Γ 'ΔΌ

Β. Τα ορθογώνια τρίγωνα ΑΒΔ, Α 'Β ' Δ ' είναι ίσα, αφού έχουν ΑΔ=Α'Δ ' και ΑΒΔ = 2 = 2ορθ - Β = 2ορθ - Β ' = Α 'Β ' Δ ' Άρα, ΒΔ=Β 'ΔΌ Παρατηρούμε ότι: Β = Β ', Γ = Γ · και

ΒΓ=ΔΓ -ΒΔ=Δ 'Γ-Β ' Δ '=Β 'Γ . Από το κριτήριο ισότητας Γ-Π-Γ προκύπτει ότι τριγ ΑΒΓ = τριγ Α 'Β 'Γ .

Π α ρ ατή ρ ηση : Το θέμα που ακολουθεί έχει «πολιτογραφηθεί» σαν το 4" ΚΡΠΗ P I O ισότητας τριγι!η·ων.

Άσκηση 7 Αν για τα τρίγωνα ΑΒΓ, Α 'Β'Γ ισχύουν :

ΑΒ=Α'Β, ΑΓ=Α'Γ, ΑΒ < ΑΓ, Β = Β ' , τότε τα

τρίγωνα αυτά είναι ΙΣΑ.

Απόδειξη Α

" ' '

Α '

' ' ' ' ' ' ' ' Β '-------�Δ-_:::::,. Γ

8 , '--------------� Γ

ΕΥΚΛΕΙΔΗΣ Β' τ. Ι /41

Page 44: Ευκλειδης Β 61

Μαθηματικά για την Α ' Λυκείου

Αρκεί να αποδείξουμε ότι τα τρίγωνα ΑΒΓ, ΑΈ 'Γ ' έχουν και την τρίτη πλευρά ίση . Δηλ. ΒΓ=Β 'Γ ' . Εφαρμόζουμε την μέθοδο απαγωγής στο άτοπο. Δεχόμαστε ότι ΒΓ =ι:. Β 'Γ ' . Αν υποθέ­σουμε ότι ισχύει ΒΓ > Β 'Γ ' , τότε υπάρχει σημείο Δ εσωτερικό του τμήματος ΒΓ τέτοιο, ώστε να είναι ΒΔ = Β 'Γ ' . Συγκρίνοντας τα τρίγωνα ΑΒΔ, Α Έ 'Γ ' διαπιστώ-

νου με ότι: Β = Β ' {ΑΒ = ΑΈ'

ΒΔ = Β 'Γ ' Από το κριτήριο ισότητας Π-Γ -Π τα τρίγωνα ΑΒΔ, Α Έ 'Γ ' είναι ΙΣΑ. Άρα, θα είναι και ΑΔ=Α'Γ ' . Επειδή όμως είναι και ΑΓ=Α'Γ ' θα έ­χουμε ότι ΑΔ=ΑΓ. Από το ισοσκελές τρίγωνο ΑΔΓ είναι ΑΔΓ = Γ . Σύμφωνα με την υπόθεση, η ανισότητα ΑΒ<ΑΓ οδηγεί στην Β > Γ και λόγω της ΑΔΓ = Γ καταλήγουμε ότι Β > ΑΔΓ . Δηλ. σε άτοπο, αφού η γωνία ΑΔΓ είναι μεγαλύτερη της Β σαν εξωτερική γωνία του τριγώνου ΑΒΔ. Σε άτοπο καταλήγουμε και πάλι με παρόμοιο τρό­πο, αν δεχθούμε ότι ισχύει ΒΓ<Β 'Γ ' . Τελικά είναι : ΒΓ=Β 'Γ ' , οπότε με εφαρμογή του κριτηρίου Π-Π-Π τα δύο τρίγωνα είναι ΙΣΑ.

Άσκηση 8. Σε οξυγώνιο τρίγωνο ΑΒΓ με ΑΒ<ΑΓ η διχοτό-

μος της Α τέμνει τη μεσοκάθετο t της πλευράς ΒΓ σε σημείο Ρ.

Να αποδείξετε ότι: ΡΒΑ + Pr Α = 2 ορθές.

Απόδειξη Α

Ρ Φέρουμε ΡΔ .l ΑΒ , ΡΕ .l ΑΓ και συγκρίνουμε τα ορθογώνια τρίγωνα ΡΒΔ, ΡΓΕ. Έχουμε :

Α. ΡΒ=ΡΓ , αφού το Ρ ανήκει στη μεσοκάθετο της ΒΓ

Β. Ρ Δ=ΡΕ, αφού το Ρ ανήκει στη διχοτόμο της Α Συνεπώς ,τα τρίγωνα ΡΒΔ, ΡΓΕ είναι ίσα, με απο-

τέλεσμα ΡΓΕ = ΡΒΔ = 2ορθ - ΡΒΑ , απ όπου προ­κύπτει το ζητούμενο.

Άσκηση 9. Δίνεται αμβλεία γωνία χόy , δύο σημεία Α, Α'

στην Οχ, δύο σημεία Β, Β ' στην Oy τέτοια, ώ­στε: ΟΑ < ΟΑ', ΟΒ < ΟΒ' .

Ν α αποδείξετε ότι : ΑΒ < Α 'Β ' .

Απόδειξη Φέρουμε την ΑΈ. Παρατηρούμε ότι η γωνία Α 'ΒΒ ' είναι εξωτερική γωνία του τριγώνου ΟΑ 'Β, οπότε: Α'ΒΒ ' > Α'ΟΒ > Ιορθ .

ο

φ

Α ' \...ω _ _ _ _ _ Υ

χ

Στο τρίγωνο ΑΈΒ ' η γωνία Α'ΒΒ ' είναι η μεγα­λύτερη γωνία του (αφού είναι αμβλεία) και συνε-πώς ισχύει : Α'ΒΒ ' > Α'ΒΈ . Σύμφωνα με γνωστο θεώρημα, ισχύει

ΑΈ ' > ΑΈ ( 1 ) . Η γωνία φ είναι εξωτερική γωνία του τριγώνου ΟΑΒ , οπότε ισχύει: φ > xOy > 1 ορθ και άρα φ > & . Από το ίδιο θεώ­ρημα στο τρίγωνο ΑΑ 'Β προκύπτει

ΑΈ > ΑΒ (2). Από τις σχέσεις ( 1 ), (2) έχουμε ΑΒ<Α'Β ' .

Άσκηση 1 0. Δίνεται τρίγωνο ΑΒΓ : ΑΒ < ΑΓ , η διχοτόμος

"

ΑΔ της γωνίας Α και Ρ τυχαίο σημείο της ΑΔ. Ν α αποδείξετε ότι:

" "

1) ΡΒΑ > ΡΓΑ 3) ΡΓ > ΡΒ

Απόδειξη

Ε

2) 4)

Α

ΔΒ < ΔΓ ΡΓ - ΡΒ < β - γ

Γ

ΕΥΚΛΕΙΔΗΣ Β' τ. Ι/42

Page 45: Ευκλειδης Β 61

Μαθηματικά για την Α ' Λυκείου

Η κάθετος από το Γστην διχοτόμο ΑΔ τέμνει την προέκταση της ΑΒ στο Ε. Διαπιστώνουμε ότι τα τρίγωνα ΑΓΕ, ΡΓΕ, ΔΓΕ είναι ΙΣΟΣΚΕΛΉ με κο­ρυφές Α,Ρ,Δ αντίστοιχα. (Να το δικαιολογήσετε)

ι ) Η γωνία ΡΒΑ είναι εξωτερική του τριγώνου ΡΒΕ οπότε ισχύει ΡΒΑ > ΡΕΒ = θ = ΡΓ Α . Άρα, ΡΒΑ > ΡΓ Α

2) Τα τρίγωνα ΑΔΕ, ΑΔΓ είναι ίσα, αφού έχουν: {ΑΕ = ΑΓ και την ΡΑ κοινή πλευρά. Από ΡΕ = ΡΓ την ισότητα των τριγώνων αυτών προκύπτει ΑΕΔ = ΑΓΔ ή το αυτό ΒΕΔ = Γ . Η γωνία ΔΒΕ είναι εξωτερική του τριγώνου ΑΒΓ ά­ρα, ισχύει ΔΒΕ > Γ = ΒΕΔ , οπότε ΔΕ>ΔΒ και τέλος ΔΓ>ΔΒ.

3) Τα τρίγωνα ΑΡΕ, ΑΡΓ είναι ΙΣΑ (να το αιτιο­λογήσετε)

Η γωνία ΡΒΕ είναι εξωτερική του τριγώνου ΑΒΡ οπότε ισχύει ΡΒΕ > ΑΡΒ ( 1 ) . Η γωνία ΑΡΒ είναι εξωτερική του τριγώνου ΒΡΔ οπότε ισχύει ΑΡΒ > ΡΔΒ (2). Η γωνία ΡΔΒ είναι εξωτερική του τριγώνου ΑΔΓ οπότε ισχύει

Ρ ΔΒ > Γ > θ = ΡΕΒ (3 ) . Από τις σχέσεις (1 ), (2), (3) προκύπτει :

ΡΒΕ > ΡΕΒ με αποτέλεσμα ΡΕ>ΡΒ δηλ ΡΓ>ΡΒ.

4) Στο τρίγωνο ΡΒΕ από την τριγωνική ανισό­τητα έχουμε:

ΒΕ > ΡΕ - ΡΒ <:::> ΑΕ - ΑΒ > ΡΓ - ΡΒ <:::> ΑΓ - ΑΒ > ΡΓ - ΡΒ <:::> β - γ > ΡΓ - ΡΒ

Οι Πανελλήνιοι Μαθηματικοί Διαγωνισμοί της Ελληνικής Μαθηματικής Εταιρείας θα πραγματοποιηθούν ως εξής:

«0 ΘΑΛΗΣ» 4 Νοεμβρίου 2006

«0 ΕΥΚΛΕΙΔΗΣ» 20 Ιανουαρίου 2007 «0 ΑΡΧΙΜΗΔΗΣ)) 24 Φεβρουαρίου 2007

'Όπως γνωρίζετε ιι Ε.Μ.Ε. χρηματοδοτεί τις δραστηριότητές της διαμέσου οικονομικών πό­ρων, που προέρχονται ι<υρίως από τις συνδρομές των μελών της. Η ετήσια συνδρομή, που ανέρχεται για το 2006 στα 20 Ευρώ, καλύπτει την αποστολή του Ευκλεί­

δη Α και Β δηλαδή τέσσερις αποστολές το χρόνο από 2 περιοδικά κάθε φορά. Για ενημέρωσή σας, επάνω στην ετικέτα με το όνομα σας αναγράφεται το συνολικό ποσό το οποίο οφείλετε στην Ε.Μ . Ε.

Η τακτοποίηση των συνδρομών μπορεί να γίνει: ι . Στα γραφεία της Ε.Μ.Ε. 2. Στα γραφεία των παραρτημάτων της Ε.Μ.Ε. 3 . Με ταχυδρομική επιταγή σε διαταγή,

ΕΛΛΗΝΙΚΉ ΜΑΘΗΜΑτΙΚΉ ΕΤΑΙΡΕΙΑ, Τ ΑΧ. ΓΡΑΦΕΙΟ ΑΘΗΝΑ 54, Τ.Θ. 30044 4. Με κατάθεση του αντιτίμου της συνδρομής στους παρακάτω λογαριασμούς:

ο Τράπεζα ΕΘΝΙΚΉ, λογαριασμός όψεως 080/48002300 ο Τράπεζα ALPHA, λογαριασμός όψεως 1 0 1 00 200 20 1 9 988

Στην περίπτωση που η πληρωμή θα γίνει σε τράπεζα πρέπει να αποστείλετε στο Fax της ΕΜΕ (2 1 0-364 1 025) την απόδειξη κατάθεσης συμπληρωμένη με τα πλήρη στοιχεία του καταθέτη ( ονομα­τεπώνυμο, διεύθυνση, τηλέφωνο, περιοδικό για το οποίο ενδιαφέρεστε) ώστε να είναι εφικτή η ορθή αποστολή των περιοδικών.

Για περισσότερες πληροφορίες μπορείτε να καλέσετε στα γραφεία της ΕΜΕ 2ι Ο-36 ι 6532 και 2ι Ο-36ι 7784.

ΕΥΚΛΕΙΔΗΣ Β' τ. Ι/43

Page 46: Ευκλειδης Β 61

•-•ιιιι-�ι-• !Jοω rJQJ!J lif uc!J�m rJ(j)rJJ ώrJJωe5fJ(J)rJJ

Αλ β - -____ Ή _ _ γε . . . ·-ρα

Τριγωνομετρία. Νίκος Σ. Ταπεινός

ε την έναρξη της νέας σχολικής χρονιάς εύχομαι απόκτηση γνώσεων, που θα σε οδηγήσουν στην εκπλήρωση των ονείρων σου και στην κατάκτηση της επιτυχίας. Η τριγωνομετρία που ακολουθεί, ελπίζω να σε βοηθήσει στην κατανόηση της ενότητας αυτής, καθώς περιλαμβάνει

ασκήσεις που αναφέρονται στην ύλη της Α' και Β ' τάξης του Ενιαίου Λυκείου.

1 . Αν ημχ = � και χ ε (; , π) να υπολογίσε-

τε την τιμή της παράστασης

Α = 2εφχ - 3ημχ

1 Οσφχ + 6συνχ

Λύση

Επειδή χ ε (� ,π

) δηλαδή είναι τόξο του 2°υ

τεταρτημορίου έχουμε:

συνχ = -�1 - ημ2χ = -�1 -% = -

� = -%

-15 ημχ 3 -15 εφχ = -- = - = -- και συνχ 2 2

3 Ι 2 2-/5 σφχ = - = -- = ---εφχ -/5 5

2 (-fi

25 ) - 3

Ji

35

--15--15 Άρα Α = ι ο(- 2:) + 6Η) = -4./5 - 4

=

2-/5 -15 -15(-15- ι) = = = = 4 ( -15 + ι) 2 (-/5 + 1) 2 (-!5 + ι) (-!5 - ι) 5 --/5 5 --/5 = = 2 (5 - 1) 8

2. Αν ημχ+συνχ=κ (1), χ ε R να υπολογίσετε

την τιμή της παράστασης: Α= ημ6χ+συν6χ

Λύση

Από ( 1 ) έχουμε: ( ημχ + συνχ )2 = κ2 <::::>

ημ2χ + συν2χ + 2ημχσυνχ = κ2 <::::> Ι + 2ημχ + συνχ = κ2 <::::>

κ2 - Ι ημχσυνχ = -2-Έχουμε: Α = (ημ2 χ )3

+ (συν2 χ )3 =

(2)

= (ημ2χ + συν2χ ) ( ημ4χ - ημ2 χσυν2χ + συν4χ ) =

= ι [ ( ημ2χ )2 + ( συν2 χ )2 - ημ2χσυν2χ

J =

= { ημ2χ + συν2χ) - 2ημ2χσυν2χ -ημ2χσυν2χ = = 12 - 3ημ2χσυν2χ = Ι - 3 ( ημχσυνχ )2 =

ΕΥΚΛΕΙΔΗΣ Β' τ.l/44

Page 47: Ευκλειδης Β 61

Μαθηματικά για την Β ' Λυκείου

�� -{ κ'2- ι )' � ( κ2 - 1 )2 = 1 - 3 -'-----'---4

4 - 3 (κ4 - 2Κ2 + 1 ) -3κ4 + 6κ2 + 1 = 4 4

3 . Να απλοποιηθεί η παράσταση :

ημ ( 1

�π

+ χ }φ (9π + χ)συν ( 32π + χ)

Α = --�����----��--�

συν ( 9; - χ )σφ ( 1�π- χ )συν ( 5π - χ)

για όλες τις τιμές του χ, που ορίζονται οι αναγραφόμενοι τριγωνομετρικοί αριθμοί.

Λύση

Είναι: ημ( 1�π + χ) =ημ( 8π-� + χ) =ημ( -�+ χ) = = -ημ(� - χ ) = -συνχ

εφ (9π + χ ) = εφ (8π + π + χ ) = εφ( π + χ) = εφχ συν( 3; + χ ) =συν( 2π -� + χ) = = συν( -% + χ ) = συνχ (� - χ ) = ημχ

συν ( 92π - χ ) = συν( 4π +% - χ ) = = συν (% - χ) = ημχ σ φ ( 1 �π _ χ) = σ φ ( 6π -%- χ) = = σφ ( -% - χ) = -σφ (% + χ) = = -σφ [ π - (; - χ)] = σφ(; - χ) = εφχ συν (5π - χ ) = συν ( 4π + π - χ ) = = συν ( π - χ ) = -συνχ Άρα: Α = -συνχεφχημχ = 1 ημχεφχ ( -συνχ)

4. Να χαρακτηρίσετε τις παρακάτω προτάσεις με Σωστό (Σ) ή Λάθος(Λ)

i) Η εξίσωση συνχ=3/2 είναι αδύνατη

i i ) i i i )

ίν)

ν)

vi)

εφχ=εφα <=> χ=2κπ+α με κe Ζ

Η εξίσωση εφ (χ + �) = 8 έχει λύσεις

Ισχύει συν( α-β)=συνα--συνβ για κάθε α,βe R

Η συνάρτηση f(χ)=3ημ(χ-π) έχει ελά­χιστη θετική περίοδο Τ=π Ι , 2 2 2 σχυει συν χ=ημ χ-συν χ

νίί ) σφσφβ + 1 Ισχύει σφ (β - α) =

β για

σφα - σφ

όλα τα α,β που ορίζονται οι παραστά­

σεις

νίίί) 1 + συν2α , Ισχύει σφ2α = για ολα τα α 1 - συν2α

i x )

χ)

που ορίζονται οι παραστάσεις Μια από τις λύσεις της εξίσωσης 2ημ2χ=1 είναι χ=π/1 2

α 2εφ -

Ισχύει εφ2α = 2 για όλα τα α z α

1 - εφ -2

που ορίζονται οι παραστάσεις

5. Να επιλέξετε τη σωστή απάντηση : i ) Περίοδος της συνάρτησης

f(x) = 2συν( 3χ - �) είναι:

π � π α) Τ = -, β) Τ = π, γ) Τ = 2π, δ) Τ = -, ε) Τ = -

2 3 3

i i ) Μια λύση της εξίσωσης 2ημχ = J2 εί-

ναι π π 2π 5π 3π

α) - 4 , β) 2 , γ) 3 , δ) 4 , ε) 4 π

2εφ -ί ί ί) Η τιμή της παράστασης _ ____.:8� είναι: z Π 1 - εφ S

J2 Γ- Jj α) -, β) ν3 , γ) ι, δ) - , ε) ο 2 3 iv) Το ημ4α αeR, είναι ίσο με:

α) 2ημα·συνα, β) 4ημα·συνα.συν2α, α α 2 γ) 2ημ - συν - , δ) 1-2ημ 2α, 2 2

ε) κανένα από τα προηγούμενα

ν) Αν χ=2ημ3α·συν3α και y=1-2ημ23α τότε , 2+ 2 , , η παρασταση χ y ειναι ιση με:

ΕΥΚΛΕΙΔΗΣ Β' τ.Ι/45

Page 48: Ευκλειδης Β 61

Μαθηματικά για την Β ' Λυκείου

J2 δ) J3 α) 1 , β) -, γ) -1, ε) Ο 2 2

6. Να aντιστοιχίσετε κάθε τριγωνομετρικό α­ριθμό της στήλης Α στην παράσταση της στή­λης Β, με την οποία είναι ίσος

Στ ' λ 1 .ημ4χ

2. συν9χ

3. ημ5χ

4.συν7χ

Α Στ ' λ Β

α. ημχ.συν5χ+ημ5χ. συνχ β. συν8χ. συνχ-ημ8χ. ημχ γ. ημ12χ. ημ5χ+ συν12χ. συν5χ δ. συν12χ. ημ5χ+ ημ12χ.συν5χ ε. ημ3χ.συν2χ+ συν3χ.ημ2χ στ. ημ5χ. συνχ-ημχ. συν5χ

χ. συν9χ

7. Να aντιστοιχίσετε κάθε τριγωνομετρικό α­ριθμό της στήλης Α στην παράσταση της στή­λης Β, με την οποία είναι ίσος

Στήλ,η Α Στήλη Β 1 .ημ22χ 1 - συν4χ α.

1 + συν4χ 2. συν22χ β. συν4χ - 1

2 3. εφ22χ συν4χ + 1 γ.

2 4.σφ22χ δ. συν4χ - 1

συν4χ + 1 1 + συν4χ ε. 1 - συν4χ -1 - συν4χ στ.

2 ζ. 1 - συν4χ

2

8. Να λυθεί η εξίσωση :

Λύση

2ημ2χ+ 3· 1 ημχ l -2=0 (1)

( 1 ) <=> 2 1ημχl2+ 3 · 1 ημχ l -2=0 Θέτουμε I ημχ I =y και έχουμε 2/+3y-2=0 που αληθεύει για y=1 /2 ή y=-2

Α I ' ' I I 1 1 ' • ν y = l τοτε εχουμε: ημχ = 2 <=> ημχ = 2 η 1 ημχ = --2

1 π π α) ημχ =- <=>ημχ =ημ-<=>χ = 2κπ+-, κ Ε Ζ 2 6 6 ' 2 5π '71 η Χ = ΚΠ +-, Κ Ε ι.ι.

6. β) ημχ=-�<=>ημχ=ημ(��)<=>χ=2κπ-�, Κ ΕΖ

' 2 7π '71 η Χ = ΚΠ +-, Κ Ε ι.ι. 6 • Α ν y= -2 τότε έχουμε . ι ημχ Ι = -2 που είναι

αδύνατον γιατί . ι ημχ Ι s 1

9. Να βρεθεί το πεδίο ορισμού της συνάρτη­σης:

Λύση

f(x) = 2ημχ

εφ2χ - 3

π Για να ορίζεται η εφχ πρέπει χ * κπ + - με 2 κ Ε Ζ . Ακόμα πρέπει: εφ2χ - 3 * Ο Αν εφ2χ = 3 <=> εφχ = J3 ή εφχ = -J3 •

r:; π π εφχ = ν-' <=> εφχ = εφ- <=> χ = κπ+- με κ Ε Ζ 3 3 εφχ =.J3 <=>εφχ = εφ( -� )<=>χ =κπ-� με κ Ε Ζ

1 Ο. Να λυθεί η εξίσωση συν2χ - J3ημ2χ = Ο (1)

αν χ ε [ Ο, 3;)

Λύση

r:; Jj (1) <=> ν -'ημ2χ = συν2χ <=> εφ2χ =- (2) με 3 π κπ π συν2χ * Ο<=> 2χ * κπ +- <=> χ * -+- με κ Ε Ζ 2 2 4

' 3π κπ π 3π Επειδη : Ο � χ < - <=> Ο � -+- < - <=> 2 2 4 2

1 κ 5 1 5 -- � - < - <=> -- � κ < -4 2 4 2 2

Επειδή κ Ε Ζ έχω κ = Ο <=> χ * π 4

' π π 3π η κ = 1 <::::> x :;t: - +- <=> x :;t: -2 4 4

ΕΥΚΛΕΙΔΗΣ Β' τ.l/46

Page 49: Ευκλειδης Β 61

Μαθηματικά για την Β ' Λυκείου

' 2 π 5π η κ = <:::::> x :;t: π + - <:::::> x :;t: -4 4

Τότε (2) <=:> εφ2χ = εφ� <=:>2χ = κπ+� 6 6

κπ π <=:> χ = -+- κ ε Ζ 2 1 2 ' Επειδή

3π κπ π 3π 1 1 7 ο ::; χ < - <:::::> ο ::; - +-< - <=> -- :::;; κ < -2 2 1 2 2 6 6 κ ε Ζ έχω κ = Ο <:::::> χ = � δεκτή 1 2 ' 1 π π 7π δ ' η κ = <:::::> χ = - + - <:::::> χ = - εκτη 2 1 2 1 2 ' 2 π 1 3π δ ' η κ = <:::::> χ = π + U <:::::> χ = U εκτη

Άρα χ ε ι� , �;, \32π }

1 1 . Να λυθεί η εξίσωση : συν2χ+ ημ22χ=l (1)

Λύση ( 1 ) <:::::> συν2χ- 1 +(2ημχ . συνχ)2=0 <=:>-ημ2χ+4ημ2χ . συν2χ=Ο <=:>ημ2χ( 4συν2χ- 1 )=Ο <:::::> ημ2χ=Ο ή 4συν2χ-1 =Ο

• Αν ημ2χ=Ο<=:> ημχ=Ο <:::::> ημχ=ημΟ<=:>χ=κπ με κεΖ

• Αν 4συν2χ- 1 =0 <:::::> 4συν2χ= 1 1 <:::::> συν2χ= 1 /4 <=> συνχ = -2

π π <:::::> συνχ = συν- <:::::> χ = 2κπ±- με κ ε Ζ 3 3 . ' 1 2π 2π η συνχ = -- <:::::> συνχ =συν- <:::::> χ = 2κπ ±- με κ ε Ζ

2 3 3

1 2 . Να αποδείξετε ότι

Λύση

1 + συν8α + συν4α 1 - εφ2 2α ------- = -----'---

ημ8α + ημ4α 2εφ2α

1 + συν8α + συν4α 1 + 2συν2 4α - 1 + συν4α ημ8α + ημ4α 2ημ4ασυν4α + ημ4α συν4α (2συν4α + Ι ) Ι = = σφ4α = -- = ημ4α( 2συν4α + 1 ) εφ4α

= ---

2εφ2α

1 3 . Αν εφχ=2/5 να υπολογισθεί η τιμή της πα­ράστασης Α=2ημ2χ+5συν2χ

Λύση 2εφχ Ι - εφ2χ Α = 2ημ2χ + 5συν2χ = 2 2 + 5 2 Ι + εφ χ 1 + εφ χ

4εφχ + 5 - 5εφ2χ =

Ι + εφ2χ

� + 5 _ _± 29

4� + 5 - 5__±_ ---=-.5 ----=2c:;_5 =

Ι + __±_ 25

= 5 5 = _2_ = 5 29 29 . 25 25

1 4 . Αν ημχ+συνχ=S/4 (1) να υπολογίσετε το

ημ2χ

Λύση Από την ( 1 ) έχω:

2 25 25 ( ημχ +συνχ) =- <=:>ημ2χ +συν2χ + 2ημχσυνχ =- <:::::> Ι6 16 25 9 Ι + ημ2χ = - <:::::> ημ2χ = -. Ι6 Ι6

1 5. Αν α+β= � να δείξετε ότι: 3

(2συνα+συνβ)2+(2ημα-ημβ)2=7

Λύση (2συνα+συνβ)2+(2ημα-ημβ)2= =4συν2α+συν2β+4συνασυνβ+4 ημ2α + ημ2β-4 ημαημβ= =4( συν2α+ ημ2α)+( συν2β+ ημ2β) +4( συνα.συνβ-ημα.ημβ)=

π Ι =4. Ι + Ι +4συν(α+β)=5+4 · συν-=5+4 · - =7 3 2

1 6. Να δείξετε ότι: ( 2 π )( 2 π )( 2 π) π 1 - εφ - 1 - εφ - 1 - εφ - = 8εφ-

32 16 8 32

Λύση

ΕΥΚΛΕΙΔΗΣ Β ' τ.Ι/47

Page 50: Ευκλειδης Β 61

Μαθηματικά για την Β' Λυκείου

Γνωρίζουμε ότι: εφ2χ = 2εφχ <:::::> 1 - εφ2χ = 2εφχ

1 - εφ2χ εφ2χ Τότε έχουμε: ( 1 - εφ2

3� )( 1 - εφ2 1: )( 1 - εφ2 �) =

π π π π εφ- εφ- εφ- εφ-= 2 ____21_ 2 ___l§_ 2 -

8 = 8 _____R = 8εφ � . π π π π 32 εφ- εφ- εφ- εφ-1 6 8 4 4

17. Αν σε τρίγωνο ισχύει: συν2Α-συν2Α=ημΑ(1-ημΑ) (1) να δείξετε ότι:

Β+Γ=150° ή Β+Γ=30°

Λύση ( 1 ) <:::::> συν2 Α-συν2Α=ημΑ-ημ2 Α

<:::::> 1 - συν2Α=ημΑ <:::::> 1 -( 1 -2 ημ2 Α)= η μΑ <:::::>2 ημ2 Α= η μΑ <:::::> ημΑ(2ημΑ-l )=Ο

Επειδή Α γωνία τριγώνου ισχύει ότι ημΑ;CΟ Οπότε έχουμε: 2ημΑ-1 =0<:=> ημΑ= l /2 <:::::> ημΑ=ημ30° <:=>Α=30°<:=> Β+Γ= 1 50° ή ημΑ=ημ 1 50° <:=>Α= 1 50°<:=> Β+Γ=30°

1 8. Σε τρίγωνο ΑΒΓ είναι:

J3 J3 εφΒ = - και εφΓ = -

2 5 Να υπολογίσετε τη γωνία Α

Λύση Σε τρίγωνο ΑΒΓ γνωρίζουμε ότι

Α+Β+Γ= 1 80°<:=>Α= 1 80°-(Β+Γ) Οπότε εφΑ= -εφ(Β+Γ)

Α εφΒ + εφΓ <:::::> εφ = <:::::> 1 - εφΒεφΓ Jj Jj 7J3 - + -

εφΑ = 2 5 --- εφΑ 1 0 ...-. Jj Jj .... = - 1 0 - 3 -..-..

l - 25 1() εφΑ = -J-3 <:::::> εφΑ = εφ120 <:::::> Α = 1 20°

1 9. Σε κυρτό τετράπλευρο ΑΒΓΔ να αποδείξετε

, Α Β Γ Δ οτι: εφ- + εφ - + εφ - + εφ- =

2 2 2 2

Λύση

Α Β Γ Α Β Δ εφ-εφ-εφ - + εφ-εφ-εφ- +

2 2 2 2 2 2 Α Γ Δ Β Γ Δ

+εφ-εφ -εφ- + εφ-εφ -εφ-2 2 2 2 2 2

Σε κάθε κυρτό τετράπλευρο ΑΒΓ Δ γνωρίζουμε ό­τι: Α + Β + Γ + Δ = 360° <:::::> Α + Β + Γ + Δ = 1 80ο <:::::> 2 2 2 2 Α + Β = 1 80ο _ ( Γ + Δ ) 2 2 2 2 Οπότε έχουμε: εφ ( � + �) = -εφ(� + �) <=>

Α Β Γ Δ εφ- + εφ- εφ- + εφ-2 2 = 2 2 <:::::> Α Β Γ Δ 1 - εφ-εφ- 1 - εφ-εφ-2 2 2 2 Α Β Α Γ Δ Β Γ Δ �-+�--�-�-�--�-�-�- = 2 2 2 2 2 2 2 2

Γ Δ Α Β Γ Α Β Δ =-�--εφ- +�-�-εφ-+�-�-�-<:=> 2 2 2 2 2 2 2 2 Α Β Γ Δ εφ- + εφ-+ εφ-+ εφ- = 2 2 2 2 Α Β Γ Α Β Δ = εφ-εφ-εφ- + εφ-εφ-εφ-+ 2 2 2 2 2 2

Α Γ Δ Β Γ Δ εφ-εφ-εφ-+ εφ-εφ-εφ-2 2 2 2 2 2

20. Αν σε τρίγωνο ΑΒΓ ισχύει ότι: ημΒ(συνΓ -ημΒ)=συνΒ(συνΒ-ημΓ) (1),

aποδείξτε ότι αυτό είναι ορθογώνιο

Λύση ( 1 ) <:=>ημΒσυνΓ -ημ2Β=συν2Β-συνΒημΓ

<:=>ημΒσυνΓ +συνΒημΓ= ημ2Β+συν2Β <:=>η μ(Β+ Γ)= 1 <:=>ημΑ= l (γιατί Α+Β+Γ= 1 80°) οπότε Α=90°

Απαντήσεις : Άσκηση4. Άσκη ση 5 . Άσκη ση 6 .

Σ, Λ, Σ, Λ, Λ, Λ, Σ, Σ, Σ, Λ δ, ε, γ, β, α 1-στ, 2-β, 3-ε, 4-γ

ΕΥΚΛΕΙΔΗΣ Β' τ.l/48

Page 51: Ευκλειδης Β 61

Μαθηματικά για την Β ' Λυκείου

Γεωμετρία Μετρικές Σχέσεις στο Τρίγωνο

του Αποστόλη Κακαβά

Οι Μετρικές σχέσεις στο τρίγωνο βασίζονται στο πυθαγόρειο Θεώρημα (Π.Θ.) με τις συναφείς προτάσεις και στις συνέπειές του, δηλαδή στα θεωρήματα οξείας και αμβλείας γωνίας που αποτελούν την επέκταση του Πυθαγορείου θεωρήματος (Ε.Π.Θ.) καθώς και στα θεωρήματα διαμέσων (Θ.Δ.) ι. Έστω τρίγωνο ΑΒΓ με πλευρές α,β,γ. Άντι­

στοιχίστε τη σχέση που συνδέει τις πλευρές Α

του, της στήλης Α με τη γωνία Α του τρι-γώνου της στήλης Β.

Στήλη Α Στήλη Β

ι . α2 = β2 + γ2 + βγJ3 α. Α = 30°

2. α2 = β2 + γ2

3. α2 = β2 + γ2 _ βγ.fi

4. α2 = β2 + γ2 + βγ

5. α2 = β2 + γ2 _ βγJ3

6. α2 = β2 + γ2 _ βγ 7. α2 = β2 + γ2 + βγ.fi

Λίι ση

β. Α = 45°

γ. Α = 50°

δ. Α = 60°

ε. Α = 90°

στ. A = l20°

ζ. Α = ι25°

η. A = l35°

θ. Α = ι5ο0

1 � θ , διότι α2 = β2 + γ2 + βγ.J3 �

�-2βγσυνΑ = βγ../3 �συνΑ=-.J3 �Α= 150° 2 Ομοίως 2 � ε, 3 � β, 4 � στ, 5 � α, 6 � δ, 7 � η

2. Δίνεται ορθογώνιο τρίγωνο ΑΒΓ με Α = 90° και μήκη πλευρών ΒΓ=20 και ΑΓ=ι2. Στο μέσο Μ της υποτείνουσας ΒΓ φέρω ευθεί­α(ε) κάθετη στη ΒΓ που τέμνει την ΑΒ στο εσωτερικό της σημείο Δ. Να υπολογιστούν τα μήκη των τμημάτων ΔΓ και ΔΑ. Λύση Εφαρμόζουμε το Π. Θ. στο τρίγωνο ΑΒΓ οπότε ΑΒ2=ΒΓ2- ΑΓ2 = 400-144=256 Άρα: ΑΒ= 1 6

Γ

Β

Τα τρίγωνα ΑΒΓ και ΒΜΔ είναι όμοια ( Α = Μ = 90° και Β κοινή) άρα

ΔΒ = ΜΒ <:::::> ΔΒ = 1 0 <:::::> ΔΒ = 200 <:::::> ΔΒ = 25 ΒΓ ΑΒ 20 1 6 1 6 2 Το Δ σημείο της μεσοκαθέτου του ΒΓ άρα

25 25 7 ΔΓ = ΔΒ = - και ΑΔ =ΑΒ-ΔΒ= 16-- =-2 2 2 3. Δίνεται ορθογώνιο τρίγωνο ΑΒΓ με υποτεί­

νουσα ΒΓ=5 και ύψος ΑΔ=2,4. Να υπολογίσε­τε τα μήκη των τμημάτων ΔΒ, ΔΓ, ΑΒ, ΑΓ. Λύση Έχουμε ΔΒ+ΔΓ = ΒΓ = 5 και

ΔΒ · ΔΓ = ΑΔ2 = 2,42 = 5 ,76. Άρα τα ΔΒ, ΔΓ είναι ρίζες της εξίσωσης:

χ2-5χ+5 ,76=0 με Ο<χ<5 Η εξίσωση αυτή έχει διακρίνουσα Δ=25-23 ,04= 1 ,96 και ρίζες χ 1 = 5 + 1 • 4 = 3, 2 2

5 - 1 , 4 I 8 , , και χ2 = -2- = , που προφανως ανηκουν στο (0,5).

Γ

JL.l._ ____ � Α Β

Άρα (ΔΒ= 1 ,8 και ΔΓ=3 ,2) ή (ΔΒ=3 ,2 ΔΓ= 1 ,8) • Αν ΔΒ= Ι ,8 και ΔΓ=3 ,2 τότε

και

ΑΒ2=ΔΒ"ΒΓ= \ ,8 .5=9 οπότε ΑΒ=3 και

ΑΓ2=ΔΓΒΓ =3 ,2·5= 1 6 οπότε ΑΓ=4 Αν ΔΒ=3 ,2 και ΔΓ= 1 ,8 τότε ομοίως ΑΒ=4 και ΑΓ=3 β ' τρόπος

β2 + γ2 = 25 } β2 + γ2 + 2βγ = 49} βγ = αυα = 5 · 2, 4 = 1 2 �

β2 + γ2 - 2βγ = 1 �

ΕΥΚΛΕΙΔΗΣ Β ' τ. t /49

Page 52: Ευκλειδης Β 61

Μαθηματικά για την Β ' Λυκείου

β + γ = 7} ' β + γ = 7} η κ.λπ. β - γ = Ι γ - β = Ι

4. Δίνεται ορθογώνιο και ισοσκελές τρίγωνο

ΑΒΓ με Α = 90° και ΒΓ=α, η διάμεσός του ΑΜ και Ν το μέσο της. Στη ΒΓ έστω σημείο

Κ με ΓΚ= 3α

. Να δειχθεί ότι η ΚΝ είναι 8

κάθετη στη ΝΒ.

Απόδειξη Α = 90° }=> ΑΜ = ΒΓ = � ΑΜ διάμεσος 2 2

Άρα ΝΜ = ΑΜ = � 2 4

α 3α α ΜΚ = ΜΓ - ΓΚ = --- = - . 2 8 8

Εφαρμόζουμε το Πυθαγόρειο Θεώρημα στο ΚΜΝ οπότε ΚΝ2 =ΚΜ2 +ΝΜ2 = (�)2 + (�)2 =

α2 α2 5α2 = -+- = - (1 ) 64 1 6 64 Γ

Β

Εφαρμόζουμε το Πυθαγόρειο Θεώρημα στο ΜΝΒ οπότε ΒΝ2 =ΜΝ2 +ΜΒ2 = ( �)2 + (�)2 =

α2 α2 5α2 = -+- = - (2) Ι 6 4 Ι 6 Εξάλλου: ΒΚ2 = (α -ΓΚ)2 = ( 5; )2 = 2:2

Δ Από ( Ι ), (2) έχουμε στο Κ Ν Β :

5 2 5 2 25 2 (3) ΚΝ2 + ΒΝ2 =�+�=�=ΒΚ2 64 Ι 6 64

Άρα ΚΝΒ = 90° δηλαδή ΚΝ 1_ ΝΒ .

(3)

5. Έστω ορθογώνιο τρίγωνο ΑΒΓ με Α = 90° •

Κατασκευάζουμε εξωτερικά του τριγώνου, τετράγωνα ΒΓ ΔΕ πλευράς α, ΑΒΘΗ πλευ­ράς γ και ΑΓΖΙ πλευράς β. Φέρω ΔΚ κάθε­τη στην ΖΓ που τέμνει την προέκτασή της στο Κ. Να δειχθεί ότι ί) τα τρίγωνα ΑΒΓ και ΔΚΓ είναι ίσα και ii) ΖΔ 2+ΙΗ2+ΕΘ2=6α2

Λύση i) Τα τρίγωνα ΔΚΓ και ΑΒΓ είναι ίσα, διότι είναι

ορθογώνια και έχουν ΒΓ=Γ Δ=α και AfB = 90° - φ = κfΔ .

ii) Από (i) παίρνουμε ΓΚ=ΑΓ=β και ΔΚ=ΑΒ= γ Δ

Εφαρμόζουμε το Π. Θ. στο Δ ΚΖ και έχουμε: ΔΖ2=κz2+ΚΔ 2=(2β)2+γ2 =4β2+ γ2 ( Ι ) Ομοίως ΕΘ2 =4γ2+β2 (2), ενώ ΙΗ2=β2+γ2 (3) Από τις ( Ι ), (2), (3) έχουμε:

ΔΖ2+1Η2+ ΕΘ2=4β2+γ2+(β2+γ2)+4γ2+β2= =6β2+6γ2=6(β2+γ2)= 6α2.

Δ I I I I I I I I I

r"""'-------'-<-r - - - - - - - - - - - - - - - Ω φ κ

β

β ' τρόπος: ΔΖ2 = α2 + β2 - 2αβσυνΔf'Ζ = = α 2 + β 2 - 2αβσυν ( Ι 80° - Γ) =

α

= α2 + β2 + 2αβσυνΓ = α2 + β2 + 2αβ · � = α

= α2 + 3β2 = β2 + γ2 + 3β2 = 4β2 + γ2 κ.λ.π.

Ε

6. Δίνεται ισόπλευρο τρίγωνο ΑΒΓ πλευράς α. Προεκτείνουμε τις πλευρές ΑΒ, ΒΓ, ΓΑ κα­τά ΒΖ=ΓΔ=ΑΕ=α. Να βρεθούν τα μήκη των πλευρών του τριγώνου ΔΕΖ.

Λύση Ι3 = 60° � Βεξ = 1 20° Εφαρμόζουμε το νόμο συνημιτόνων στο

ΕΥΚΛΕΙΔΗΣ Β' τ.l/50

Page 53: Ευκλειδης Β 61

Μαθηματικά για την Β ' Λυκείου

Δ ΒΖΔ . ΔΖ2 = ΒΖ2 + ΒΔ2 - 2ΒΖ · ΒΔσυνΖΒΔ = = α2 + (2αγ - 2α2ασυνΙ 20° = 5α2 - 4α2 ( -�) = = 5α2 + 2α2 = 7α2 .

Ε

Δ

z

Άρα ΔΖ = α-fi . Ομοίως ΔΕ = ΕΖ = αJΊ .

7. Δίνεται ισόπλευρο τρίγωνο ΑΒΓ πλευράς α

και σημείο Δ της ΒΓ ώστε ΒΔ = ! . Αν Κ 3

σημείο της πλευράς ΑΒ ώστε ΑΚ=ΚΔ τότε: i) Να υπολογίσετε τα τμήματα ΚΔ, ΚΓ ii) Να δείξετε ότι δεν υπάρχει τρίγωνο με

πλευρές ΚΒ, ΚΓ, ΒΔ.

Απόδειξη ί) Εφαρμόζουμε το νόμο συνημιτότων στο

Δ ΚΒΔ και έχουμε: ΚΔ2 = ΚΒ2 + ΒΔ2 - 2 · ΒΚ · ΒΔσυνΒ =

2 = (α - ΑΚ)2 +� - 2 (α - ΑΚ)�συν60° = 9 3 2 α2 α Ι = (α - ΚΔ) + 9 - 2 (α - ΚΔ)"3 · "2 =

2 2 α2 α2 α · ΚΔ = α - 2αΚΔ + ΚΔ +---+--9 3 3 ΚΔ 2 2 Ά 2 ΚΔ α · 2 α α δ λ δ ' ρα: α --3- = α +9-3, η α η

ΚΔ α α , 7α 2ΚΔ -- = α +- -- οποτε ΚΔ = - ( Ι ) 3 9 3 ' Ι 5

Α

Β

Δ Ομοίως από το νόμο συνημιτότων στο Α Γ Κ έχουμε: ΓΚ2 = ΑΚ2 + ΑΓ2 - 2 · ΑΚ · ΑΓ · συνΑ =

2 2 Ι = ΑΚ + α - 2 · ΑΚ · α- = 2 2 2 < Ι J 49α2 2 7α = ΚΔ + α - α · ΚΔ =--+ α - α- = 225 Ι 5

49α2 + 225α2 - 1 05α2 Ι 69α2 = = 225 225 Οπότε: ΓΚ = Ι 3α (2) Ι 5

ίί) Έχουμε: 7α 8α ΚΒ = α - ΑΚ = α - ΚΔ = α -- = ­Ι 5 Ι 5

Άρα ΚΒ + ΒΔ = 8α + � = Ι 3α � ΚΓ Ι 5 3 Ι 5

(3)

δηλαδή δεν υπάρχει τρίγωνο με πλευρές ΚΒ, ΚΓ, ΒΔ.

8. Έστω τρίγωνο ΑΒΓ με μήκη διαμέσων

� Jϊii5 m , μα = ν-'-' ' μμ = -2 - , μ1 = -2- , να βρεθει

το είδος του τριγώνου ως προς τις γωνίες του.

Λύση

2 2β2 + 2γ2 - α2 μα = 4 2 2α2 + 2γ2 - β2 μβ = 4 2 2α2 + 2β2 - γ2 μγ = 4

2β2 + 2γ2 - α2 = 1 32} <::::> 2α2 + 2γ2 - β2 = 1 05

2α2 + 2β2 - γ2 = 33

(Ι ) (2) (3)

(+) (Ι) , (2) , (3)=:>3α2 + 3β2 + 3γ2 = 270=:>α2 +β2 + γ2 = 90 2α2 + 2β2 + 2γ2 = Ι 80 (4)

(-) ( 4) , ( Ι )=:>3α2 = 48 :::::> α2 = Ι 6

(-) (4) , (2 )=:>3β2 = 75 =:> β2 = 25

ΕΥΚΛΕΙΔΗΣ Β' τ. Ι /51

Page 54: Ευκλειδης Β 61

Μαθηματικά για την Β' Λυκείου

(- ) (4) , (3)=>3γ2 = 147 => γ2 = 49 Άρα α2 +β2 = 1 6+ 25 = 41 < 49 = γ2 οπότε: f > 90° .

9. Αν σε ένα τρίγωνο ΑΒΓ ισχύουν οι σχέσεις

4μ2 α=α2+2γ2 και α2+β2=2γ·μy τότε το τρίγωνο είναι ορθογώνιο και ισοσκελές.

Απόδειξη

4 2 2 2 2 4 2β2 + 2γ2 - α2 μα = α + γ => 4

Δ α = β => Α Β Γ ισοσκελές.

Από υπόθεση έχουμε: α2 + β2 = 2γ · μr ( l )

Από ( 1 ° Θ. Δ.) έχουμε : 2 α2 + β2 = 2 · μ2 +r_

Ύ 2 (2)

10. Δίνεται ρόμβος ΑΒΓΔ και Μ σημείο της προέκτασης της διαγωνίου Γ Α προς το Α.

Να δειχθεί ότι ΜΔ2 - ΑΔ2 = ΜΑ · ΜΓ

Απόδειξη : Έστω Κ το μέσο του ΜΑ και Ο το σημείο το­μής της διαγωνίου του. Τότε ΟΑ < ΟΜ => ΔΑ < ΔΜ .

Μ

Γ Δ

Εφαρμόζουμε (2° Θ. Δ.) στο ΜΔΑ :

ΜΔ2 - ΑΔ2 = 2ΜΑΚΟ = ΜΑ2(ΚΑ+ΑΟ) = ΜΑ (2ΚΑ+2ΑΟ) = ΜΑ (ΜΑ+ΑΓ) = ΜΑΜΓ

1 1 . Δίνεται τραπέζιο ΑΒΓΔ με ΑΒ//ΓΔ (ΑΒ>ΓΔ). Να δειχθεί ότι:

ΑΓ2+ΒΔ 2=ΒΓ2+ΔΑ 2+2ΑΒ·Γ Δ

Απόδειξη Έστω Μ το μέσο της ΑΓ και Ν το μέσο της

ΑΒ - ΓΔ ΒΔ τότε ΜΝ = ( l ) 2

Α Β

Δ Δ Εφαρμόζουμε το { 1 ° Θ. Δ.) στα ΑΒΔ, ΒΓ Δ ,

Δ ΝΑΓ οπότε

ΑΒ2 + ΑΔ2 = 2ΑΝ2 + ΔΒ2 (2) 2 ΒΓ2 + ΓΔ2 = 2ΓΝ2 + ΔΒ2 (3) 2 ΑΝ2 + ΓΝ2 = 2ΜΝ2 + ΑΓ2 (4) 2

(+ ) (2) , (3)=>ΑΒ2 + ΒΓ2 + ΓΔ2 + ΔΑ2 =

< 4 > ( ΑΓ2 ) = 2 (ΑΝ2 + ΓΝ2 ) + ΔΒ2 = 2 2ΜΝ2 + -2- + ΔΒ2

( ι ) (ΑΒ-ΓΔ)2 = 4ΜΝ2 +ΑΓ2 + ΔΒ2 =4 2 +ΑΓ2 +ΔΒ2

Άρα: ΑΒ2 + ΒΓ2 + Γ Δ2 + ΔΑ 2 = = ΑΒ2 - 2ΑΒ · ΓΔ + ΓΔ2 +ΑΓ2 + ΔΒ2 Οπότε: ΑΓ2 + ΔΒ2 = ΒΓ2 + ΑΔ2 + 2ΑΒ · Γ Δ

12. Έστω ισόπλευρο τρίγωνο ΑΒΓ πλευράς α και ευθεία ε//ΒΓ που διέρχεται από το κέ­ντρο βάρους θ του τριγώνου ΑΒΓ. Αν Μ σημείο της ε, να δειχθεί ότι

ΜΒ2 + ΜΓ2 = 2ΜΑ2

Απόδειξη Έστω Δ το ίχνος του ύψους ΑΔ=υ. Τότε

αJ3 2υ 2 4υ2 υ = -- <=> α = - <=> α = - ( l ) 2 J3 3 ΑΘ = �υ (2) 3

ΕΥΚΛΕΙΔΗΣ Β' τ. t /52

Page 55: Ευκλειδης Β 61

Μαθηματικά για την Β ' Λυκείου

1 ΘΔ = -υ (3) 3

Β

Α

Μ (ε)

Δ Εφαρμόζω το ( 1 ° Θ. Δ.) στο ΜΒΓ οπότε:

ΒΓ2 ΜΒ2 + ΜΓ2 = 2ΜΔ2 + --2 α2 ( ι ) 2 = 2 (ΘΔ2 + ΜΘ2 ) +2= 2ΘΔ2 + 2ΜΘ2 + }υ2

�2 (�)2 + 2 (ΑΜ2 - ΑΘ2 ) +%υ2

(2 ) υ2 2 ( 2υ )2 2 2 = 29+ 2ΑΜ - 2 3 + 3υ

= �υ2 + 2ΑΜ2 -�υ2 +�υ2 = 2 · ΑΜ2 • 9 9 9

13. Δίνεται ισόπλευρο τρίγωνο ΑΒΓ με πλευρά α και Μ τυχαίο εσωτερικό σημείο της πλευ­ράς ΑΒ. Ν α δείξετε ότι:

ΜΓ2 - ΜΒ2 i) = α (1) ΜΑ και ii) υπάρχει τρίγωνο με πλευρές ΜΑ, ΜΒ, ΜΓ

Απόδειξη Έστω ΑΛ, ΜΚ .l ΒΓ

Δ i) Εφαρμόζουμε το ( 1 ° Θ. Δ.) Μ ΒΓ και έ-

χουμε: B = f > fΊ � ΜΓ > ΜΒ � ΜΓ2 - ΜΒ2 = 2α · ΚΛ ,

2ΚΛ , ΚΛ 1 οπότε αρκεί ΜΑ = 1 , η ΜΑ = 2 Α

Β Κ Λ Γ ΚΛ ΚΒ 1 Πράγματι: ΜΚ // ΑΛ � ΜΑ = ΜΒ = 2 , αφού

Α 1 Μ, = 30° � ΚΒ = 2ΜΒ ii) Από την ( 1 ) παίρνουμε:

ΜΓ + ΜΒ = α = ΒΓ > 1} ΜΑ ΜΓ - ΜΒ ΜΓ - ΜΒ ΜΓ - ΜΒ = α = ΒΓ < 1 ΜΑ ΜΓ + ΜΒ ΜΓ + ΜΒ (τριγωνική ανισότητα στο Μ Β Γ)

Άρα ΜΓ+ΜΒ>ΜΑ>ΜΓ - ΜΒ, οπότε υπάρχει τρίγωνο με πλευρές ΜΑ, ΜΒ, ΜΓ

β ' τρόπος: Δ

Αν ΜΝ//ΒΓ, τότε το ΑΜΝ είναι ισόπλευρο και το ΜΒΓΝ ισοσκελές τραπέζιο . Επομένως

Δ ΜΝ=ΜΑ, ΝΓ=ΜΒ. Άρα το ΜΝΓ είναι το ζητούμενο τρίγωνο.

a Μετρικές Σχέσεις στον Κύκλο

Τέμνουσες Κύκλου Αν θεωρήσουμε έναν κύκλο (O,R) και σημείο Σ του επιπέδου του, τότε αποδεικνύεται ότι το γινό-μενο ΣΑΣΒ (όπου ΣΑΒ τέμνουσα του κύκλου) εί-ναι σταθερό, δηλαδή ανεξάρτητο από τη θέση της τέμνουσας, αλλά εξαρτάται από τη θέση του Σ ως προς το κέντρο του κύκλου. Έτσι έχουμε τις εξής περιπτώσεις: Ι. Αν Σ εσωτερικό του κύκλου και ΟΣ=δ τότε

ισχύει

ΕΥΚΛΕΙΔΗΣ Β' τ. Ι/53

Θανάσης Τσιούμας

Δ Β

Σχ. 1

Page 56: Ευκλειδης Β 61

Μαθηματικά για την Β ' Λυκείου

Π. Σ εξωτερικό του κύκλου τότε ισχύει

Μ Σχ. 2

ΣΑ-ΣΒ=ΣΓ·ΣΔ= ΣΜ2= δ2 -R2 όπου δ=ΟΣ και ΣΜ το εφαπτόμενο τμήμα

Ση μείωση

Σ

Η τέμνουσα ΣΑΒ κατά την κίνησή της γύρω από το Σ παίρνει την οριακή θέση ΣΜ γι' αυτό έχουμε ΣΑ·ΣΒ = ΣΜ·ΣΜ= ΣΜ2

Παρατη ρήσεις Ι . Η ισότητα ΣΑ · ΣΒ = ΣΓ · ΣΔ εκφράζει το

θεώρημα των τεμνόμενων χορδών, που έχει εφαρμογή σε υπολογισμούς τμημάτων που τε­λειώνουν σε κύκλο.

ΑΓ=5 ΒΓ=6 ΑΣ=χ;

Λύση

Α

Γ

Έχουμε ΓΣ·ΓΑ=ΓΜ·ΓΒ ή (5-χ)-5=3 ·6� x= l ,4

2 . Ισχύει και το αντίστροφο του θεωρήματος των τεμνόμενων χορδών, δηλαδή αν ΣΑ·ΣΒ=ΣΓ·ΣΔ όπου Σ, Α, Β και Σ, Γ, Δ συ­νευθειακά σημεία, τότε τα Α, Β, Γ, Δ είναι ο­

μοκυκλικά.

Π αράδειγμα Γιατί στο παρακάτω σχήμα το ΣΡΓ Δ είναι

εγγράψιμο σε κύκλο; όπου ΑΒ διάμετρος

και ΑΣ εφαπτόμενο τμήμα. Σ

Β

Απάντηση Είναι ΒΓ·ΒΡ=ΒΑ2 (μετρικές σχέσεις στα ορ­θογώνια τρίγωνα) Και ΒΔ· ΒΣ= ΒΑ 2 (μετρικές σχέσεις στα ορθο­γώνια τρίγωνα) Άρα ΒΓ·ΒΡ=ΒΔ·ΒΣ οπότε τα Σ,Ρ,Γ,Δ είναι ομοκυκλικά.

Σημείωση Αν ΣΡ2 =ΣΑ·ΣΒ όπου Σ,Α,Β συνευθειακά τότε ο κύκλος που διέρχεται από τα Ρ ,Α,Β είναι ε­φαπτόμενος της ΣΡ.

Δύναμη σημείου ως προς κύκλο. Η δύναμη σημείου Σ ως προς κύκλο (O,R) ο-ρίζεται ως εξής: IΔ�o,R > = ΟΣ2 - R 2 1 Έχουμε τις περιπτώσεις: i) Αν Σ εξωτερικό του κύκλου (δ>R)

(σχ.2) τότε η Δ�ο,R > συμπίπτει με το γι-νόμενο ΣΑ-ΣΒ και το ΣΜ2, δηλαδή έ­χουμε

ΔΣ = ΣΑ · ΣΒ = ΣΓ · ΣΔ = δ2 - R2 = ΣΜ2 > 0 (O, R )

ii) Αν Σ εσωτερικό του κύκλου (δ<R) (σχ. Ι ) τότε

Δ�ο,R > = -ΣΑ · ΣΒ = -ΣΓ · ΣΔ = δ2 - R2 < Ο iii) Αν Σ είναι πάνω στον κύκλο τότε

Δ�Ο,R ) = ο

Π αράδειγμα Έστω Ρ τυχαίο σημείο χορδής ΑΒ κύκλου

(0, R) να αποδείξετε ότι:

ΟΒ2 = 0Ρ2 + ΡΑ · ΡΒ

ΕΥΚΛΕΙΔΗΣ Β' τ.Ι /54

Page 57: Ευκλειδης Β 61

Μαθηματικά για την Β ' Λυκείου

Λύση Το Ρ είναι εσωτερικό σημείο του κύκλου άρα Δ�ο,R > = ΟΡ2 - R 2 = -ΡΑ · ΡΒ .

Επομένως ΟΡ2 + ΡΑ · ΡΒ = R 2 = 082

Σχόλιο. Οι μετρικές σχέσεις του κύκλου, μας βοηθούν

να κατασκευάσουμε γεωμετρικά τις θετικές ρίζες ορισμένων Β/θμιων εξισώσεων. Γνωστό είναι το πρόβλημα της χρυσής τομής: δηλαδή να διαιρε­θεί ένα δοσμένο τμήμα ΑΒ=α σε μέσο και άκρο λόγο, δηλαδή χ=; (χ<α) ώστε χ2=α(α-χ) προκύ-

πτει: ι� �� � φι (χρυσός λόγος) αρχικό της λέξης Φειδίας. Επειδή ο χρυσός λόγος καθορίζει τις τέλειες αναλογίες σε ένα σώμα, χρησιμοποιή­θηκε κυρίως στην τέχνη. Ακόμα και ο αριθμός των κερκίδων, του κάτω διαζώματος, προς των αριθ­μών των κερκίδων του πάνω διαζώματος του Αρ-

, θ ' Ε δ ' ' λ ' 34 Φ χαιου εατρου της πι α υ ρου εχει ογο 2J = .

Το παραπάνω θέατρο όπως είναι γνωστό έχει εκ­πληκτική ακουστική .

Λυμένες Ασκήσεις Ι . Δίνεται κύκλος διαμέτρου ΑΒ=2 και οι χορ­

δές του ΑΓ και ΒΔ που τέμνονται στο Σ. Να

αποδείξετε ότι ΑΣ·ΑΓ+ΒΣ·ΒΔ=4 R2

Λύση Φέρουμε ΣΡ l. ΑΒ τότε το τετράπλευρο ΣΡΒΓ

είναι εγγράψιμο σε κύκλο διότι και f = 90° ( εγγε­γραμμένη σε ημικύκλιο) οπότε ΑΣ·ΑΓ=ΑΡ-ΑΒ

( Ι ) . Ομοίως το ΑΡΣΔ είναι εγγράψιμο άρα ΒΣ·ΒΔ=ΒΡ·ΒΑ. Με πρόσθεση κατά μέλη των ( 1 ), (2) έχουμε: ΑΣ· ΑΓ +ΒΣ· ΒΔ=ΑΒ(ΑΡ+ΡΒ)= ΑΒ · ΑΒ=ΑΒ2= (2R2)= 4R2 •

2 . Δίνεται κύκλος (0,3). Στην προέκταση της

διαμέτρου ΑΒ, να βρεθεί σημείο Σ, τέτοιο ώστε το εφαπτόμενο τμήμα ΣΜ, να είναι δι­

πλάσιο του ΣΒ.

Λύση Από το θεώρημα τέμνουσας και εφαπτομένης έχουμε ΣΜ2 =ΣΒ·ΣΑ ( 1 )

Σ Α

Έστω ΣΒ=χ τότε ΣΜ= 2χ και ΣΑ= 6+ χ οπότε η ( 1 ) γράφεται (2χ)2= χ· (6+ χ) <::::> 4χ=6+ χ<::> 3χ=6<::> χ=2. Άρα το Σ απέχει από το κέντρο απόσταση ΟΣ=5 .

3 . Δίνεται τρίγωνο εγγεγραμμένο σε κύκλο (0,

R). Έστω ΑΜ η διάμεσος του, που τέμνει

τον κύκλο στο Ρ και Θ το βαρύκεντρο του

τριγώνου. Να αποδειχτεί ότι:

α2 i) ΜΑ · ΜΡ = -

4

ii) Δe = _.!_ (α2 + β2 + γ2 ) (O,R) 9

Λύση i) Από το θεώρημα των τεμνόμενων χορδών έ­

α α α2 χουμε: ΜΑ · ΜΡ = ΜΒ · ΜΓ = - · - =- ( 1 ) 2 2 4 ii) Το Θ είναι εσωτερικό του κύκλου (0, R).

Άρα Δ�ο.R > = -ΘΑ · ΘΡ = -ΘΑ (ΘΜ + ΜΡ ) = = -ΘΑ · ΘΜ - ΘΑ · ΜΡ =

2 Ι 2 ι • > 2 2 2 α2 2 2 α2 =--μα ·-μα --ΜΑ·.ΜΡ=--μα --- =--μα -- (2) 3 3 3 3 3 4 3 6 ΕΥΚΛΕΙΔΗΣ Β ' τ.t /55

Page 58: Ευκλειδης Β 61

Μαθηματικά για την Β ' Λυκείου

Ρ

, 2 2β2 + 2γ2 - α2 Όμως μα = 4 (3)

Από (2), (3) έχουμε : λ0 _ 2 ( 2β2 + 2γ2 - α2 ) α2 _ 1 ( 2 β2 2 u<O. R J - -3 4 - 6 - -9 α + + γ

4 . Δίνεται τρίγωνο ΑΒΓ (ΑΒ<ΑΓ) εγγεγραμμέ­

νο σε κύκλο (0, R). Από το μέσο Μ της ΒΓ

φέρουμε ΜΡ κάθετη στη διάμετρο ΑΔ, που

τέμνει τον κύκλο στα Ε και Ζ. Να αποδείξε­

τε ότι: i) το τρίγωνο ΑΕΖ είναι ισοσκελές ii) ΑΒ2+ΑΓ2=ΑΕ2+ΑΖ2

Λύση ί) Επειδή η ΟΡ, άρα και η ΑΡ είναι κάθετη στη χορδή ΕΖ τότε το Ρ θα είναι το μέσο της ΕΖ, επομένως η διάμεσος ΑΡ είναι και ύψος του τριγώνου ΑΕΖ οπότε αυτό θα είναι ισοσκελές.

ii) Εφαρμόζουμε το 1 ο θεώρημα της διαμέσου στο τρίγωνο ΑΒΓ και έχουμε ΑΒ2+ΑΓ2=2ΑΜ2+2ΒΜ2 ( 1 ) Είναι ΒΜ2=ΒΜ·ΜΓ=ΜΕ·ΜΖ= =(ΡΕ - ΜΡ)(ΡΖ + ΜΡ)= =(ΡΕ - ΜΡ)(ΡΕ + ΜΡ)=ΡΕ2 - ΜΡ2 (2) . (ΡΕ = ΡΖ) Από ( 1 ) , (2) προκύπτει ότι ΑΒ2+ΑΓ2= =2ΑΜ2 + 2ΡΕ2 - 2ΜΡ2=2ΡΕ2 + 2(ΑΜ2 -ΜΡ2)

=2ΡΕ2 + 2ΑΡ2=2(ΡΕ2 + ΑΡ2)=2ΑΕ2= =ΑΕ2+ ΑΖ2 αφού ΑΕ=ΑΖ

5 Έστω κύκλος (Ο, R) και ορθογώνιο τρίγωνο

ΑΒΓ εγγεγραμμένο στον κύκλο, με ΑΒ=2.

Προεκτείνουμε την Γ Α κατά ΑΣ=2. Α ν το

εφαπτόμενο τμήμα ΣΜ είναι ΣΜ=4 να υπο­λογίσετε α) τη χορδή ΑΓ β) την ΟΣ γ) την

6

προβολή του ΑΒ πάνω στη ΒΓ Σ

Γ

Λύση α) Από το θεώρημα τέμνουσας και εφαπτομέ­νης έχουμε:

ΣΜ 2= ΣΑ. ΣΓ <:::>4 2=2(2 +Α Γ)<:=> I ΑΓ=6 1

β) Υπολογίζουμε τη διάμετρο από το Π.Θ. στο ορθογώνιο τρίγωνο ΑΒΓ είναι ΒΓ2=ΑΒ2+ΑΓ2 ή ΒΓ2=22+62<:::> IBΓ = 2MI δηλαδή R = Μ . Επειδή ΣΜ2=ΣΟ2-R2<:::>42=Σd-fι02 �ΣΟ=J261

γ) Έστω ΒΔ = προβ�� . Έχουμε ΑΒ2 = ΒΔ · ΒΓ (μετρικές στα ορθογώνια τρί­

γωνα) ή 2' � ΒΔ · 2Ji0 <ο> ΙΒΔ � �� · Θεωρούμε κύκλο (0, R) και ΑΒ μια διάμε­

τρός του. Α ν Μ τυχαίο σημείο του κύκλου

(διαφορετικό των Α,Β) και Ρ,Σ τα μέσα των

ακτίνων ΟΑ και ΟΒ αντίστοιχα, να αποδει­

χθεί ότι:

3R2 3R2 i) ΡΚ =-- και ΣΛ = --

4ΜΡ 4ΜΣ

ii) ΜΡ + ΜΣ

= 1 Ο

ΡΚ ΣΛ 3

ΛίJση ί) Από το θεώρημα των τεμνόμενων χορδών έ­

χουμε: ΕΥΚΛΕΙΔΗΣ Β' τ. Ι /56

Page 59: Ευκλειδης Β 61

Μαθηματικά για την Β' Λυκείου

1 3 3 2 ΡΚ · ΡΜ = ΡΑ · ΡΒ =-R · - R =-R 2 2 4 ' 3R2

οπότε ΡΚ = -- ( 1 ) . 4ΜΡ ' 3 1 3 2 Ομοιως ΜΣ · ΣΛ = ΑΣ · ΣΒ = -R-R = -R . 2 2 4

Ά α ΣΛ = 3R 2 (2) ρ 4ΜΣ

Μ

ii) Από ( 1 ), (2) έχουμε:

Όμως η ΜΟ είναι διάμεσος του τριγώνου ΜΡΣ, άρα από το 1 ο θεώρημα της διαμέσου είναι:

MPz + ΜΣz = 2ΜΟ2 + ΡΣ2 = 2R2 + R

2 = 5R

2 (4) 2 2 2 Από τις (3), (4) προκύπτει ότι

ΜΡ ΜΣ 4 · 5R2 1 0 - +- = = -ΡΚ ΣΛ 3 · 2R2 3 .

7 Σε κύκλο (0, R) η εφαπτομένη σε τυχαίο του, σημείο Γ, τέμνει την προέκταση της δι­αμέτρου ΑΒ στο Σ. Α ν η κάθετη στην ΑΒ στο Σ, τέμνει την ευθεία ΑΓ στο Ε, να απο­δείξετε ότι:

ΑΓ · ΑΕ = ΣΑ 2 - ΣΓ2 •

Λύση Το Σ είναι εξωτερικό του κύκλου οπότε

Δ�ο,R > = ΣΓ2 = ΣΒ · ΣΑ ( 1 ).

Άρα ΣΑ 2 - ΣΓ2 = ΣΑ 2 - ΣΒ · ΣΑ = = ΣΑ (ΣΑ - ΣΒ) = ΣΑ · ΑΒ (2).

Αν φέρουμε τη ΒΓ τότε Br Α = 90° ως εγγεγραμ­μένη σε ημικύκλιο, άρα το τετράπλευρο ΓΒΣΕ εί­ναι εγγράψιμο, αφού και ΑΣΕ = 90° , Επομένως:

ΑΒ · ΑΣ = ΑΓ · ΑΕ (3). Από (2), (3) προκύπτει ότι ΑΓ · ΑΕ = ΣΑ 2 - ΣΓ2 •

8) Θεωρούμε οξυγώνιο τρίγωνο ΑΒΓ και ΑΔ α2

το ύψος του. Αν ΑΗ · ΑΔ = - (1) όπου Η 2 το ορθόκεντρο του τριγώνου, να αποδείξετε

αJ3 ότι: i) β2 + γ2 = 2α2 ii) μα = -2-

Λύση i) Φέρουμε το ύψος ΒΕ, επειδή Α < 90° από το

θεώρημα της οξείας γωνίας θα έχουμε: α2 = β2 + γ2 - 2ΑΕ · ΑΓ <::> <:::> β2 + γ2 = α2 + 2ΑΕ · ΑΓ

Α

Β

(2).

Όμως το τετράπλευρο ΗΔΓΕ είναι εγγράψιμο

( ) < ι > αz Δ + Ε = 1 80° άρα ΑΕ · ΑΓ = ΑΗ · ΑΔ =τ οπό-

2 τε η (2) γίνεται: β2 + γ2 = α2 + 2� ή β2 + γ2 = 2α2 • 2 ι· ι· ) Ε' 2 2β 2 + 2γ2 - α2 , ιναι μα = 4 η

2 2 (β2 + γ2 ) - α2 2 · 2α2 - α2 3α2 , . μα = 4 = 4 = 4 απο (ι) .

Άρα μα = α.J3 . 2 ΕΥΚΛΕΙΔΗΣ Β' τ. Ι /57

Page 60: Ευκλειδης Β 61

Μαθηματικά για την Β ' Λυκείου

Μαθη ματικά Κατεύθυνσης Διανύσματα

Ένα πλήθος ασκήσεων του Διανυσματικού Λογι­σμού αντιμετωπίζεται με την χρήση της άσκησης 4, σελίδα 28 του Σχολικού βιβλίου, σύμφωνα με την οποία:

ο

Γ Α ν Δ είναι σημείο μεταξύ των Β και Γ τέτοιο

ώστε �� = t ( Ι ) όπου μ,λ>Ο, τότε για κάθε ση­μείο Ο, θα έχουμε:

� =t }<=> ΔΒ = -t:ΔΓ <::> λΔΒ + μΔΓ = δ ( 1 )

ΔΒ i-1- ΔΓ <=> λ ( ΟΒ - ΟΔ ) + μ ( ΟΓ - ΟΔ ) = δ <::> ΟΔ = -λ λ ΟΒ +-λ μ ΟΓ + μ + μ <=> ΟΔ = λ� μ ( λΟΒ + μΟΓ )

Άρα από την λΔΒ + μΔΓ = Ο προκύπτει ότι ΟΔ // ν = λΟΒ + μΟΓ .

Εφαρμογές α) Αν ΟΔ διάμεσος τότε μ = λ οπότε:

- λ - λ - 1 - 1 -ΟΔ = lλ ΟΒ + lλ ΟΓ = lOB + lOΓ

β ΟΔ δ , , , ΔΒ β , ) Α ν ιχοτομος, τοτε ισχυει: ΔΓ = γ , αρα

λ β = . μ γ Επομένως από την (1), θα έχουμε:

ΟΔ = _β_ΟΒ + _γ_ΟΓ β + γ β + γ

γ) Αν Bor = 90° και ΟΔ είναι το ύψος τότε: ΔΒ ΟΒ2 , μ γ2

ΔΓ = ΟΓ2 . Ετσι λ = pz ·

Επομένως από την (1 ), προκύπτει:

Θάνος Χαραλάμπους - Βαγγέλης Ευσταθίου

β2 γ2 ΟΔ = 2 2 ΟΒ + 2 2 ΟΓ

β + γ β + γ Σχόλιο :

Αν ΔΒ = μ και Δ στην προέκταση του ΒΓ ΔΓ λ δηλαδή ΔΒ ίί ΔΓ τότε ΔΒ = � ΔΓ οπότε λ λΔΒ - μΔΓ = δ και ΟΔ = -λ-ΟΒ--μ-ΟΓ λ - μ λ - μ

'Ε , , , ΔΒ μ τσι γενικοτερα αντι της σχεσης ΔΓ = λ με μ,λ>Ο μπορούμε να θεωρήσουμε την χΔΒ + ψΔΓ = δ με χ,ψ ε JR* Άσκηση Ι η Έστω τρίγωνο ΑΒΓ και Ι το έγκεντρό του. Να υπολογισθεί το διάνυσμα θέσης του εγκέντρου Ι,

συναρτήσει των διανυσμάτων θέσης των κορυ­φών του τριγώνου.

Λύση Σύμφωνα με τα προηγούμενα,

ΟΔ = -β-ΟΒ + -γ-ΟΓ β + γ β + γ όπου Δ το ίχνος της εσωτερικής διχοτόμου της

γωνίας Α . Ομοίως από το τρίγωνο ΑΒΔ, έχουμε:

α γ ΙΔ ΒΔ β + γ α - = - = -- = --ΙΑ ΑΒ γ β + γ - -

Δηλαδή αΑΙ + (β + γ)ΔΙ = Ο �

α ( ΟΙ - ΟΑ ) + (β + γ) ( ΟΙ - ΟΔ ) = δ <=>

ΟΙ = α ΟΑ + β + γ ΟΔ <=> α + (β + γ) α + (β + γ) ΟΙ = α ΟΑ + ( β ΟΒ + γ or ) = α + β + γ α + β + γ α + β + γ - - -αΟΑ + βΟΒ + γΟΓ = α + β + γ

ΕΥΚΛΕΙΔΗΣ Β' τ.l/58

Page 61: Ευκλειδης Β 61

Μαθηματικά για την Β ' Λυκείου

Άσκηση 2 '1 Δίνεται ισόπλευρο τρίγωνο ΑΒΓ και σημείο Ο του επιπέδου του, ώστε να ισχύει:

- - - -

60Α + 150Β + 50Γ = Ο (1), Να αποδείξετε ότι: ΟΑ .l ΟΓ .

Λύση 1 "' τρόπος Α ν v = 1 50Β + 50 Γ τότε σύμφωνα με τα όσα α­ναφέραμε προηγουμένως θα είναι ν // ΟΔ όπου Δ

- - -

σημείο .τέτοιο ώστε: 1 5ΔΒ + 5ΔΓ = Ο (2), (προφα-νώς επί της ΒΓ) και μάλιστα - 1 5 - 5 - 1 50Β + 5ΟΓ < 1 > -6ΟΑ ΟΔ = 20 ΟΒ + 20 ΟΓ = 20 =----w---3 -= - 1 0 ΟΑ.

- 3 -Άρα ΟΔ = - 1 0 ΟΑ (3) .

Β

Έχουμε:

Α

Γ

- - 3 - - 10-( 3 ) � ΑΔ - ΑΟ = 1 0ΑΟ � ΑΟ = Ϊ3ΑΔ (4) - 15 - 5 - 3- Ι ­Επίσης ( 2 ) � ΑΔ = 20 ΑΒ + 20 ΑΓ = 4 ΑΒ + 4 ΑΓ

και - 10 ( 3 - 1 - ) 5 ( - -) ( 4 ) � ΑΟ = u 4AB + 4Ar = 26 3ΑΒ + ΑΓ

Άρα: ΓΟ = ΑΟ - ΑΓ = ;6 ( 3ΑΒ + ΑΓ) - ΑΓ =

= �6 ( 5ΑΒ - 7ΑΓ ) .

Επομένως: - - 5 3 ( -

- ) ( - - ) ΑΟ · ΓΟ = 26 26 3ΑΒ + ΑΓ 5ΑΒ - 7ΑΓ = - 1 5ΑΒ - 1 6ΑΒ · ΑΓ - 7ΑΓ 1 5 ( -2 - - -2 )

262 1 5 =-( 1 5α2 - 1 6α2συν60 - 5α2 ) 262 1 5 ( 2 2 1 z ) =- 1 5α - 1 6α - - 5α = 0 262 2

zος τρόπος (Με συντεταγμένες) Θεωρούμε ορθοκανονικό σύστημα συντεταγ-, ( α α./3 ) μενων χΒψ με Γ(α,Ο) και Α 2 ,-2- .

Υ

Γ(α,Ο) χ

Τότε από την 60Α + 1 50Β + 50Γ = δ προκύπτει: 6ΑΟ + 1 5ΒΟ + 5ΓΟ = δ , οπότε 6( χ -% , ψ - α�) + 1 5(χ, ψ) + 5(χ - α, ψ) = (0, 0)

, 4α 3J3α Άρα: χ = u Ψ = 13 . 'Ετσι ΑΟ = -�:( 1, 2./3 ) και ΓΟ = �:( -6,./3 )

- - 1 5α2 Επομένως: ΑΟ · ΓΟ = - 262 ( -6 + 2 · 3 ) = Ο

Άσκηση 3 η Αν για τυχαίο σημείο Ο στο επίπεδο ισο­

πλεύρου τριγώνου ΑΒΓ ισχύει

κΟΑ + λΟΒ + μΟΓ = Ο (1) τότε να βρεθεί η

ικανή και αναγκαία σχέση μεταξύ των κ,λ,μ ώστε ΟΑ .l ΟΓ

Λύση κΑΟ + λΒΟ + μΓΟ = δ �

κ( χ -% , ψ- α�) + λ( χ,ψ) + μ( χ - α,ψ ) = (Ο, Ο) �

ΕΥΚΛΕΙΔΗΣ Β ' τ.l/59

Page 62: Ευκλειδης Β 61

Μαθηματικά για την Β ' Λυκείου

κα (� + μ )α (κ + λ + μ)χ = - + μα} χ = λ 2 κ + + μ

� κα.J3 α.J3 (κ + λ + μ)ψ = -- κ-2 2 ψ = κ + λ + μ Επειδή Α, Β, Γ είναι μη συγγραμμικά θα ισχύει κ + λ + μ * Ο . Έτσι:

ΑΟ = 2 - � κ-2- _ α.J3 = [ (� + μ )α α.J3 J κ + λ + μ 2 ' κ + λ + μ 2

= 2( κ :� + μ) ( λ - μ, (λ + μ )J3

)

και ΓΟ = - α 2 = [ ( � + μ )α κ αJ3 J κ + λ + μ ' κ + λ + μ [ ( � + λ )α κ αJ3 J = κ + λ + μ ' κ +

/+ μ =

= - 2(κ +:

+ μ)( κ + 2λ, -κJ3

)

Τότε ΑΟ l_ ΓΟ � ( λ - μ ) ( κ + 2λ ) - 3κ ( λ + μ ) = Ο � κλ + 2λ2 - κμ - 2λμ - 3κλ - 3κμ = Ο � λ2 - (κ + μ)λ - 2κμ = Ο ( Ι ) π. χ. Για κ = 6 , λ = 1 5 παίρνουμε μ = 5

Άσκηση 4η Δίνονται τα συνεπίπεδα διανύσματα α, β, γ για τα οποία ισχύει ι α ι = j β j = I Ύ I = .J2 και

2r( α+β) = α · β + 6 Να αποδειχθεί ότι α + β = 2r

Απόδειξη Αρκεί να αποδείξουμε ότι α + β - 2r = Ο . Όμως για να αποδείξουμε ότι ένα διάνυσμα ισού­ται με το μηδενικό διάνυσμα, αρκεί να δείξουμε ότι το μέτρο του είναι ίσο με το μηδέν. Αρκεί λοιπόν να δείξουμε ότι I α + β - 2y I = ο . Έχουμε: 1 α + β - 2::Υ Ι 2 = α2 + β2 + 4::Υ2 + 2α . β - 4β · ::Υ - 4::Υ · α = Ι α 1 2 + Ι β 1 2 + 4 Ι Ύ Ι2 + 2 [ α . β - 2::Υ ( α + β ) J = 2 + 2 + 4 . 2 + 2( -6) = ο

Άσκηση 511 Δίνονται τα μη συγγραμμικά διανύσματα με I α I = 2 . Να δειχθεί ότι:

ί) - α . χ -

προβ� = -- α α - 2 α

ίί) Α ν για ένα διάνυσμα χ ισχύει: ( α · � ) � = 12προβ� + 4β (t) - - -ι . χ = 3α + λβ

α, β

2. Αν το διάστυμα χ που ικανοποιεί την (1) είναι μοναδικό τότε να βρεθεί το λ.

Απόδειξη Έστω � = u + ν , όπου u = προβ� , άρα ν l_ α . Τότε u 1 1 α άρα υπάρχει λ Ε ]R* ώστε u = λα ή - 2 α · u = λα . i) Όμως α · � = α( u + ν ) = α · u + α · ν , αλλά

α . ν = ο , άρα α . � = α . υ i

/ν ά - - - z α · χ Έτσι έχουμε α · χ = λα ή λ = --=-z · α

- α · χ - - α · χ -Τελικά u = --=-zα ή προβ� = --=-zα α α

ίί) ( 1 ) ::::} αχ ::;Ζ: Ο διαφορετικά θα έχουμε β = Ο δηλαδή β ιι α πράγμα άτοπο. Άρα: (1 ) ::::} (α� ) � = 1 2 αχ α + β ::::} � = 3α + _4_ β 4 αχ - - -::::} χ = 3α + λβ 2. Ισχύει: ( α · � ) � = 1 2προβ� + 4β ( α · � ) � · α = ι 2α · προβ� + 4α . β ( α . � ) � . α = ι 2α . � + 4α . β ( α . � )2 = 1 2α . � + 4α . β ( α . � )2 - 1 2α . � - 4α . β = ο Θέτουμε α · � = ω και η τελευταία γίνεται

ω2 - 1 2ω - 4α . β = ο Αφού το χ είναι μοναδικό θα είναι και το α · χ = ω μοναδικό. Συνεπώς η εξίσωση

ΕΥΚΛΕΙΔΗΣ Β ' τ. Ι /60

Page 63: Ευκλειδης Β 61

Μαθηματικά για την Β ' Λυκείου

ω2 - 1 2ω - 4α · β = Ο θα έχει διπλή ρίζα, άρα Δ = ο � ( - 1 2 )2 - 4 ( -4α . β ) = ο

Απόδειξη Εφόσον τα α, β είναι μη συγγραμμικά, άρα θα

� 144 + 1 6α . β = ο � α . β = -9 Άρα έχω ω2 - 1 2ω + 36 = Ο � ( ω - 6/ = Ο

� ω = 6 � α · χ = 6

υπάρχουν μοναδικοί κ, λ ώστε: - - -χ = κα + λβ

' α . χ = κα2 + λα . β } α\ + α . βλ = α . χ Άρα: - - - - -2 � - - -2 - -

Άσκηση 6'1

β · χ = κα · β + λβ α · βκ + λβ = β · χ Λύνουμε το σύστημα ως προς κ, λ: -2 - - - - - - -2 α α · β α · χ α · β α α · χ D = -2 ' DK = - - - 2 ' Dλ =

Εάν α, β είναι δύο μη συγγραμμικά διανύσμα- α . β β β · χ β α · β β · χ Όμως D = α2 β2 - ( αβ )2 > Ο , αφού τα, να αποδείξετε ότι για κάθε διάνυσμα χ του

επιπέδου των α, β ισχύει η σχέση : α � β 9� =l � l βl lω.(�)l <l � l βl =>cα. βy <a2 . β2 • α · χ α · β

- 2 β · χ β

Χ = - 2 α α · β

- 2 α · β β

ΜΑθΗΜΑΥΙΚΑ & ΠΟΙΧΕΙΑ ΠΑτΙΠΙΚΗΣ

Γ' Ενιαfαυ Λυκεfου Διαφορικό<; Αοyιομός. Σtιmιmι<ή. Πιθανότητες

- 2 α

α + α · β

- 2 α

α · β

α · χ

β · χ

α · β - 2 β

β

'Ε DK λ = Dλ τσι: κ = 0, D - D - D -Οπότε: χ = -κ α + -λ β D D

.,. ΕΠΙΣΤΗΜΟΝΙΚΑ Για ΑΕΙ, ΤΕΙ, ΙΕΚ, ΑΣΕΠ ��> ΕΚΠΑΙΔΕΠΙΚΑ Για Δημοτικό, Γυμνάσιο, λύκειο, ΤΕΕ Ι> ΛΟΓΟΤΕΧΝΙΑ, ΜΕΛΕΤΕΣ, ΛΕΥΚΩΜΑ Τ Α

ΜΑθΗΜΑΥΙΚΑ θετικ4ς & τεχνολ. Κατεuθ.

Γ' Ενισfσυ Λuκεlοu Τdμας 1: Μιyαδtκοl οριθμοl, Όριο & συνέχεια σννόρτησης

ΜΑΟΗΜΑΥIΚΑ θετικ4ς & τεχνολ. Κστευθ. Γ' Ενισlου Λuκεlου

Τόμος 2: Διαφορικό<; λΟ'(ΙΟμός. Ολοιιληρωnκός λο'(Ιομός

θΟΜΑΣ ΚΥΙΙΕΝΠΔΗΣ, ΛΙΙΙΠΕΡΛ ΜΛ8ΗΜΛΠΙΙΛ, 3 -• θlθΛΙΟΠΩΛ'ΕΙΟ • Ι<ΕΝΊ'ΡΙΚΗ ΑΙΑΘΕ:ΣΗ: ΑρμενοnοuλοU 'D · 546 35 Θεοσαλοli(κη • Τηλ. 2310..203.720 • Fax 231Q-2 1 1 .305 • e-mai: saleι@zili.g • θlθΝΟΠΩΛΕΙΟ .ΑΘΗΝΩΝ • •ΕΝΩΣΗ ΕΚΔΟmΝ ΒΙβΛΙΟV θΕΣΣΑΛΟΝΙΚΗLο Σtοό τοιι Βιβλίου (ΠεσμοζόγλοU 5) - 105 84 ΑθΗΝΑ • Τηλ..faχ 210-321 1 .097 • ΑΠΟΘΗΚΗ ΑΘΗΝΩΝ - ΠΩΛΗΣΗ ΧΟΝΔΡIΚΗ: Ασι<ληπιού 80 - Εξάρχεια 1 1 4 7 1 . Αθήνα • Τηλ.-Faχ 210-3818.1150 • e-mail: athinι@zili.gr

ΕΥΚΛΕΙΔΗΣ Β ' τ.l/61

Page 64: Ευκλειδης Β 61

�-•ιιιι8-ιι8• [YO(JJ fJ[Jjf!J r fJc!J&j[[J f](f)[!) !Jr!Jωe56(f)r!J

Μαθημα,τικά Κατεύθυνσης Μιγαδικοί Αριθμοί

Γεράσιμος Κουτσανδρέας

Λίγο πριν : Είναι χρήσιμο να έχουμε μια γενική εικόνα στους μιγαδικούς αριθμούς μέσα από μερικές

ασκήσεις που θεωρούμε ότι έχουν ενδιαφέρον σε σημεία που παρουσιάζουν δυσκολίες.

Έτσι οι κάποιες πρώτες παρατηρήσεις δίνουν ένα δείγμα ουσιαστικής παρέμβασης στο ξεκαθάρισμα εννοιών.

Ακολουθούν ασκήσεις - Θέματα που υλοποιούν το γενικότερο πνεύμα της συνολικής κάλυψης μέσα από θέματα πανελλαδικών εξετάσεων (εξετάσεις 2006) αλλά και επισημάνσεων που διαμορφώνουν μια καλή θεώρηση στους μιγαδικούς αριθμούς και βοηθούν πάρα πολύ τους αναγνώστες να έχουν μια πιο πλήρη κάλυψη ουσιαστικού ενδιαφέροντος.

Παρατηρήσεις: • Οι δυνάμεις ενός μιγαδικού αριθμού z με

εκθέτη ακέραιο, ορίζονται ακριβώς όπως και στους πραγματικούς αριθμούς.

αριθμοί να έχουν άθροισμα πραγματικό αριθμό χωρίς να είναι συζυγείς π. χ. z1 = 2 + 3ί , Ζ2 = 5 - 3ί . ii) z - z = 2βί .Δηλαδή η διαφορά δυο συζυγών

• Επίσης για z :;z: ο ορίζουμε zo = Ι και μιγαδικών αριθμών είναι φανταστικός αριθμός.

z-ν = _..!__ , για κάθε ν Ε Ν . Ζν

• Δεν έχει νόημα η διάταξη στους μιγαδικούς αριθμούς.

• Δεν έχει νόημα το σύμβολο της � στους μιγαδικούς αριθμούς, δηλαδή δεν γράφουμε ..Γz .

• Για τους συζυγείς μιγαδικούς αριθμούς z = α + βί και z = α - βί ισχύουν :

Το αντίστροφο δεν ισχύει. Υπάρχουν μιγαδικοί αριθμοί που έχουν διαφορά φανταστικό αριθμό χωρίς να είναι συζυγείς π. χ. z1 = 5 + 3ί , Ζ2 = 5 - i . iii) Αποδεικνύονται εύκολα οι προτάσεις

z · � = α2 + β2 -

z E R {:} z = z -

z ε Ι {::} Ζ = -z • Δυο ίσοι μιγαδικοί έχουν ίσα μέτρα. Το

αντίστροφο δεν ισχύει. Υπάρχουν άπειροι i) z + z = 2α . Δηλαδή το άθροισμα δυο συζυγών μιγαδικοί αριθμοί με το ίδιο μέτρο. μιγαδικών αριθμών είναι πραγματικός αριθμός. π 2 + · Ι + 2 · 2 · λ .χ. z1 = ι, z2 = ι , z3 = - ι κ.τ . . Το αντίστροφο δεν ισχύει: μπορεί δυο μιγαδικοί

ΕΥΚΛΕΙΔΗΣ Β' τ.Ι /62

Page 65: Ευκλειδης Β 61

Μαθηματικά για την Γ Λυκείου

• Για το μέτρο των μιγαδικών αριθμών ισχύουν: i) l zl = ι� ι = 1-zl = ι-�ι ii) l z l2 = z · � i i i ) l z Ι · z2 Ι = lz � l l z2 1 ίν) l zv l = lzl v

ν) l zl = Ο � z = Ο νί) l l z � l - l z2 1 1 � Ι z1 + 22 l � l z Ι I + lz2 1

• Το μέτρο της διαφοράς δυο μιγαδικών αριθμών είναι ίσο με την απόσταση των εικόνων τους στο μιγαδικό επίπεδο.

• Βασικοί γεωμετρικοί τόποι των σημείων M(z) για τους οποίους ισχύει:

i) lz - z0 1 = ρ, ρ > Ο , κύκλος με κέντρο το σημείο Α ( z0 ) και ακτίνα ρ. ii) lz - z 1 1 = lz - z2 1 , η μεσοκάθετη του ευθυγράμμου τμήματος με άκρα τα σημεία A (z1 ) , B ( z� ) . ί ί ί) lz - z1 1 = ρ · l z - z2 1 , ρ 7: 1, ρ > Ο , απολλώνιος κύκλος. ίν) lz - z1 1 + 1 z - z2 1 = 2α, α > Ο, και lz 1 - z2 1 < 2α , έλλειψη . ν) l l z - z1 1 - l z - z2 1 1 = 2α, α > Ο, και l z 1 - z2 1 > 2α , υπερβολή .

Θέμα 1 " Θεωρούμε τους μιγαδικούς

z 1 = - _!_ + J3 i και z2 = ι + z 1 2 2

α) Αποδείξτε ότι: ί) ι + Ζ1 + z ; = 0

ii) z: = ι

αριθμούς

1•1•1.) 2ν+ Ι Ζ ν+2 z 2 = - ι ' 2ν ν ( * ) z2 = z1 ν ε Ν

β) Να γράψετε στη μορφή α + βί τους z�6 και

z•9 2

Λύση :

Είναι z2 = [-.!_ + J3 ί ] 2 = .!_ - 2 . .!_ _ J3 i - i =

I 2 2 4 2 2 4

1 J3 . = ----ι.

Άρα 2 2

(ί) 1 + z + z2 = 1 - .!_ + J3 ί - .!_ - J3 ί = Ο I I 2 2 2 2 (ii) 1 + z1 + z� = Ο {::} ( Ζ1 - 1 ) (ι + Ζ1 + z� ) = Ο {::}

{::} z� - 1 = Ο {::} z� = 1 . (ί ίί) Είναι 1 + z1 = -z� δηλαδή Επομένως : z;v+l =( -� )2ν+Ι =-Ζι4ν+2 =-zι"+2 . z;v =-zι"+2 ( zΠ =

2// - (- 2 )2 ιι - 4// - μ • Jμ - μ • 1 - Ι/ z2 - ΖΙ - ΖΙ - ΖΙ ΖΙ - ΖΙ - ΖΙ β) Λόγω

της (ίίί) έχουμε: 36 1 8 ( 3 )6 16 1 0. z2 = z1 = z 1 = = + ι

2 1 9 _ 2z.9+ Ι _ _ 29+2 _ _ 22 . 29 _ _ 22 . 1 _

2 - 2 - I - I 1 - I -

2 1 J3 ο

= -z = - + - ι I 2 2

Θέμα 2" Δίνεται ο μιγαδικός αριθμός z = χ + yi,

R , ( )

2z-i:Z ..ι. ι x,y Ε και εστω f z = __ , z .,... .

z-ι α) Αποδείξτε ότι f {l - i) = 3 + 3ί. β) Αποδείξτε ότι ο αριθμός ( f (ι - i)}

2004 είναι

πραγματικός αριθμός.

γ) Έστω Α, Β οι εικόνες των μιγαδικών αριθμών f (ι - i) και f (l+i) στο μιγαδικό

επίπεδο. Αποδείξτε ότι το τρίγωνο ΟΑΒ είναι

ορθογώνιο στο Ο . (Ο η αρχή των αξόνων)

Λύση :

) ( "

) 2 ( 1 - i) - i (l + i) 3 - 3i ο α f 1 - ι = = -- = 3 + 3ι

1 - i - 1 -i

β) (f (l - i ))2004 = (3 + 3i )2004 = (3 (1 + i ))2004 =

32004 [( 1 + i )2Γ02 = 32004 (2i )I 002 = -21 002 ο 32004 Ε R

( " ) 2 (1 + i ) - i (l - i ) γ) f 1 + ι = = 1 + i - 1 2 + 2i - i + i2 = 1 -:- ί = 1 - ί ι

ΕΥΚΛΕΙΔΗΣ Β ' τ. Ι /63

Page 66: Ευκλειδης Β 61

Μαθηματικά για την Γ Λυκείου

άρα Α (3 , 3) , Β ( 1 , - 1 ) είναι ΟΑ · ΟΒ = (3, 3 ) ( 1 , - 1 ) = 3 - 3 = 0 άρα το τρίγωνο ΟΑΒ έχει Ο = 90° .

Θέμα 3" Δίνεται ο μιγαδικός αριθμός z και έστω η συνάρτηση f (v) = iv Χ z , ν ε Ν*.

α) Αποδείξτε ότι: f (I)+f (6)+f (7)+f (16)=0

β) Αποδείξτε ότι: f(v) + f(v + 2) + f (v + 4) + f (v + 6) = Ο

γ) Αν Ιz Ι = 2 aποδείξτε ότι l f (2001)+f (2004�=2ν'2.

Λύση : α) f (1 ) + f (6) + f (7) + f (1 6) =

= i · z + i6 · z + i7 · z + i 1 6 · z = = iz + i 2z + i3z + z = = iz - z - iz + z = Ο

β) f (ν) + f (ν + 2) + f (ν + 4) + f (ν + 6) = ί ν · z + i v+2 · z + iv+4 · z + iv+6 · Ζ = i vz - ivz + i vz - i vz = O

γ) jf(2001) + f(2004) j = J i2001z + i2004zJ = l iz + zl = jz( 1 + i) j = lzlfi = 2fi

Θέμα 4" Δίνεται ο μιγαδικός αριθμός z = χ + yi, x,y Ε R

α) Αποδείξτε ότι αν l z l 2 - (Im (z) + t)2 + 1 = 0 ,

τότε οι εικόνες του z στο μιγαδικό επίπεδο

βρίσκονται στην παραβολή y = .!. χ2 • 2

β) Από τους μιγαδικούς αριθμούς z του (α)

ερωτήματος να βρεθούν αυτοί που έχουν μέτρο

JS . γ) Αποδείξτε ότι για κάθε θετικό πραγματικό

αριθμό ρ υπάρχουν πάντα δυο μιγαδικοί

αριθμοί z, που ικανοποιούν το (α) ερώτημα

τέτοιοι ώστε να ισχύει Ιz l = ρ.

Λύση : α) l z l 2 - ( Im (z) + 1)2 + 1 = 0 {::} {:} (�x2 + y2 ) 2 - (y + 1)2 + 1 = 0 {::} {::} χ2 + y2 - y2 - 2y - 1 + 1 = ο {::} Χ2 - 2y = ο {::}

1 {::} Υ = - χ 2 (I) 2 β) Η δοθείσα σχέση γράφεται: (J8)

2 - (y + l)2 + 1 = 0 {:}8- (y + l)2 + 1 = 0 {::} {:} (y + l)2 = 9 {:} y+ l = ±3 {:} {::} y = 2 ή y = -4 (απορρίπτεται) βρίσκουμε z1 = 2 + 2i, z2 = -2 + 2i γ) ρ2 -(y+l)2 + 1 =0{:}ρ2 -1 -2y- 1+ 1 =0{:}

y2 + 2y - ρ2 = 0 Δ = 4 + 4ρ2 > Ο και Υι · Υ2 = -ρ2 < Ο

άρα το τριώνυμο έχει δυο ρίζες ετερόσημες από τις οποίες δεχόμαστε λόγω της (I) μόνο τη θετική, η οποία μας δίνει δυο τιμές για το χ.

Θέμα 5" α) Να περιγράψετε γεωμετρικά το σύν,ολο (Σ) των εικόνων των μιγαδικών αριθμών z που ικανοποιούν τις σχέσεις: Ιz l = 2 και Re(z) > O .

β) Αν οι εικόνες του z ανήκουν στο σύνολο (Σ) να βρεθεί ο γεωμετρικό τόπος των εικόνων του

μιγαδικού w = z - 4 + 3ί • γ) Να βρεθεί ο μιγαδικός αριθμός w με το

ελάχιστο μέτρο.

Λύση: α) Οι σχέσεις l zl = 2 και Re(z) 2:: Ο ορίζουν το ημικύκλιο με διάμετρο το τμήμα του άξονα y 'y με άκρα τα σημεία Β(Ο,2) και Β '(Ο, -2) που διέρχεται από το σημείο Α (2, Ο) του άξονα χ'χ. β) Έστω w = α + βi και z = x + yi τότε w = z - 4 + 3i {::} z = w + 4 - 3i {::} {::} χ + yi = (α + 4) + (β - 3) i ισχύει όμως Ο � χ � 2 και -2 � y � 2 άρα : Ο � α + 4 � 2 {::} -4 � α � -2 και

ΕΥΚΛΕΙΔΗΣ Β' τ. Ι/64

Page 67: Ευκλειδης Β 61

Μαθηματικά για την Γ Λυκείου

Δηλαδή οι εικόνες του W κινούνται σε κύκλο με κέντρο Κ ( -4, 3) και ακτίνα ρ = 2 . Επειδή όμως -4 :::; α :::; -2 και Ι :::; β :::; 5 ο γεωμ. τόπος των εικόνων του w είναι το δεξί ημικύκλιο του παραπάνω κύκλου με διάμετρο στην ευθεία

χ = - 4. γ) Η ΚΟ έχει εξίσωση y = λχ και επειδή διέρχεται από το Κ ( -4, 3 ) επαληθεύεται από αυτό, δηλαδή 3 = λ( -4) {:} λ = -� . 4

Λύνοντας το σύστημα 4 jy = -�χ, - 4 :::; χ :::; -2

(χ + 4)2 + (y - 3)2 = 4 προκύπτει το ζητούμενο.

Θέμα 6" Δίνονται οι μιγαδικοί αριθμοί z και w με

εικόνες τα σημεία Α και Β αντιστοίχως στο

μιγαδικό επίπεδο.

α) Αποδείξτε ότι Re( z · w) = ΟΑ · ΟΒ .

(Ο η αρχή των αξόνων) .

β) Ν α βρεθεί η γωνία που σχηματίζει με τον

άξονα χ ' χ η ευθεία που ορίζεται από τις εικόνες

των μιγαδικών αριθμών z-1, z-i.

γ) Αποδείξτε ότι οι εικόνες των μιγαδικών

αριθμών z, z-1 , z-i στο μιγαδικό επίπεδο,

σχηματίζουν ορθογώνιο τρίγωνο . .

Λύση : α) Έστω z = χ + yi και w = α + βi τότε

Α(χ, y) και Β(α, β) οπότε ΟΑ · ΟΒ = (χ, y) · (α, β) = αχ + βy ( Ι )

z . w = (χ + yi ) (α - βi) = αχ - βχi + αyi + βy = = (αχ + βy) + (αy - βχ ) i ( 11 ) ό (I) και (II) προκύπτει Re ( z · w) = ΟΑ · ΟΒ .

Απ

β) z - Ι = (χ - Ι ) + yi άρα η εικόνα του είναι το σημείο Γ ( χ - l , y) ο

z - i = χ + (y - I ) i άρα η εικόνα του είναι το σημείο Δ (χ, y - 1 ) .

y - I - y λrΔ = = - Ι επομένως η ΓΔ σχηματίζει χ - χ + Ι με τον άξονα χ' χ γωνία 1 35°.

γ) A (x , y) , Γ(χ - I, y) , Δ (χ, y - Ι ) παρατηρούμε ότι ΑΓ // χ' χ και ΑΔ // y 'y άρα ΑΓ ..l ΑΔ επομένως το τρίγωνο ΑΓ Δ είναι ορθογώνιο στο Α.

Θέμα 7° Θεωρούμε τους μιγαδικούς αριθμούς z για τους

οποίους ισχύει: Ζ6 = (z + 1)6 =1 . α) Αποδείξτε ότι:

ί) z = ! και ίί) Z2 + z +1= 0 . (I) z β) Ένα τρίγωνο έχει κορυφές τις εικόνες των

ριζών της εξίσωσης (Ι) και την εικόνα του

μιγαδικού αριθμού Ζ3 = 1 .

Να βρεθεί το είδος του τριγώνου ως προς τις

πλευρές του.

Λύση :

α) i) z6 = Ι άρα l zl6 = Ι {:} l zl 2 = I {:}

- Ι {:} Ζ · Ζ = Ι {:} Ζ = =. z ii) (z + l)6 = I άρα l z + I I 6 = I {:} Iz + l i 2 = I {:} {:} ( z + Ι ) (� + Ι ) = Ι {:}

ΕΥΚΛΕΙΔΗΣ Β ' τ. Ι/65

Page 68: Ευκλειδης Β 61

Μαθηματικά για την Γ Λυκείου

- - -{::} z · z + z + z + 1 = 1 {::} 1 + z+ z = 0 {:}

1 z + z2 + 1 {::} 1 + z +- = Ο {::} = Ο {::} z2 + z + 1 = Ο. z z β) Οι ρίζες της εξίσωσης z2 + z + 1 = Ο είναι z = _ _!_ + i vf3 z = _ _!_- i vf3 άρα

I 2 2 ' 2 2 2 ' οι

κορυφές του τριγώνου θα είναι τα σημεία

Α (1 , 0) , Β[-_!_ vf3] 2 ' 2 ' r[-..!.. -vl3] 2 ' 2 '

βρίσκουμε ότι: (ΑΒ) = (ΑΓ) = (ΒΓ) = J3 άρα το τρίγωνο είναι ισόπλευρο.

Θέμα 8"

Θεωρούμε τους μιγαδικούς αριθμούς z1, z2 για

τους οποίους ισχύει:

I Ζ2 I = ι και I Ζι - Ζ2 I = I Ζι I . α) Αποδείξτε ότι Re(z1 • Ζ2 ) = .!.

2 β) Να προσδιοριστεί ο θετικός πραγματικός

αριθμός λ, για τον οποίο ισχύει: Ζ2 = Ζ1 ( 1 - 2λί) .

γ) Για λ = .!. aποδείξτε ότι l z ι l = J2 . 2 2

δ) Αν Α η εικόνα του z1 και Β η εικόνα του z2 στο μιγαδικό επίπεδο, aποδείξτε ότι το τρίγωνο

ΟΑΒ είναι ορθογώνιο στο Α και ισοσκελές .

( Ο η αρχή των αξόνων).

Λύση :

α) lz ι - Zz l = lz ι l {::? l z ι - Zz l 2 = lzι l 2 {::?

{::? (z1 - z2 ) (� - Ζ2 ) = Ζ1 � {::?

{::? z1 z1 - z1 z2 - Ζ2 Ζ1 + z2 z2 = z1 z1 {::? z1 z2 + z1z2 = 1

{::? 2Re( z1 z2 ) = 1 {::? Re( Ζ1 z2 ) =� β) z2 = Ζ1 ( 1 - 2λί ) {::? Ζ2 = Ζ1 - 2z1 • λί {::?

{::? Ζ1 - z2 = 2z 1 • λί άρα lz 1 - z2 1 = l2z1 λi l {::? l z ι l = 2 lz ι l lλi l {::? l λl = _!_ 2 άρα λ =� , (λ > Ο) .

γ) Ζ2 = z1 ( 1 - 2λί ) για λ = _!_ έχουμε 2

δ) l z ι - Z2 j = l z ι I = .fi και επειδή το μέτρο της 2

διαφοράς δύο μιγαδικών αριθμών είναι η ' ' θ ' (ΑΒ) .fi αποσταση των εικονων τους α ειναι = 2 .

Είναι (ΟΑ) = J2 και (ΟΒ) = 1 . 2 Με το Πυθαγόρειο Θεώρημα αποδεικνύουμε ότι το τρίγωνο είναι ορθογώνιο στο Α και επειδή (ΟΑ) = ( ΑΒ) είναι ισοσκελές.

Θέματα για λύση Θέμα 1 " α) Να περιγράψετε γεωμετρικά το σύνολο (Σ) των εικόνων των μιγαδικών αριθμών z που ικανοποιούν τις σχέσεις: Ιz l = 3 και Im ( z) 2: Ο .

β) Να αποδείξτε ότι αν η εικόνα του μιγαδικού αριθμού z ανήκει στο σύνολο (Σ) τότε η εικόνα του μιγαδικού αριθμού ω= Μ z +�) κινείται σε

ευθύγραμμο τμήμα το οποίο βρίσκεται στον άξονα n'.

Θέμα 2" Δίνονται οι μιγαδικοί αριθμοί z = α + βί όπου α,β

-ε:R και ω = 2z + iz - 3 , όπου z ο συζυγής του z. α) Αποδείξτε ότι Re( ω) = 2α + β - 3 και Ιm(ω) = α + 2β . β) Να αποδείξετε ότι αν οι εικόνες του ω στο μιγαδικό επίπεδο κινούνται στην ευθεία με εξίσωση y = 3χ + 2, τότε οι εικόνες του z κινούνται στην ευθεία με εξίσωση y = - 5χ + 7. γ) Να βρείτε ποιος από τους μιγαδικούς αριθμούς z οι εικόνες των οποίων κινούνται στην ευθεία y = - 5χ + 7 έχει ελάχιστο μέτρο.

Θέμα 3°

α) Να περιγράψετε γεωμετρικά το σύνολο των

ΕΥΚΛΕΙΔΗΣ Β' τ. Ι /66

Page 69: Ευκλειδης Β 61

Μαθηματικά για την Γ Λυκείου

εικόνων των μιγαδικών αριθμών z που ικανοποιούν τη σχέση Ιz - 3 - i Ι � 2 . β) Αποδείξτε ότι :

z - (3 + i) - (3 - i ) = z - 6 . γ) Αποδείξτε ότι : lz - 61 ::; 2 + JlO δ) Α ν Ζ1 , Ζ2 είναι δυο μιγαδικοί που ικανοποιούν τη συνθήκη του ερωτήματος (α) να αποδείξετε ότι l z ι - z2 1 ::; 4 .

Θέμα 4" Για τους μιγαδικούς αριθμούς z και w ισχύουν: z� + z + z = 3 , w = λ - (λ + Ι ) i , λ ε R και Re(z) � Ο .

α) Αποδείξτε ότι οι εικόνες του z στο μιγαδικό επίπεδο βρίσκονται σε κύκλο (C) με κέντρο Κ(- Ι , Ο) και ακτίνα ρ = 2 . β) Αποδείξτε ότι οι εικόνες του w στο μιγαδικό επίπεδο βρίσκονται στην ευθεία (ε) : χ + y + Ι = Ο. γ) Αποδείξτε ότι η ευθεία (ε) τέμνει τον κύκλο (C) σε δυο αντιδιαμετρικά σημεία. δ) Να βρεθεί ο μιγαδικός z που έχει το μέγιστο μέτρο. ε) Να βρεθεί η εικόνα Μ του μιγαδικού w που έχει το ελάχιστο μέτρο.

Θέμα 5" Δίνεται ο μιγαδικός αριθμός #Ο και η συνάρτηση f (ν) = ( iv - I ) · z , ν ΕΝ* . α) Ν α δείξετε ότι για κάθε ν Ε Ν* ισχύει:

f ( ν) · f (ν + Ι ) · f ( ν + 2) · f (ν + 3) = Ο . β) Αν ισχύει f ( 5 ) = -3 + ί , δείξτε ότι z = 2 + ί . γ) Αν z = 2 + ί . Αποδείξτε ότι

ιr (ν + 3) - f (ν + Ι )I = 2J5 ' για κάθε ν Ε Ν* .

Θέμα 6" 'Ε δ ' θ ' z στω ο μιγα ικος αρι μος z και W = 2 l z +

α) Να αποδείξετε ότι αν ο w είναι πραγματικός αριθμός, τότε ο z είναι πραγματικός ή Ιzl = Ι .

β) Να λύσετε στο σύνολο των μιγαδικών αριθμών, ξ' z .fj την ε ισωση : z2 + Ι = 3

γ) Αν z 1 , z2 είναι οι ρίζες της εξίσωσης του ερωτήματος (β) να υπολογίσετε την τιμή της

(z · z )2 - i , Κ ι 2 παραστασης : = 2006 • •

4 + (z1 + z2 ) ι

Θέμα 7" Δίνονται οι μιγαδικοί αριθμοί z ι , z2, Ζ3 με I z ι l = I Ζ2 I = I Ζ3 I = 3 .

9 α) Αποδείξτε ότι z1 = - . Ζι

β) Αποδείξτε την ισοδυναμία z Ε R {:::} z = z και στη συνέχεια δείξτε ότι ο αριθμός � + � είναι

πραγματικός. Ζ2 z,

1 γ) Αποδείξτε ότι Ι z, + z, + z, Ι =3 Ιz,z, + z,z, + z,z, Ι .

δ) Αν I z , Ι = Ι z, Ι = Ι z, - z, Ι αποδείξτε ότι Re(z, z, ) = � .

Θέμα 8" Θεωρούμε τους μιγαδικούς αριθμούς z f:. Ο, για τους οποίους ισχύει: lzl = I I - zl = ι.;ι (I) .

-α) Αποδείξτε ότι z + z = Ι . β) Αποδείξτε ότι οι μιγαδικοί αριθμοί που ικανοποιούν τις ισότητες (Ι) είναι οι

Ι . .fj Ι . .fj z = - + ι- z = - - ι- . 1 2 2 ' 2 2 2 γ) Αν Α, Β οι εικόνες των z1 , z2 αντιστοίχως στο μιγαδικό επίπεδο, να υπολογίσετε τη κυρτή --γωνία ΑΟΒ , ( Ο η αρχή των αξόνων).

δ) Αποδείξτε ότι οι αριθμός ω = ( Ζι + z2 )v είναι z� + z� πραγματικός αριθμός.

Σχόλιο : Σχετικά με το 3° θέμα των πανελλαδικών εξετάσεων 2006.

ΕΥΚΛΕΙΔΗΣ Β ' τ.Ι /67

Page 70: Ευκλειδης Β 61

Μαθηματικά για την Γ Λυκείου

Θέμα 3" ( αίί ) Από την υπόθεση οι εικόνες των Ζ1 , Ζ2 , z3 Άρα υπάρχει λάθος στις ενδεικτικές λύσεις της βρίσκονται στον κύκλο με κέντρο Ο και ακτίνα ρ= 1 . Από το αί οι αποστάσεις των εικόνων τους είναι ίσες άρα σχηματίζουν ισόπλευρο τρίγωνο. Δηλ .το · ιz ι - z2 j είναι πλευρά ισόπλευρου τριγώνου εγγεγραμμένου σε κύκλο, άρα lzι - z2 1 = R J3 '* lzι - z2 1 = J3 '* lzι - z2 12 = 3 . (γιατί δίνεται το παραπλανητικό ερώτημα lzι - z2 12 ::; 4 ;)

ΚΕΕΕΛ. Πιστεύουμε ότι το θέμα θα έπρεπε να διατυπωθεί ως εξής: Δίνονται οι μιγαδικοί αριθμοί z1 , zυ z3 για τους οποίους ισχύουν:

jz 1 j = jz2 j = jz3 j = 1 και z1 + z2 + z3 = 0 . Αποδείξτε ότι: α) lzι + z2 j 2 + lzι - z2 l 2 = 2 lz ι l2 + 2 jz2 1 2 β) jzι - z2 l = lz ι - z3 1 = Ιz2 - z3 1

1 I 1 2 ( -) 1 Ομοίως ενώ το Re ( z 1 z2 ) = -2 γιατί δίνεται γ) Ζι - Ζ2 = 3 και Re Ζι z2 = -2 ·

Re ( z1 z2 ) � - 1 ; Από τα παραπάνω προκύπτει ότι : Το 3° θέμα από πλευράς αξιολόγησης είναι εσφαλμένο διότι: 1 ") Προτάσσει το δύσκολο ερώτημα, το οποίο μάλιστα αν δεν λυθεί γεωμετρικά απαιτεί συγκεκριμένη αντικατάσταση . 2") Το ερώτημα ( αίί ) χωρίς να έχει σφάλμα υπό στενή μαθηματική έννοια, είναι παραπλανητικό για τους μαθητές. 3") Στο τελευταίο ερώτημα δημιουργεί σύγχυση ο όρος γεωμ, τόπος, γιατί από την εκφώνηση η σχέση jz1 j = jz2 j = jz3 j = Ι συνδέεται με το και

δ) Οι εικόνες των μιγαδικών αριθμών z1 , Ζυ z3 στο μιγαδικό επίπεδο σχηματίζουν ισόπλευρο τρίγωνο και βρίσκονται σε κύκλο του οποίου να βρείτε την εξίσωση . (το πρώτο ερώτημα είναι η άσκηση 9 του σχολικού βιβλίου σελ 1 Ο 1 και βοηθά στην επίλυση του Β. Δεν καταλαβαίνουμε γιατί η επιτροπή παίρνει με μεγάλη ευκολία μια άσκηση από εξωσχολικά βιβλία (και μάλιστα με τα ίδια νούμερα) και δεν αξιοποιεί το σχολικό βιβλίο. Από πλευράς αξιολόγησης είναι απαράδεκτο.)

Μαθηματικά Γενικής Παιδείας

Ανάλυση Του Γιώργου Κοτσιφάκη

1. Να βρείτε το σύνολο ορισμού των Λύση : συναρτήσεων: α) Η συνάρτηση f ορίζεται για τους αριθμούς

α) r (x) = � + JZ - x x + t

χ ε IR . για τους οποίους ισχύουν:

χ3 - 1 � ο {χ3 � 1 {χ � 1 x + l :;t: O <::::> x + 1 :;t: O <::::> x :;t: -1 2 - χ � 0 (2 - χ) ( χ + 1 ) � 0 (x - 2) ( x + 1 ) :s; O χ + 1

ΕΥΚΛΕΙΔΗΣ Β' τ.Ι/68

Page 71: Ευκλειδης Β 61

Μαθηματικά για την Γ Λυκείου

<::::> {χ ;::: 1 <::::> 1 ::; χ ::; 2 . -1 ::; χ ::; 2 Άρα το σύνολο ορισμού της f είναι: A=[ l ,2] .

β) Η συνάρτηση g ορίζεται για τους αριθμούς ΧΕ JR , για τους οποίους ισχύουν: {χ2 - ι ;::: ο {χ2 ;::: ι {1x l ;::: 1 <=> 6+ χ - χ2 > 0 <=> χ2 - χ - 6 < 0 <=> -2 < χ < 3

<::::> {χ ;::: 1 ή χ ::; -ι -2 < χ < 3

- 00 -2 - ι ι 2 3 +οο

___ ?_���J----�1:=:1==-J��:-

Συμπεραίνουμε ότι το σύνολο ορισμού της g είναι: Β=(-2,-ι ]U [ ι ,3) .

2. Έστω η συνάρτηση : f(x)=ιn(2x-e). α) Ο αριθμός 1 είναι τιμή της f; β) Η γραφική παράσταση της f τέμνει τον

άξονα χ 'χ; γ) Η γραφική παράσταση της f δεν

διέρχεται από το σημείο M(2e,ιn3);

Λύση . Η συνάρτηση f ορίζεται για τους αριθμούς

χ Ε R . για τους οποίους ισχύει: 2χ - e > Ο <::::> χ > � 2 Άρα το σύνολο ορισμού της f είναι:

Α= (� , + οο) . α) Ο αριθμός 1 είναι τιμή της f αν, και μόνο αν,

υπάρχει χΕΑ με: f(x)= 1 <::::> ιη(2χ - e)=ιne <::::> 2χ - e=e <::::> x=e Η τιμή αυτή του χ είναι δεκτή γιατί e Ε Α .

Άρα το 1 είναι τιμή της f.

β) Η γραφική παράσταση της f τέμνει τον άξονα χ 'χ αν, και μόνο αν, υπάρχει χΕΑ με:

e + ι f(x)=O <::::> Ιη(2χ--e)=Ιη ι <::::> 2x--e= l <::::> χ= --2 Η τιμή αυτή του χ είναι δεκτή, γιατί ανήκει

στο Α. Άρα, η γραφική παράσταση της f τέμνει τον ' ξ ' ' ( e + 1 ο) α ονα χ χ στο σημειο -2- ,

γ) Έχουμε: f(2e) = ln( 4e - e) = ln(3e) = ln3+lne = ι +ln3 * ln3

Άρα η γραφική παράσταση της f δεν διέρχεται από το σημείο Μ.

3. Το βάρος της φέτας (τυρί) σε κιλά που πουλάει ένα νέο σούπερ μάρκετ συναρτήσει των ημερών λειτουργίας του t είναι:

r ( t) = 3o(2t - ι + -1-)

t + 2 Να βρείτε πόσα κιλά φέτα πούλησε: α) Τις 8 πρώτες μέρες β) τη 2η και την 3η μέρα μαζί γ) την 4η μέρα.

Λύση α) Η φέτα που πούλησε τις 8 πρώτες μέρες είναι:

f {8) = 30(2 · 8 - 1 +-ι-) = 3ο( ι 5 +_!_) = 453 κιλά 8+ 2 1 0

β) Η φέτα που πούλησε τη 2η και την 3η μέρα μαζί είναι:

f {3) -f ( I) = 30( 6- ι +* )-30( 2- ι +i) = ι ι 6 κιλά

γ) Η φέτα που πούλησε την 4η μέρα είναι: f { 4) -f (3) = 30( 8- ι +i )-30( 6- ι +*) = 59 κιλά

4. Έστω η συνάρτηση :

r (x) = χ2 - 5χ + 4 Fx - 2

α) Να βρείτε το πεδίο ορισμού της f. β) Να βρείτε το όριο: L = lim f{ χ) Χ--+4

Λύση : α) Η συνάρτηση f ορίζεται για τους αριθμούς

x E R, για τους οποίους ισχύουν: {χ ;::: ο {χ ;::: ο {χ ;::: ο Γχ -2 :�; 0 <=> Γχ :�; 2

<::::> χ :�; 4 Άρα, το σύνολο ορισμού της f είναι:

Α=[Ο,4) υ (4,+=) β) Έχουμε:

. χ2 -5χ +4 . (χ2 -5χ +4) (Γχ + 2) L = lιm lιm---:---=-.,......,--=--------,---'-

x --+4 fx - 2 χ--+4 (Γχ -2) (Γχ + 2)

= lim (χ - t) ( χ -4) { fx +2) = lim[( x- I) {fx +2)] χ->4 χ -4 χ ->4

= 3 · 4 = 1 2

5. Να βρείτε τα παρακάτω όρια:

L ι· χ2 - 7χ + 6 L ι

· J4+h - 2 I = xl� Χ2 - 9Χ + 8 2 = hl.!!: h

ΕΥΚΛΕΙΔΗΣ Β ' τ. Ι /69

Page 72: Ευκλειδης Β 61

Μαθηματικά για την Γ Λυκείου

χ - 3 L3 = lim � >--+3 ν χ + 6 - χ

. r (t + h) - r(ι) L4 = lιm ,

h--+0 h χ - 3 Όπου f {x) = ­x + l

Λύση Έχουμε:

. χ2 - 7χ + 6 . (χ - 1) {χ - 6) • ι = lιm = lιm =

I χ-+ Ι Χ 2 - 9Χ + 8 χ-+ Ι (Χ - 1 ) (Χ - 8)

. χ - 6 1 - 6 5 lιm-- =- =- . χ-+ Ι χ - 8 1 - 8 7

ι. �4 + h - 2 ι = ιm = 2 h--->0 h

. (�4 + h - 2) {�4 + h + 2) = lιm = h--+O h {�4 + h + 2)

= lim 4 + h - 4 = lim 1 = h--+O h {�4 + h + 2) h--->0 �4 + h + 2 4

χ - 3 ι3 = lim = Χ-+3 �χ + 6 - Χ

. ( χ - 3) {�χ + 6 + χ ) = lιm = χ--+3 (�χ + 6 - χ) ( �χ + 6 + χ)

. (χ -3) {"'χ + 6 + χ) . (χ - 3)("'χ + 6 + χ) = Ιιm = lιm---'-------:---'-..... 3 χ + 6-χ2 χ-+3 -(χ - 3)(χ + 2)

= lim Γχ+6 + χ = -� χ-+3 -(χ + 2) 5 l + h - 3 1 . f ( 1 + h ) - f ( 1 ) 1. 1 + h + 1 + ι4 = lιm = ιm��--"-----h-+0 h h-+0 h

h - 2 + 1 = lim h + 2 = lim 2h = lim-2- = 1 h-+0 h h-+O h (h + 2) h--->0 h + 2

6. Μια συνάρτηση f είναι συνεχής στο χσ=2 και ισχύει:

r ( χ) - 2 lim = 5 (1) Χ-+2 Χ - 2

α) Να βρείτε τον αριθμό f(2). β) Να βρείτε το όριο :

Λύση. α) Επειδή η f είναι συνεχής στο χο=2, έχουμε:

lim f (x ) = f (2) . χ-->2 Από την ( 1 ) και επειδή lim (χ - 2) = Ο , χ-->2

έχουμε: lim f ( χ ) - 2 (χ - 2) = 5 · Ο => lim[f ( χ ) - 2] = Ο Χ-+2 Χ - 2 Χ-+2

και επειδή lim 2 = 2 , έχουμε: χ-+2 lim[f ( x ) - 2 + 2] = 0 + 2 :::::> lim f (x ) = 2 :::::> f (2) = 2 Χ--->2 Χ-+2

β) Επειδή f(2)=2, έχουμε: . χ 2 - 2χ - 8 . ( χ - 4) ( χ + 2) L = 1ιm = lιm�-�-.:...,-

..... 4 χ (χ 2 - 1 6) χ-+4 χ ( χ - 4 ) ( χ + 4)

= lim χ + 2 = � = � . χ-+4 χ (χ - 4) 32 1 6

7. Έστω η συνάρτηση :

f {x) = � χ + 2

α) Με τον ορισμό της παραγώγου, να βρείτε την f' (2).

β) Να βρείτε την εξίσωση της εφαπτομένης ε της γραφικής παράστασης της f στο σημείο της Α(2, f(2)).

Λύση . α) Έχουμε:

2(2 + h) 2 · 2 f(2) = 1im f(2 + h) - f(2) = lim 2 + h + 2 - 2+2 = h-+0 h h-+0 h

4 + 2h - 1 = lim 4 + h h->0 h

. h 1 . ι ι = lιm = ιm-- =- . h-+O h (4 + h) h-+0 4 + h 4

β) Η εφαπτομένη ε της γραφικής παράστασης της f στο σημείο Α(2, f(2)) έχει συντελεστή διεύθυνσης τον αριθμό f'(2)= ..!_ . 4

Έτσι, η εξίσωση της ε είναι της μορφής: y = � χ + β , όπου β ε JR . Και επειδή η ε διέρχεται από το σημείο Α(2, f(2))=A(2, 1 ), έχουμε:

ΕΥΚΛΕΙΔΗΣ Β' τ.l/70

Page 73: Ευκλειδης Β 61

Μαθηματικά για την Γ Λυκείου

Ι Ι Ι Ι = -2 + β � β = Ι - - � β = - . 4 2 2 Άρα η εξίσωση της εφαπτομένης ε είναι:

Ι Ι y = - x +- . 4 2

8. Να βρείτε τις παραγώγους των παρακάτω συναρτήσεων:

1 . f(x)=xlnx 2.

3. f(χ)=ημ3χ+eημχ 4.

χ - 1 f {χ) = -χ2_+_χ_+_1

5. f(x)=(x3-x2+1)5

6. f {x) = �1 + .,Γχ, χ > Ο

Λύση. Έχουμε:

f(χ)=συν23χ

Ι . f' (x)=(x) Ίnx + x(lnx) ' = lnx+x ·_!_ = lnx+ I χ

, ( χ - Ι ) ' ( χ 2 + χ + Ι ) - (χ - Ι ) (χ2 + χ + Ι ) ' 2. f ( χ ) = 2 (χ2 + χ + Ι ) x2 + x + l - (x - I ) (2x + I ) -χ2 + 2χ + 2

(xz + χ + Ιγ := (xz + x + I )z.

3 . f'(x) = (ημ3χ) '+(eημχ) '= = (συν3χ)(3χ) '+ eημ\ημχ) '=3συν3χ+ eημχσυνχ.

4. f'(χ)=2(συν3χ) · (συν3χ) '=2(συν3χ) · (-ημ3χ) · (3χ) '= -6 · συν3χημ3χ = -3ημ6χ

5. f'(x)=5(x3-x2+ I )4 • (χ3-χ2+ Ι ) '= = 5(χ3 -xz+ Ι )4 . ( 3xz -2χ)=5χ(χ3 -xz+ Ι )4( 3χ-2)

6. f (x ) = Ι { I + .J�} = Ι ·-Ι- = 2�Ι + -Γχ 2�Ι + -Γχ 2-Γχ Ι = ----;===:=

4�χ + χ-Γχ.

9. Να λυθεί η εξίσωση : 7 · Γ(χ) - f'(x) = 7 · f' "(x) - f(x)+ 4e (1)

όπου

. \ ίι σ η . Μ ε χ Ε � , έχουμε:

f(x ) = _!_(ex + e-x ) ' = _!_( ex - e-x ) . 4 4

f " ( χ) = ± ( ex - e-x ) ' = ±( ex + e-x ) = f ( Χ ) .

f"(3) = _!_(ex + e-x ) ' = _!_(ex - e-x ) = f(x ) . 4 4 Έτσι έχουμε: ( Ι ) �7 · f(x) - f(x) = 7 · f(x) - f(x) + 4e

� 8 · f(x)-8f(x)=4e�2(ex+e-x) -2(ex+e-x)=4e � (ex+e-x) - (ex+e-x) = 2e � e-x = e � -χ = Ι � χ = -Ι

10. Έστω η συνάρτηση : f(x) = 3συν2χ+4ημ2χ.

Να δείξετε ότι η εξίσωση : f"(x) + 4 f(x) + συν2χ = J2 , είναι αδύνατη.

Λύση : Με χ Ε JR , έχουμε:

• f(x) = 3(-ημ2χ)-(2χ) '+ 4(συν2χ) · (2χ) '= = -6ημ2χ + 8συν2χ

• f" (x)= -6(συν2χ) · (2χ) ' + 8(-ημ2χ) · (2χ) '= = - Ι2 συν2χ - Ι 6ημ2χ Έτσι, η δοσμένη εξίσωση είναι ισοδύναμη με

την εξίσωση : -Ι2συν2χ-16ημ2χ + 4(3συν2χ + 4ημ2χ) + συν2χ= .J2 � συν2χ = .J2 , αδύνατη, γιατί .J2 > Ι . 1 1 . Μια συνάρτηση f είναι ορισμένη και

παραγωγίσιμη στο R και για κάθε χ Ε JR ισχύει: f(x2 - 2χ + 3) = f(2x - 1) (1) Να δείξετε ότι η εφαπτομένη ε της γραφικής παράστασης της f στο σημείο της με τετμημένη 1 είναι κάθετη στον άξονα y 'y.

Λύση. Από την ( Ι ), παραγωγίζοντας, έχουμε για κάθε

χ E lR : f(x2 - 2χ + 3) · (χ2 - 2χ + 3) ' = f(2x - Ι ) ·(2χ - Ι ) '=> (2χ - 2)f(x2 - 2χ + 3)= 2f(2x- I ) (2) Από τη (2) με χ = Ι βρίσκουμε 0=2 f ( Ι ),

οπότε f( l )=O. Συνεπώς η εφαπτομένη ε είναι είναι παράλληλη στον άξονα χ 'χ δηλαδή κάθετη στον άξονα y 'y.

ΕΥΚΛΕΙΔΗΣ Β ' τ.Ι /71

Page 74: Ευκλειδης Β 61

Μαθηματικά για την Γ Λυκείου

12. Θεωρούμε τις συναρτήσεις: f(x) = (α - 2)Ιηχ και g(x) = χ2 + βχ - 3

Να βρείτε τους αριθμούς α,β ε IR , για τους οποίους οι γραφικές παραστάσεις των συναρτήσεων f και g να έχουν κοινή εφαπτομένη στο σημείο με τετμημένη χο=1. ΛίJση . Οι συναρτήσεις f και g είναι ορισμένες και

παραγωγίσιμες στο (0,-too) με: f' ( χ ) = (α - 2)_!_ και g ' ( χ ) = 2χ + β χ

Για να συμβαίνει το ζητούμενο πρέπει και αρκεί (γιατί;) : {f ( I ) = g ( I ) {Ο = β - 2 {β = 2

f' ( I ) = g ' ( I ) � α - 2 = 2 + β � α = 6

Άρα οι ζητούμενες τιμές των α και β είναι: α = 6 και β = 2

13. Η γραφική παράσταση της συνάρτησης: f( χ) = (αχ + β)·eΧ, όπου α, β ε IR , διέρχεται από το σημείο Α(1 , -2e) και η εξίσωση της εφαπτομένης της στο σημείο αυτό είναι:

y = - ex - e.

α) Να βρείτε τους αριθμούς α και β

β) Να βρείτε τα διαστήματα μονοτονίας και τα ακρότατα της f.

Λύση α) Έστω ότι για δυο αριθμούς α,βε JR. οι

υποθέσεις ισχύουν. Η συνάρτηση f είναι ορισμένη και παραγωγίσιμη στο JR. με:

f'(x) = αeχ + (αχ + β)eχ = (αχ + α + β)eχ Από τις υποθέσεις έχουμε: {f( I) = -2e => {(α+β)e = -2e

=> {α+β = -2 => {α = Ι f(I) =--e (2α+β) e =--e 2α+β = -Ι β = -

Α ντιστρόφως. Όπως βρίσκουμε εύκολα, με α = Ι και β = -3

οι υποθέσεις ισχύουν. Άρα, οι ζητούμενοι αριθμοί είναι: α = 1 και β = -3

β) Επειδή α = Ι και β = -3 , έχουμε: f(x) = (x-3)ex και {> Ο, αν χ > 2

f' ( x ) = (x - 2 ) ex = 0, αν χ = 2 < 0, αν χ < 2

Συμπεραίνουμε όtι η f είναι γνησίως φθίνουσα στο διάστημα (-οο, 2] , γνησίως αύξουσα στο [2 , +οο) και παρουσιάζει ελάχιστο στη θέση

χ = 2, ίσο με f(2) = --e2

14. Έστω η συνάρτηση : f(x) = χ2 + 2χ - λlnx, χ>Ο (λ ε IR ) . Η εφαπτομένη ε της γραφικής παράστασης της f στο σημείο της Α(1, f(1 )) είναι παράλληλη στον άξονα χ 'χ.

α) Να δείξετε ότι λ=4

β) Να βρείτε τα διαστήματα μονοτονίας και τα ακρότατα της f.

γ) Να δείξετε ότι για κάθε χ>Ο, ισχύει:

χ2 + 2χ � 3 + 4Ιηχ Λύση α) Η συνάρτηση f είναι ορισμένη και

παραγωγίσιμη στο Α=(Ο, +οο) με: 1 f' ( χ ) = 2χ + 2 - λ- ( 1 ) χ

Ο συντελεστής διεύθυνσης της εφαπτομένης ε είναι: f'( l ) = 2 + 2 - λ = 4 - λ. Έτσι, έχουμε:

ε // χ 'χ � f'( l ) = Ο � 4 - λ = Ο � λ = 4.

β) Επειδή λ = 4, από την ( 1 ) έχουμε στο Α = (0, +οο) :

4 χ2 + χ - 2 Γ( χ ) = 2χ + 2 -- = 2 = χ χ {> Ο, αν χ > 1 χ - Ι χ + 2 = 2 ( )� ) = Ο, αν χ = Ι < Ο, αν Ο < χ < Ι

γ) Από το προηγούμενο ερώτημα συμπεραίνουμε ότι για κάθε χ > Ο, ισχύει:

f(x) � f( l ) ::::> χ2 + 2χ - 4Inx � 3 (αφού λ = 4) ::::> χ2 + 2χ � 3 + 4lnx

ΕΥΚΛΕΙΔΗΣ Β' τ.Ι /72

Page 75: Ευκλειδης Β 61

ΕΘΝ Ι ΚΗ : · ·

Η Π ΡΩΤΗ ΑΣΦΑΛΙΣΥΙΚΗ �/ •

Όταν n ιστορ ία 2 . 500 χρόνων χωράει σ' ένα παράθυρο . . . τότε αλλάζει

όλn n εργασιακn φ ιλοσοφία Είναι σnμ αντικό. όταν εργάζεσαι . ονειρεύεσαι . σχεδιάζεις.

το μ άτι να ελευθερώνεται σ· αυτn τnν εικόνα .

Απ' τn θέσn του νέου μ α ς κτnρίου. έχουμε δικαίω μα

και στον Ιερό βράχο τnς Ακρόπολnς.

Θεωρούμε ότι τίποτα δεν είνα ι τυχαίο και ότι οι λεπτομέρειες

συνθέτουν το σύνολο. Γιατί n εργασιακn φιλοσοφία τnς

Εθνικnς Ασφαλιστικnς. που δ ιαμορφώνεται και από το

περιβάλλον. είναι α υτn που π άνω από έναν αιώνα δnμ ιουργεί

τnν άρρnκτn σχέσn με τους π ελάτες τnς.

Ο Ν ΕΟΣ ΠΑΘΜ ΟΣ ΑΠΟΓΕΙΩΣΗΣ ΤΗΣ ΕΘΝΙΚΗΣ ΑΣΦΑΛΙΠΙ ΚΗΣ ΠΕΓΑΖΕΙ ΣΕ ΕΝΑ ΧΩΡΟ ΟΛΕΣ τJΣ ΥΠΗ Ρ ΕΣΙΕΣ ΤΗΣ.

Λεωφ. Συγγρού 1 03- 1 05, 1 1 7 45 ΑθΗ ΝΑ. Τηλ. Κέντρο: 21 Ο 90 99 000, www.ethniki-astaιistiki.gr

Page 76: Ευκλειδης Β 61

Ο Εuκλεiδnς

προτεiνε•

«Η καρδιά των μαθηματικών είναι τα προβλήματα και οι λύσεις και ο κύριος λόγος ύπαρξης του μαθηματικού είναι να λύνει προβλήματω>.

P. R. HALMOS

Επιμέλεια: Α. Κυριακόπουλος, Γ. Στρατής, Γ. Τριάντος, Ν. Αντωνόπουλος

Λύσεις Προτεινομένων Ασκήσεων

80. Να αποδείξετε ότι για κάθε δοθέντα

ακέραιο αριθμό α, η διοφαντική εξίσωση

( α2 -1 )(β2 -1) = γ2 -1 έχει τουλάχιστον δυο

διακεκριμένες ακέραιες λύσεις ως προς β

και γ.

(Επροτάθη από τον Ακαδημαϊκό κ. Νικόλαο

Αρτεμιάδη)

Λύση (από τον ίδιο)

Από τη γνωστή ταυτότητα

χ(χ+ 1 ) (χ+2) (χ+3) + 1 = (χ2+3χ+ 1 )2

αν θέσουμε χ=α-1 , έχουμε: (α2- 1 ) (α2+2α)=(α2+α+ 1 )2 - 1 ( 1 )

Α ν τώρα στην ( 1 ) θέσουμε β=α+ 1 και α2 +α-1 =γ, τότε α2 + 2α=β2 - 1 και η ( 1 ) γράφεται: (α2- 1 ) (β2- 1 )=γ2-1 απ' όπου προκύπτει ότι ο ι αριθμοί β=α+ 1 και γ=α2+α-1 αποτελούν λύση δοσμένης εξίσωσης. Αν επαναλάβουμε την ίδια διαδικασία με χ=α-2 και κατόπιν β=α- 1 βρίσκουμε και δεύτερη λύση της δοσμένης εξίσωσης, την: β=α- 1 , γ=α2 -α-1 .

Σχόλιο : Είναι φανερό ότι αναζητούμε λύσεις

εξαρτώμενες από τον αριθμό α. Λύσεις έστειλαν επίσης ο κύριος Ρ. Μπόρης

- Δαφνής Αθηνά και ο μηχανικός Α νδρής

Ιωάννης -Αθηνά.

86. Δίνεται τρίγωνο ΑΒΓ και η διχοτόμος

του ΒΔ. Α ν Μ είναι το μέσο της πλευράς ΑΒ

και ΜΔ l. ΒΔ, να αποδείξετε ότι ΑΒ = 3ΒΓ.

(Επροτάθη από το συνάδελφο Μπάμπη Στεργίου - Χαλκίδα)

Λύση (Από το συνάδελφο Σαμπά Θεόδωρο

- Πάτρα) Α

Ρ Από το Μ φέρνουμε παράλληλη στη διχοτόμο ΒΔ και έστω Κ το σημείο τομής της με την ΑΔ.

Τότε ΜΚ =ΒΔ

( 1 ) 2

Α ν η προέκταση του ΜΔ τέμνει την προέκταση της ΒΓ στο σημείο Λ, τότε στο τρίγωνο ΒΛΜ η διχοτόμος ΒΔ της γωνίας ΛΒΜ είναι και ύψος, οπότε το τρίγωνο είναι

ισοσκελές, άρα ΒΛ = ΒΜ = ΑΒ (2) 2

Τέλος αν φέρουμε κάθετη στη ΜΛ στο σημείο Λ και ονομάσουμε Ρ το σημείο τομής της με την προέκταση της πλευράς ΑΒ, τότε τα τρίγωνα ΜΚΔ και ΑΡ Δ είναι ίσα, οπότε

ΕΥΚΛΕΙΔΗΣ Β ' τ.Ι/74

Page 77: Ευκλειδης Β 61

Ο Ευκλείδης προτείνει ... Ευκλείδη ... και Διόφαντο

< ' > ΒΔ ΡΛ = ΜΚ = -

2 Από την ομοιότητα των τριγώνων ΒΓ Δ και

ΛΓΡ , ΒΓ ΒΔ

2 ,

, παιρνουμε: - = - = οποτε ΓΑ ΡΑ

ΒΓ ΒΓ = 2ΓΛ δηλ. ΓΛ = τ (3)

Τελικά, ( 2 ) ( 3 )

ΑΒ = 2ΒΜ = 2ΒΛ = 2(ΒΓ + ΓΛ) =

2 ( ΒΓ + Β;) = 3ΒΓ

που είναι και το ζητούμενο.

2η λύση (Από το συνάδελφο Κυριακόπουλο Θανάση - Αθήνα)

Α

Β

Γ

Θεωρούμε σημείο Λ στην ΑΒ, ώστε ΒΛ=ΒΓ. Τότε το τρίγωνο ΒΛΓ είναι ισοσκελές και επειδή η ΒΔ είναι διχοτόμος της γωνίας Β, ισχύει: ΛΓ .l ΒΔ , οπότε ΛΓ I I ΜΔ.

Άρα ΑΔ

= ΑΜ

= γ I 2

= γ

( 1 ) ΑΓ ΑΛ γ - α 2(γ - α)

Από το θεώρημα της εσωτερικής διχοτόμου στο τρίγωνο ΑΒΔ προκύπτει ότι:

ΑΔ γ - =--ΑΓ α + γ

λ;τό n; ( l ) . (2) παίρνουμε :

(2)

., .. , ·

= -· - ::::> α _._ γ = 2 ( γ - α) ::::> γ = 3α � ( ·�ι - α . α - .,

που είναι και το ζητούμενο.

Λύσεις Επίσης έστειλαν ο χημικός Καρβελάς Δημήτρης - Πεύκη Αττικής οι συνάδελφοι Καραχάλιος Βασίλης Τρίπολη, Κούρτης Χρυσόστομος - Λάρισα, Σταματογιάννης Γιάννης - Δροσιά Αττικής, Μάγκος Αθανάσιος - Κοζάνη, ο κύριος Ρ. Μπορής - Δάφνη Αθήνα και ο μαθητής Σακελλάρης Γιώργος, Μουσικό Λύκειο Βόλου.

90. Έστω μια συνάρτηση f : JR � JR η οποία είναι παραγωγίσιμη στο R, χωρίς να είναι σταθερή σε κάποιο διάστημα, με f(α)=Ο, και f(x0)>0 για κάποιο Χο >α. Αν ισχύουν: f"(x) = p(x)f(x), όπου p(x) συνάρτηση συνεχής μη σταθερή στο R ώστε p(x)>O για κάθε χ ε JR και f'(κ) f(m) = f'(m)f(κ) για κάποια κ,m με α<κ<m, να αποδείξετε ότι υπάρχει ξ ε (κ, m) με την

, � I Ιn f(m) - ln f(κ) l ιδιοτητα ν Ρ(ξ) > .:..__ ____ ___,_

m - κ

(Επροτάθη από τον κ. Ροδόλφο Μπόρη -Δάφνη, Αθήνα)

Λύση (από τον ίδιο) Αρχικά θα αποδείξουμε ότι ο αριθμός α είναι μοναδική λύση της εξίσωσης f(x)=O. Έστω ότι η εξίσωση έχει και δεύτερη ρίζα β και ας είναι β> α.

Τότε επειδή η f δεν είναι σταθερή στο [α,β] , υπάρχει χ ι ε (α,β) ώστε f(x 1 ):;t:O. Χωρίς βλάβη της γενικότητας θεωρούμε f(x 1 )>0. Η f ως συνεχής στο [α,β] παρουσιάζει μέγιστο στο διάστημα αυτό. Α ν γ το σημείο του (α, β) στο οποίο η f παρουσιάζει μέγιστο, τότε προφανώς f( γ )>0. Τότε: f"(γ) = p(γ)f(γ)>Ο, οπότε από το κριτήριο της δεύτερη παραγώγου η μη σταθερή στο [α, β] συνάρτηση παρουσιάζει και ελάχιστο στο γ, που αποκλείεται. Άρα η εξίσωση f(x)=O έχει μοναδική ρίζα τον αριθμό α και δεδομένου ότι f(x0)>0, ισχύει f(x)>O στο (α, +ω), διότι η f διατηρεί σταθερό πρόσημο στο (α, +ω) . Με χ>α έχουμε:

Ε\Κ\ΕΗΗΣ Β ' τ.Ι /75

Page 78: Ευκλειδης Β 61

Ο Ευκλείδης προτείνει ... Ευκλείδη ... και Διόφαντο

(f(x) J . = f '(x) - (f(x) J2 = P(x) - (f(x) J2 => f(x) f(x) f(x) f(x)

=> Ρ( χ) = (f(x) J + ( f(x))2 f(x) f(x)

=> [ P(x)dx = [f(x) ]m + r ( f(x) )2dx = f(x) κ f(x)

= 0 + r ( f(x) J2 dx f(x)

οπότε υπάρχει ξ Ε (κ, m) ώστε να ισχύει:

( m - κ ) Ρ(ξ) = r ( f'(x) J2 dx ( 1 ) f(x) Από την ανισότητα B .C .S . για τα ολοκληρώματα έχουμε:

r (�i:iJ'dx [ Ι'dχ � ( r (�i:iJ' · ldx )' �

(m - κ ) r ( �i:iJ'dx ψn f(x)J: )' �

r ( f'(x) J2 ( 1n f(m) - ln f(κ) )2 => --- dx � �--------�-

f(x) m - κ

Απ' όπου, εξαιτίας της ( Ι ) έχουμε:

(In f(m) - ln f(κ) )2 Ρ( ξ) � 2 (m - κ) =>JP© � ! In f(m) - ln f(κ) l

m-κ

Η ισότητα ισχύει όταν έχουμε «ίσον» στην ανισότητα B.C .S . δηλ. όταν υπάρχει λ Ε R, ώστε για κάθε χ>α να είναι

f(x) = λ <=> f(x) = ceλx c Ε IR που f(x) ' '

αποκλείεται διότι τότε με αντικατάσταση στην f"(x)=p(x)f(x) οδηγούμαστε στο συμπέρασμα ότι η p(x) είναι σταθερή . Επομένως η ισότητα δεν ισχύει ποτέ, πράγμα που σημαίνει ότι

JP© > I In f(m) -ln f(κ) l m-κ

93 . Ν α βρείτε διψήφιο ακέραιο αριθμό, ο οποίος να είναι ίσος με το άθροισμα του κύβου του ψηφίου των δεκάδων του και του τετραγώνου των ψηφίων των μονάδων του.

(Επροτάθη από τους συναδέλφους Τάκη Δρούτσα και Νίκο Πανουσάκη - Αθήνα)

Λύση (από το χημικό Καρβελά Δημήτρη ­Πεύκη Αττικής) Α ν χ είναι το ψηφίο των δεκάδων και το y το ψηφίο των μονάδων του αριθμού, τότε ισχύει

1 0x+y=x3+y2 ( 1 ) με Χ Ε { 1 ,2, . . . ,9} και

y E {0, 1 , . . . ,9} Είναι: ( 1 ) <=>χ( Ι Ο-χ2) = y(y- 1 )

όπου το δεύτερο μέλος είναι άρτιος ως γινόμενο δυο διαδοχικών ακεραίων. Άρα και το πρώτο μέλος πρέπει να είναι άρτιος, οπότε εύκολα βρίσκουμε ότι ο αριθμός χ είναι άρτιος. Εξάλλου y(y-1 )�0, οπότε: 1 0- χ22: Ο => χ Ε { 1 ,2,3 } και δεδομένου ότι είναι άρτιος, συνεπάγεται χ = 2.

Με χ = 2 η ( 1 ) γράφεται

20+y = 8+y2<::>y=4 ή y= -3, οπότε y=4. Επομένως ο ζητούμενος αριθμός είναι ο

24.

Λύσεις έστειλαν επίσης οι συνάδελφοι Κούρτης Χρυσόστομος Λάρισα, Ηλιόπουλος Γιάννης - Καλαμάτα, Μάγκος Αθανάσιος - Κοζάνη, και ο κύριος Ρ. Μπόρης Δάφνη - Αττικής.

94. Α ν για τους πραγματικούς αριθμούς α,β,γ ισχύουν αβγ1:0 και

β2 + γ2 _ α2 γ2 + α2 _ β2 α2 + β2 _ γ2 .:....______:__ __ + + = 1 (1)

2βγ 2αγ 2αβ

να αποδείξετε ότι δυο από τα κλάσματα είναι ίσα με + 1 και το άλλο ίσο με -1 . (Επροτάθη από το συνάδελφο Νίκο Αντωνόπουλο - Ίλιον, Αθήνα)

Λύση (Από το συνάδελφο Σαμπά Θεόδωρο - Πάτρα) Είναι:

( 1 ) <=> α(β2+γ2-α2)+β(γ2+α2-β2)+γ(α2+β2-γ2)

=2αβγ

<::>αβ2+αγ2 -α3+βγ2+βα2 -β3+γα2+γβ2 -γ3 _2αβγ=Ο

ΕΥΚΛΕΙΔΗΣ Β ' τ. Ι/76

Page 79: Ευκλειδης Β 61

Ο ΕυκλΕίδης προτείνει ... ΕυκλΕίδη ... και Διόφαντο

<=> α2(β-α) -β2(β-α)+γ(α2+β2-2αβ)+γ2

<=> -(β-α )2(β2 -αz)+γ(β-α )2 +γz( α+β-γ )=Ο

<=>-(β-α)2(β+α)+γ(β-α)2+ γ2(α+β-γ)=Ο <:::> (β-α)2(γ-β-α)+ γ2(α+β-γ)= Ο

<=> (α+β-γ) [γz-(β-α)z]=Ο

<=> ( α+β-γ )( γ-β+α )( γ+β-α )=Ο <:::>γ=α+β ή β=α+γ ή α=β+γ

και

Α ν γ=α+β τότε :

β2 + (α + β)2 - α2 2β2 + 2αβ _ ____..:. _ ____:... __

- - Ι 2αβ (α + β) - 2β2 + 2αβ - '

(α + β)2 + α2 - β2 2α2 + 2αβ -'----'------ - - Ι 2α ( α + β) - 2α2 + 2αβ - '

α2 + β2 - γ2 - α2 + β2 ( α + β)2 -2αβ = = -- = - Ι . 2αβ 2αβ 2αβ

Στο ίδιο συμπέρασμα καταλήγουμε αν υποθέσουμε ότι ισχύει β=α+γ ή α=β+γ

Λύσεις έστειλαν επίσης οι συνάδελφοι Σταματογιάννης Γιάννης - Δροσιά Αττικής, Μάρκος Αθανάσιος - Κοζάνη και ο κύριος Ρ. Μπορής - Δάφνη Αθήνα.

45 (Τεύχους 54) Δίνεται τυχαίο τρίγωνο ΑΒΓ με ορθόκεντρο Η, τυχαίο σημείο Μ του περιγεγραμμένου κύκλου του και (s) η ευθεία Simson του Μ ως προς το τρίγωνο ΑΒΓ. Να αποδείξετε ότι η (s) είναι κοινή ευθεία Simson τριών σημείων Α ', Β ',Γ ' ως προς τα τρίγωνα ΗΒΓ, ΗΓ Α, ΗΑΒ αντίστοιχα, όπου τα σημεία Α ', Β ',Γ ' είναι κορυφές τριγώνου ίσου προς το τρίγωνο ΑΒΓ και του οποίου το ορθόκεντρο είναι το σημείο Μ.

(Επροτάθη από τον συνάδελφο Γιώργο

Τρι�-το στο τεύχος 54)

Λύση (από τον ίδιο) Κατασκευάζουμε τρίγωνο Α'Β Τ ' συμμετρικό του ΑΒΓ ως προς κέντρο συμμετρίας το μέσον Τ του ΗΜ. Τα τρίγωνα Α'Β 'Γ, ΑΒΓ είναι ΙΣΑ (εκ κατασκευής , με πλευρές ίσες και παράλληλες μία προς μία και τα ομόλογα στοιχεία τους ίσα ένα προς ένα)

Γ. Θωμαlδηc;. Γ. nετράκη<;. Κ. Τουλούμη<;. Μ. Σταφυλiδου ΓΛΩΣΣΑ, ΙΠΟ Ρ Ι Α και ΕΥΚΛ ΕΙΔΕΙΑ ΓΕΩ ΜΗΡΙΑ

Μία δοκιμή διαθεματικής προσέγγισης στη Δευτεροβάθμια Εκπαίδευση

Το βιβλίο καλύmει τηv άμεση αvάγκη τωv Εκπαι· δευτικώv για εvημέρωση σχετικά με τη διαθεματική διδασκαλία. Παρουσιάζονται το υλικό και τα αποτε­λέσματα μιας διετούς πειραματικής εφαρμογής δια­θεματικών προσεγγίσεων στους τομείς της Γλώσσας, τηc:; Ιστορίας και τωv Μαθηματικών, οε πραγματικέ<; σχολικές τάξεις. Έvα χρήσιμο εργαλείο για κάθε Εκπαιδευτικό που καλείται vα εφαρμόσει τα vέα διαθεματικά αvαλυτικά προγράμματα του ΥΠΕΠΘ.

Εκδόσεις Πανεπιστημίου Μακεδονίας τ 23 ιο 891.743. 741 . & 73 1 F 23Ιο 891.730

Ε [email protected], W uom.gr/uompress

Ε\Κ\ΕΙΔΗΣ Β ' τ. l /77

Page 80: Ευκλειδης Β 61

Ο Ευκλείδης προτείνει ... Ευκλείδη ... και Διόφαντο

11 11 11 Επειδή είναι Β'Μ = ΒΗ , Γ'Μ = ΓΗ , Α'Μ = ΑΗ το

σημείο Μ είναι ορθόκεντρο του τριγώνου Α'Β 'Γ. Η ευθεία Simson (s) του σημείου Μ ως προς το τρίγωνο ΑΒΓ διέρχεται από τις ορθές προβολές Μ1 , Μ2 , Μ3 του Μ πάνω στις

πλευρές ΒΓ, ΑΓ, ΑΒ αντίστοιχα και από το σημείο Τ. (Βασική ιδιότητα της ευθείας Simson) Ονομάζουμε Ρ 1 , Ρ2, Ρ3 τα σημεία τομής των πλευρών Α 'Β ' , Α 'Γ, Β 'Γ του τριγώνου Α 'Β 'Γ με τα ύψη ΓΖ, ΒΕ, ΑΔ του τριγώνου ΑΒΓ αντίστοιχα. Παρατηρούμε , ότι το σημείο Η ανήκει στον περιγεγραμμένο κύκλο C 'του τριγώνου Α'Β'Γ και ότι τα σημεία Ρ ι , Ρ2, Ρ3 είναι οι ορθές προβολές του Η πάνω στις πλευρές Α 'Β ' , Α 'Γ, Β 'Γτου Α'Β 'Γ. Άρα ,τα σημεία αυτά ανήκουν στην ευθεία SIMSON (s ') του Η ως προς το τρίγωνο Α 'Β 'Γ, που διέρχεται και αυτή από το μέσον Τ του τμήματος ΗΜ. Θα αποδείξουμε ότι s = s ' .

Επειδή το Τ είναι κοινό μέσον των Μ Ρ , Μ Ρ , είναι Μ Μ I I Ρ Ρ , οπότε ι 3 3 ι ι 3 ι 3 s I I s ' . Επειδή οι s, s ' έχουν κοινό το σημείο

Τ, ταυτίζονται.

Τα σημεία Μ1 , Ρ1 , Ρ2

είναι οι ορθές προβολές

του σημείου Α ' πάνω στις πλευρές ΒΓ, ΗΓ, ΗΒ του τριγώνου ΗΒΓ και επειδή είναι συνευθειακά, το σημείο Α ' ανήκει στον περιγεγραμμένο κύκλο του τριγώνου ΗΒΓ.

Όμοια, Τα σημεία Μ ,Ρ ,Ρ είναι οι ορθές 2 ι 3 προβολές του σημείου Β ' πάνω στις πλευρές ΑΓ, ΗΓ, ΗΑ του τριγώνου ΗΑΓ και επειδή είναι συνευθειακά, το σημείο Β ' ανήκει στον περιγεγραμμένο κύκλο του τριγώνου ΗΑΓ.

Τα σημεία Μ3 ,

Ρ2 ,

Ρ3

είναι οι ορθές προβολές

του σημείου Γ πάνω στις πλευρές ΑΒ, ΗΒ, ΗΑ του τριγώνου ΗΑΒ και επειδή είναι συνευθειακά, το σημείο Γ ανήκει στον περιγεγραμμένο κύκλο του τριγώνου ΗΑΒ.

Β. ΑΣΚΗΣΕΙΣ ΓΙΑ Λ ΥΣΗ 99. Έστω f μια συνάρτηση η οποία είναι συνεχής στο [0,1 ] και παραγωγίσιμη στο (0,1) Αν f(1)=0 και f(x)>O για κάθε x e (-1,1), να αποδείξετε ότι δοθέντων δυο

οποιωνδήποτε θετικών αριθμών α και β, υπάρχει αριθμός γ ε (-1 , 1) ώστε να ισχύει: α f(γ)f'(-γ)=βf'(γ) f(-γ) (Προτείνεται από τον Ακαδημαϊκό κ. Νικόλαο Αρτεμιάδη) 1 00. Έστω f, g: [α,β]� JR δυο συνεχείς συναρτήσεις ώστε για κάθε χ ε [α,β] να ισχύει f(x);:::O και g(x);:::O. Αν υπάρχει Α ε JR

με Α;:::Ο ώστε f(x) ::; A + fr(t)g(t)dt για κάθε

, , r g(t)d(t) χ ε [α,β] να αποδείξετε οτι: f(x) ::; Ae

(Προτείνεται από το συνάδελφο Κυριακόπουλο Θανάση - Αθήνα) 1 0 1 . Δίνεται ισοσκελές τρίγωνο ΑΒΓ(ΑΒ=ΑΓ). Α ν υπάρχουν σημεία Δ,Ε στις πλευρές ΑΓ ,ΑΒ του τριγώνου ώστε να ισχύει ΓΔ=ΔΕ=ΕΑ=ΒΓ, να υπολογίσετε τη γωνία Α του τριγώνου. (Προτείνεται από το συνάδελφο Γκουντουβά Σωτήρη - Αργυρούπολη) 1 02. Δίνονται δυο σταθερά σημεία Α,Β και μεταβλητό σημείο Ν. Α ν Ν 1 είναι το συμμετρικό του Ν ως προς το Α, Ν2 στην προέκταση της Ν1Β ώστε να ισχύει ΒΝι=2ΒΝz και Μι το μέσο της ΒΝ, να αποδείξετε ότι το κέντρο βάρους του τριγώνου Νι Nz Μι διατηρείται σταθερό, ανεξάρτητα από τη θέση του σημείου Ν. (Προτείνεται από τον χημικό Καρβελά Δημήτρη - Πεύκη Αττικής) Οι επιστολές των φίλων της στήλης που έφτασαν στα γραφεία της Ε.Μ.Ε. μετά την 1 51612006 δεν έχουν ληφθεί υπόψιν στο τεύχος αυτό.

Ελληνική Μαθηματική Εtαι.Qεία ΙΙΑΝΕΛΛΗΝΙΟ ΣΥΝΕΔΡΙΟ

ΜΑΘΗΜΑτΙΚΗΣ ΙΙΑΙΔΕΙΑΣ Με διεθνείς συμμετοχές

Πάτρα 24 - 25 - 26 Νοεμβρίου 2006

ΤΑ JIAAtiiiJIIATΙitA D %

ΠΟΑΙΤΙ%/ΙΙΟΖ. %ΤΟ 'L Y F X Po•o

ltO'LJIAO , Π ΡΟ8ΙιΤΑ% 8 Ι % 'L T II .

lt PIT Ιltll 'Lit.ΨII . Τ ΙΙ • 8 Π Ι Χ 8 Ι ΡΙΙΙΙΑΑΤΟΑΟFΙΑ ltAI

Τ 11• AΙ'LtiiiT Ι lt ll

ΕΥΚΛΕΙΔΗΣ Β ' τ.Ι/78

Page 81: Ευκλειδης Β 61

Τα Μαθηματικά μας δ:tαο:κε:ΟΟζουν Τα μαθηματικά αν και είναι επιστήμη που απαιτεί αυστηρή διατύπωση, έχουν τη μαγεία να

αποσπούν το ενδιαφέρον όλων των ανθρώπων. Επινοήσεις σε προβλήματα ή ασκήσεις με

κατάλληλο τρόπο διατυπωμένα εξάπτουν το πνεύμα, διεγείρουν τη φαντασία και κεντρίζουν την περιέργεια. Πρώτοι οι Αρχαίοι Έλληνες όπως ο Διόφαντος, ο Ζήνωνας κ.ά. μας δίδαξαν αυτά τα

μαθηματικά. Στη στήλη αυτή θα παρουσιάζουμε θέματα τα οποία δεν απαιτούν ιδιαίτερες μαθηματικές

γνώσεις αλλά μας διασκεδάζουν με την εκφώνησή τους ή τη λύση τους και είναι μια ευχάριστη

και συναρπαστική ασχολία •

Στο καταπληκτικό του μυθιστόρημα ο DAN BROWN αναφέρει τους αριθμούς Ι , Ι ,2,3 ,5 ,8 , 1 3 ,2 Ι που είναι 8 πρώτοι όροι μιας ακολουθίας. Προσθέστε τους 2 τελευταίους να βρείτε τον επόμενο, συνεχίστε. Τώρα διαιρέστε καθένα από αυτούς με τον προηγούμενο.

Η διαπίστωση είναι ότι όλα τα αποτελέσματα προσεγγίζουν τον αριθμό Ι ,6 Ι 803 . . . . . . που λέγεται Φ «χρυσή τομή». Αν τον aντιστρέψετε απλά θα χάσετε το ακέραιο μέρος του. Μήπως ο αριθμός αυτός κρύβει το Άγιο Δισκοπότηρο και δεν το κατάλαβε ο DAN BROWN;

Επιμέλεια: Παναγιώτης Χριστόπουλος

Το χαρτζιλίκι Ο πατέρας του Πέτρου μαθητή της Β ' Λυκείου για να διαπιστώσει αν ο υιός του είναι καλός στα μαθηματικά του είπε: <<Κάθε μέρα θα σου δίνω χαρτζιλίκι 100 Ευρώ και εσύ θα μου επιστρέφεις την lη μέρα 1 λεπτό, τη 2η μέρα 2 λεπτά, την 3η μέρα 4 λεπτά και κάθε μέρα διπλάσια λεπτά από την προηγούμενη. Αυτό θα γίνει από την πρώτη μέχρι την τελευταία μέρα του μήνα Οκτώβρη. » Τι λέτε πρέπει να δεχθεί ο Πέτρος ή όχι;

Τα χρήματα4 Ξέρετε ότι τα χρήματα κυκλοφορούν σε Ι€, σε 2 €, σε 5€ στις υποδιαιρέσεις δεκά'fου ( 1 0,20,50 λεπτά) και εκατοστού ( 1 ,2,5 λεπτά) κάθώς και στα πολλαπλάσια δεκάδων ( Ι Ο,20,50 €), εκατοντάδων ( Ι ΟΟ,200,500 €), σε νομίσματα άλλων χωρών έχουμε και πολλαπλάσια μεγαλύτερα. Να βρείτε τα λιγότερα τεμάχια χρημάτων, χωρίς να λείπει κανένα είδος, που δίνουν Ι 000 €. Α ν κυκλοφορούσε το € σε 1 , 3 , 5 θα είχαμε τα ίδια τεμάχια;

Ο Αριθμομνήμον Ζήτησε έναν 3ψήφιο, πολλαπλασίασε τον με

ΕΥΚΛΕΙΔΗΣ Β ' τ.Ι/79

Page 82: Ευκλειδης Β 61

Τα Μαθηματικά μας Διασκεδάζουν

έναν άλλο 3ψήφιο αριθμό που θα σου δώσουν, Ολα Ε ίναι 992 αλλά και με έναν 3ψήφιο αριθμό που θα σου Γράψτε ένα 3ψήφιο π.χ. 267 αλλάξτε τη σειρά δώσει ο φίλος σου και πές το άθροισμα των των ψηφίων, 762, τους αφαιρούμε δύο γινομένων, όλα αυτά από μνήμης ! ! ! 762-267=495 , στο αποτέλεσμα αλλάξτε τη

-Πώς; Εύκολο. Έστω ότι σας έδωσαν τον 764 χ 265 και 764χ 734 φίλος το άθροισμα των γινομένων είναι 763 236. (το μυστικό σσσσς ! ! ! Ο φίλος δίνει 734 διότι με το 265 θα έχουμε 999 και το αποτέλεσμα; 764 - l και τα αντίστοιχα 236 για να έχουμε 999.

Ο Κ\)βος και το Τ�:τράγωνο Ποιος είναι ο 2ψήφιος που είναι και τετράγωνο και κύβος; Υπάρχει και 3ψήφιος;

Τα κελι ιi Έχουμε ι 00 κελιά αριθμημένα από το ι έως το 1 00 και 1 00 δεσμοφύλακες με τους αριθμούς από το ι έως το 1 00. Ο κάθε δεσμοφύλακας περνάει από το πρώτο κελί και από τα κελιά που έχουν αριθμό πολ/σιο του αριθμού του. Στην αρχή όλα τα κελιά είναι κλειστά .Ο κάθε δεσμοφύλακας από το κάθε κελί που περνάει, αν το βρει κλειστό το ανοίγει, ενώ αν το βρει ανοικτό το κλείνει. Όταν περάσουν όλοι οι δεσμοφύλακες πόσα κελιά θα είναι ανοικτά; (Το έστειλε ο συνάδελφος Σωτήρης Γκουντουβάς).

Ο fΗΙΟtΗωτ ικός Σε ένα σχολείο διορίστηκε πρόσφατα νέος

μαθηματικός ο οποίος το χ 2 έτος θα είναι χ χρόνων, τι ηλικία έχει σήμερα;

Η Γη Αν υποθέσουμε ότι η ΔΕΗ έχει•ένα καλώδιο γύρω από τη Γη, στον Ισημερινό, πάνω σε πυλώνες ύψους 5 μέτρων και αποφασίζει να σκάψει και να το τοποθετήσει μέσα στη Γη σε βάθος 5 μέτρα. Πόσα μέτρα καλώδιο θα εξοικονομήσει;

σειρά, 594, τους προσθέτουμε 495+594= 089, αλλάζουμε πάλι τη σειρά στο αποτέλεσμα,

980 1 =99 2 • Δοκιμάστε με όποιο 3ψήφιο θέλετε.

\ Ι άντεψε Ζητήστε από το φίλο σας να γράψει ένα 3ψήφιο αριθμό και να επαναλάβει τα ψηφία του ώστε να γίνει 6ψήφιος. Ύστερα να διαιρέσει τον 6ψήφιο διαδοχικά με τους 7, ι ι , ι 3 και θα έχει τον αριθμό του.

ΊΌ παπιη\τσι Ζητήστε από το φίλο σας να γράψει σε ένα χαρτί το νούμερο του παπουτσιού του (π.χ.44) να του προσθέσει 5 μονάδες στη συνέχεια να πολλαπλασιάσει με το 50 από το αποτέλεσμα να αφαιρέσει το έτος γέννησης του και ύστερα να προσθέσει τον ι 76 ι , τώρα σας ενημερώνει μόνο για το αποτέλεσμα στο οποίο τα δύο πρώτα ψηφία είναι το ζητούμενο νούμερο.

Ο πατiφα..; Ο Γιώργος δεν είχε ούτε ένα Ευρώ και του έδωσε ο πατέρας του 200€. Η Ελένη επίσης δεν είχε ούτε ένα Ευρώ και της έδωσε ο πατέρας της ι 50€. Συμφώνησε ο Γιώργος με την Ελένη να αγοράσουν με τα χρήματά τους ένα ποδήλατο, πλήρωσαν ι 98€ και τους περίσσεψε από ι € πώς έγινε αυτό;

Ο φοιτητίtς Όταν η κόρη πέρασε σε Πανεπιστήμιο άλλης πόλης ο πατέρας της κατέθεσε σε μια τράπεζα ένα ποσό και της έδωσε το βιβλιάριο. Η κόρη κάθε μήνα έπαιρνε για τα έξοδά της ι 000 Ευρώ. Τον πρώτο μήνα ο πατέρας βλέποντας το ποσό που έμεινε το αύξησε κατά το ι /3 του. Αυτό έγινε τον επόμενο και το μεθεπόμενο μήνα τότε ο πατέρας διαπίστωσε ότι το ποσό ήταν το ίδιο με το αρχικό. Ποιο ποσό είχε καταθέσει αρχικά; Η Ηλικία Στον τύπο Α = (2p 1 ) 4 + p � + χ αντικαταστήστε τους p 1 , p 2 με πρώτους

αριθμούς μεγαλύτερους του 5 και το χ με την ηλικία σας. από το αποτέλεσμα μπορούμε να βρούμε την ηλικία σας, εσείς; (Το έστειλε ο

χημικός Δημήτριος Καρβελάς)

ΕΥΚΛΕΙΔΗΣ Β ' τ.Ι /80

Page 83: Ευκλειδης Β 61

, ,

α. , \ Ί ι α από τις εκδόσεις <<Εν Δυνάμει>>

Για την r· Λυκείου • Φυσική 1Κστεύ8υνσηc; (ταλαντώσεις - Κί.ιματα) - Γ. Θ. Ντοί.ιβαλης

• Φυσική Κcιτεύ8υνσηc; (Μηχανική του στερεοί.ι σώματος) - Γ. θ. Ντοί.ιβαλης

• Φυσική Κατεύβυνσηc; (Κροί.ισεις - Φαινόμενο Doppler) - Γ. θ. Ντοί.ιβαλης

• Ασκήσeιc; Βιολοyίας Γενικής Παιδείας - Κ. Ρ. Παπαζήσης

• Μα8πματικά Κστεύ8υνσης (Παράγωγοι) - Γ. & Π. Λουκόποuλος

• Μcιβnμcπ:ικά Κcπεύβυνσης (Ολοκληρώματα) - Γ. & Π. Λουκόποuλος

• ·εκφρασπ -·εκecση - Α. Καλλή - Γ. Σοuλτάνης

• Ανάπτuξπ εφαρμογών σe προyραμμσnστικό περιβάλλον - Κ. Ν. Ιορδανόποuλος

• Αρχές Οικονομικής θεωρίας - Κ. Γαροφαλάκης

• Φυσική - Κ. Ρ. Παπαζήσης

• "Άλycβρα - Γ. & Π. Λοuκόπουλος

ΕvΔuvάμειι Ε Κ Δ Ο Σ Ε Ι Σ

ΧΡΥΣΙΠΠΟΥ 1 & ΟΥΛΟΦ ΠΑΛΜΕ - ΖΩΓΡΑΦΟΥ Τηλ. : 2 1 Ο 74 88 030, fax: 2 1 Ο 74 83 031

:z: ·:a ... w::;

Page 84: Ευκλειδης Β 61

• ΕΛΛΗΝΙΚΉ ΜΑΘΗΜΑΤΙΚΉ ΕΤΑΙΡΕΙΑ � Πανεπιστημίου (Ελευθερίου Βενιζέλου) 34 - 1 06 79 ΑΘΗΝΑ

Τηλ. 3 6 1 6532 - 3 6 1 7784 - Fax: 364 1 025 · •

www.hms.gr - e-mail : [email protected]

Παίγνια και Λήψη Αποφάσεων Σελίδες 3 1 8

ISBN: 960-7341 -27-9. Σχήμα 1 9Χ23 Λ. Τ. 2 8 €

Το βιβλίο πραγματεύεται με ενιαίο τρόπο το αντικείμενο της Θεωρίας Παιγνίων και της κλασικής Θεωρίας Βελτιστοποίησης υπό το πρίσμα των εφαρμογών των θεωριών και των τεχνικών τους στα οικονομικά, τις πολιτικές επιστήμες και τη διοίκηση επιχειρήσεων. Αποφεύγοντας τη σχολαστικότητα, αλλά χωρίς να υστερεί σε μαθηματική αυστηρότητα, το βιβλίο απευθύνεται κυρίως σε προπτυχιακούς αλλά και μεταπτυχιακούς φοιτητές των οικονομικών, θετικών και τεχνολογικών σχολών, καθώς και σε όσους ενδιαφέρονται εν γένει για τη Θεωρία Παιγνίων και τη Θεωρία Αποφάσεων. Ο μεγάλος αριθμός των επεξεργασμένων παραδειγμάτων και των προς επίλυση ασκήσεων που καλύπτουν ένα ευρύτατο φάσμα εφαρμογών, εισάγουν βαθμιαία τον αναγνώστη στη μεθοδολογία και τη λογική της θεωρίας, και τον διδάσκουν πώς να αξιολογεί και να εφαρμόζει κατάλληλα τις σχετικές τεχνικές

ΗΡΩΝΟΣ ΑΛΕΞΑΝΔΡΕΩΣ

Ν Ε ΕΣ Ε ΚΔΟΣΕΙΣ ΕΛΛΗΝΙΚΗ ΜΑΘΗΜΑτtΚΗ ΕΤΑΙΡΕΙΑ

I •

, .. Χ. Δ� Αλιπρόντn και 5. Κ. Chakrabartl

Η Ελληνική Μαθηματική Εταιρεία με μεγάλη ικανοποίηση και υπερηφάνεια παραδίδει το πολύ σημαντικό αυτό έργο όχι μόνο στους Μαθηματικούς αλλά και σε όλους αυτούς που θέλουν να μελετήσουν σε βάθος το Ηρωνικό έργο.

·

Ο Αλεξανδρινός «μηχανικός» Ηρών εκτός από τις περίφημες πραγματείες του που αναφέρονται σε θέματα κατασκευής και λειτουργίας διαφόρων μηχανών, μας έδωσε και αξιόλογα μαθηματικά κείμενα.

&,!iJMJi@jjCJ'\If.Λ6f?· .. · , .... -----, __ -· 4&!

ΗΡΩΝΟΣ ΑΛΕΞΑΝΔΡΕΩΣ

ΟΝΟΜΑΤΑ ΓΕΩΜΕΤΡΙΚΩΝ

ΟΡΩΝ ΓΕΩΜΕΤΡΙΚΆ

ΗΡΩΝΟΣ ΑΛΕΞΑΝΔΡΕΩΣ

ΜΕΤΡ! ΚΑ - ΔΙΟΠΤΡΑ

ΗΡΩΝΟΣ ΑΛΕΕΑΝΔΡΕΩΣ

ΣΤΕΡΕΟιΊ4ΕrΡΙΚΑ

Σχήμα 1 7Χ24 Σελίδες 5 1 6

ISBN: 960-734 1 - 1 6-3 Λ.Τ. 2 0 €

Σχήμα 1 7Χ24 Σελίδες 328

ISBN: 960-7341 -22-8 Λ.Τ. 20 €

Σχήμα 1 7Χ24 Σελίδες 4 1 4

ISBN: 960-7341 -26-0 Λ.Τ. 20 €

ΑΣΤΡΟΛΑΒΟΣ

Σχήμα 1 7Χ24 Σελίδες 1 20

ISSN: 1 1 06-2878 Λ. Τ. 10 €

ΑΣΤΡΟΜΒΟΣ Στο περιοδικό ΑΣΤΡΟΛΑΒΟΣ δημοσιεύονται επιστημονικές εργασίες πάνω σε

σύγχρονα θέματα που εμπίπτουν στο περιεχόμενο και στους στόχους του όπως: Διδακτική και Νέες Τεχνολογίες, Εξ Αποστάσεως Εκπαίδευση, Τεχνητή Νοημοσύνη, Προγραμματισμός, Τεχνολογίες Διαδικτύου, Ηλεκτρονικό Εμπόριο, Λειτουργικά Συστήματα, Αρχιτεκτονική Υπολογιστών, Αριθμητική Ανάλυση, Υπολογιστικά Μαθηματικά και Εφαρμογές, Αλγόριθμοι, Θεωρία Πληροφοριών, Υπολογιστική Γεωμετρία κ.λ.π . .

Ο κοινός τόπος των άρθρων που θα δημοσιεύονται είναι: • Η συμβολή των Μαθηματικών στη θεμελίωση και εξέλιξη της Επιστήμης της

Πληροφορικής. • Η παιδαγωγική και διδακτική αξιοποίηση των Νέων Τεχνολογιών στην εν γένει

εκπαιδευτική διαδικασία.

Συνάδελφοι Επισκεφθείτε την ιστοσελίδα της Ε.Μ.Ε. (www.hms.gr ) όπου υπάρχουν όλα τα τεύχη του Ευκλέιδη Γ και της Μ αθηματικής Επιθεώρησης με τα περιεχόμενά τους καθ(ος και όλες οι εκδόσεις της Ελληνικς Μαθηματικής Εταιρείας (Ε.Μ.Ε.).